Search Results

Search found 2650 results on 106 pages for 'pm 77 1'.

Page 81/106 | < Previous Page | 77 78 79 80 81 82 83 84 85 86 87 88  | Next Page >

  • Why does fprintf start printing out of order or not at all?

    - by Steve Melvin
    This code should take an integer, create pipes, spawn two children, wait until they are dead, and start all over again. However, around the third time around the loop I lose my prompt to enter a number and it no longer prints the number I've entered. Any ideas? #include <stdio.h> #include <stdlib.h> #include <unistd.h> #include <errno.h> #define WRITE 1 #define READ 0 int main (int argc, const char * argv[]) { //Pipe file-descriptor array unsigned int isChildA = 0; int pipeA[2]; int pipeB[2]; int num = 0; while(1){ fprintf(stderr,"Enter an integer: "); scanf("%i", &num); if(num == 0){ fprintf(stderr,"You entered zero, exiting...\n"); exit(0); } //Open Pipes if(pipe(pipeA) < 0){ fprintf(stderr,"Could not create pipe A.\n"); exit(1); } if(pipe(pipeB) < 0){ fprintf(stderr,"Could not create pipe B.\n"); exit(1); } fprintf(stderr,"Value read: %i \n", num); fprintf(stderr,"Parent PID: %i\n", getpid()); pid_t procID = fork(); switch (procID) { case -1: fprintf(stderr,"Fork error, quitting...\n"); exit(1); break; case 0: isChildA = 1; break; default: procID = fork(); if (procID<0) { fprintf(stderr,"Fork error, quitting...\n"); exit(1); } else if(procID == 0){ isChildA = 0; } else { write(pipeA[WRITE], &num, sizeof(int)); close(pipeA[WRITE]); close(pipeA[READ]); close(pipeB[WRITE]); close(pipeB[READ]); pid_t pid; while (pid = waitpid(-1, NULL, 0)) { if (errno == ECHILD) { break; } } } break; } if (procID == 0) { //We're a child, do kid-stuff. ssize_t bytesRead = 0; int response; while (1) { while (bytesRead == 0) { bytesRead = read((isChildA?pipeA[READ]:pipeB[READ]), &response, sizeof(int)); } if (response < 2) { //Kill other child and self fprintf(stderr, "Terminating PROCID: %i\n", getpid()); write((isChildA?pipeB[WRITE]:pipeA[WRITE]), &response, sizeof(int)); close(pipeA[WRITE]); close(pipeA[READ]); close(pipeB[WRITE]); close(pipeB[READ]); return 0; } else if(!(response%2)){ //Even response/=2; fprintf(stderr,"PROCID: %i, VALUE: %i\n", getpid(), response); write((isChildA?pipeB[WRITE]:pipeA[WRITE]), &response, sizeof(int)); bytesRead = 0; } else { //Odd response*=3; response++; fprintf(stderr,"PROCID: %i, VALUE: %i\n", getpid(), response); write((isChildA?pipeB[WRITE]:pipeA[WRITE]), &response, sizeof(int)); bytesRead = 0; } } } } return 0; } This is the output I am getting... bash-3.00$ ./proj2 Enter an integer: 101 Value read: 101 Parent PID: 9379 PROCID: 9380, VALUE: 304 PROCID: 9381, VALUE: 152 PROCID: 9380, VALUE: 76 PROCID: 9381, VALUE: 38 PROCID: 9380, VALUE: 19 PROCID: 9381, VALUE: 58 PROCID: 9380, VALUE: 29 PROCID: 9381, VALUE: 88 PROCID: 9380, VALUE: 44 PROCID: 9381, VALUE: 22 PROCID: 9380, VALUE: 11 PROCID: 9381, VALUE: 34 PROCID: 9380, VALUE: 17 PROCID: 9381, VALUE: 52 PROCID: 9380, VALUE: 26 PROCID: 9381, VALUE: 13 PROCID: 9380, VALUE: 40 PROCID: 9381, VALUE: 20 PROCID: 9380, VALUE: 10 PROCID: 9381, VALUE: 5 PROCID: 9380, VALUE: 16 PROCID: 9381, VALUE: 8 PROCID: 9380, VALUE: 4 PROCID: 9381, VALUE: 2 PROCID: 9380, VALUE: 1 Terminating PROCID: 9381 Terminating PROCID: 9380 Enter an integer: 102 Value read: 102 Parent PID: 9379 PROCID: 9386, VALUE: 51 PROCID: 9387, VALUE: 154 PROCID: 9386, VALUE: 77 PROCID: 9387, VALUE: 232 PROCID: 9386, VALUE: 116 PROCID: 9387, VALUE: 58 PROCID: 9386, VALUE: 29 PROCID: 9387, VALUE: 88 PROCID: 9386, VALUE: 44 PROCID: 9387, VALUE: 22 PROCID: 9386, VALUE: 11 PROCID: 9387, VALUE: 34 PROCID: 9386, VALUE: 17 PROCID: 9387, VALUE: 52 PROCID: 9386, VALUE: 26 PROCID: 9387, VALUE: 13 PROCID: 9386, VALUE: 40 PROCID: 9387, VALUE: 20 PROCID: 9386, VALUE: 10 PROCID: 9387, VALUE: 5 PROCID: 9386, VALUE: 16 PROCID: 9387, VALUE: 8 PROCID: 9386, VALUE: 4 PROCID: 9387, VALUE: 2 PROCID: 9386, VALUE: 1 Terminating PROCID: 9387 Terminating PROCID: 9386 Enter an integer: 104 Value read: 104 Parent PID: 9379 Enter an integer: PROCID: 9388, VALUE: 52 PROCID: 9389, VALUE: 26 PROCID: 9388, VALUE: 13 PROCID: 9389, VALUE: 40 PROCID: 9388, VALUE: 20 PROCID: 9389, VALUE: 10 PROCID: 9388, VALUE: 5 PROCID: 9389, VALUE: 16 PROCID: 9388, VALUE: 8 PROCID: 9389, VALUE: 4 PROCID: 9388, VALUE: 2 PROCID: 9389, VALUE: 1 Terminating PROCID: 9388 Terminating PROCID: 9389 105 Value read: 105 Parent PID: 9379 Enter an integer: PROCID: 9395, VALUE: 316 PROCID: 9396, VALUE: 158 PROCID: 9395, VALUE: 79 PROCID: 9396, VALUE: 238 PROCID: 9395, VALUE: 119 PROCID: 9396, VALUE: 358 PROCID: 9395, VALUE: 179 PROCID: 9396, VALUE: 538 PROCID: 9395, VALUE: 269 PROCID: 9396, VALUE: 808 PROCID: 9395, VALUE: 404 PROCID: 9396, VALUE: 202 PROCID: 9395, VALUE: 101 PROCID: 9396, VALUE: 304 PROCID: 9395, VALUE: 152 PROCID: 9396, VALUE: 76 PROCID: 9395, VALUE: 38 PROCID: 9396, VALUE: 19 PROCID: 9395, VALUE: 58 PROCID: 9396, VALUE: 29 PROCID: 9395, VALUE: 88 PROCID: 9396, VALUE: 44 PROCID: 9395, VALUE: 22 PROCID: 9396, VALUE: 11 PROCID: 9395, VALUE: 34 PROCID: 9396, VALUE: 17 PROCID: 9395, VALUE: 52 PROCID: 9396, VALUE: 26 PROCID: 9395, VALUE: 13 PROCID: 9396, VALUE: 40 PROCID: 9395, VALUE: 20 PROCID: 9396, VALUE: 10 PROCID: 9395, VALUE: 5 PROCID: 9396, VALUE: 16 PROCID: 9395, VALUE: 8 PROCID: 9396, VALUE: 4 PROCID: 9395, VALUE: 2 PROCID: 9396, VALUE: 1 Terminating PROCID: 9395 Terminating PROCID: 9396 105 Value read: 105 Parent PID: 9379 Enter an integer: PROCID: 9397, VALUE: 316 PROCID: 9398, VALUE: 158 PROCID: 9397, VALUE: 79 PROCID: 9398, VALUE: 238 PROCID: 9397, VALUE: 119 PROCID: 9398, VALUE: 358 PROCID: 9397, VALUE: 179 PROCID: 9398, VALUE: 538 PROCID: 9397, VALUE: 269 PROCID: 9398, VALUE: 808 PROCID: 9397, VALUE: 404 PROCID: 9398, VALUE: 202 PROCID: 9397, VALUE: 101 PROCID: 9398, VALUE: 304 PROCID: 9397, VALUE: 152 PROCID: 9398, VALUE: 76 PROCID: 9397, VALUE: 38 PROCID: 9398, VALUE: 19 PROCID: 9397, VALUE: 58 PROCID: 9398, VALUE: 29 PROCID: 9397, VALUE: 88 PROCID: 9398, VALUE: 44 PROCID: 9397, VALUE: 22 PROCID: 9398, VALUE: 11 PROCID: 9397, VALUE: 34 PROCID: 9398, VALUE: 17 PROCID: 9397, VALUE: 52 PROCID: 9398, VALUE: 26 PROCID: 9397, VALUE: 13 PROCID: 9398, VALUE: 40 PROCID: 9397, VALUE: 20 PROCID: 9398, VALUE: 10 PROCID: 9397, VALUE: 5 PROCID: 9398, VALUE: 16 PROCID: 9397, VALUE: 8 PROCID: 9398, VALUE: 4 PROCID: 9397, VALUE: 2 PROCID: 9398, VALUE: 1 Terminating PROCID: 9397 Terminating PROCID: 9398 106 Value read: 106 Parent PID: 9379 Enter an integer: PROCID: 9399, VALUE: 53 PROCID: 9400, VALUE: 160 PROCID: 9399, VALUE: 80 PROCID: 9400, VALUE: 40 PROCID: 9399, VALUE: 20 PROCID: 9400, VALUE: 10 PROCID: 9399, VALUE: 5 PROCID: 9400, VALUE: 16 PROCID: 9399, VALUE: 8 PROCID: 9400, VALUE: 4 PROCID: 9399, VALUE: 2 PROCID: 9400, VALUE: 1 Terminating PROCID: 9399 Terminating PROCID: 9400 ^C Another thing that's strange, when ran from within XCode it behaves normally. However, when ran from bash on Solaris or OSX it acts up.

    Read the article

  • SVG as CSS background for website navigation-bar

    - by Irfan Mir
    I drew a small (horizontal / in width) svg to be the background of my website's navigation. My website's navigation takes place a 100% of the browser's viewport and I want the svg image to fill that 100% space. So, using css I set the background of the navigation (.nav) to nav.svg but then I saw (whenI opened the html file in a browser) that the svg was not the full-width of the nav, but at the small width I drew it at. How can I get the SVG to stretch and fill the entire width of the navigation (100% of the page) ? Here is the code for the html file where the navigation is in: <!DOCTYPE HTML PUBLIC "-//W3C//DTD HTML 4.01//EN" "http://www.w3.org/TR/html4/strict.dtd"> <html lang="en"><head><meta http-equiv="Content-Type" content="text/html; charset=utf-8"> <title>Distributed Horizontal Menu</title> <meta name="generator" content="PSPad editor, www.pspad.com"> <style type="text/css"> *{ margin:0; padding:0; } .nav { margin:0; padding:0; min-width:42em; width:100%; height:47px; overflow:hidden; background:transparent url(nav.svg) no-repeat; text-align:justify; font:bold 88%/1.1 verdana; } .nav li { display:inline; list-style:none; } .nav li.last { margin-right:100%; } .nav li a { display:inline-block; padding:13px 4px 0; height:31px; color:#fff; vertical-align:middle; text-decoration:none; } .nav li a:hover { color:#ff6; background:#36c; } @media screen and (max-width:322px){ /* styling causing first break will go here*/ /* but in the meantime, a test */ body{ background:#ff0000; } } </style></head><body> <ul class="nav"> <!--[test to comment out random items] <li>&nbsp; <a href="#">netscape&nbsp;9</a></li> [the spacing should be distributed]--> <li>&nbsp; <a href="#">internet&nbsp;explorer&nbsp;6-8</a></li> <li>&nbsp; <a href="#">opera&nbsp;10</a></li> <li>&nbsp; <a href="#">firefox&nbsp;3</a></li> <li>&nbsp; <a href="#">safari&nbsp;4</a></li> <li class="last">&nbsp; <a href="#">chrome&nbsp;2</a> &nbsp; &nbsp;</li> </ul> </body></html> and Here is the code for the svg: <?xml version="1.0" encoding="utf-8"?> <!DOCTYPE svg PUBLIC "-//W3C//DTD SVG 1.1//EN" "http://www.w3.org/Graphics/SVG/1.1/DTD/svg11.dtd"> <svg version="1.1" id="Layer_1" xmlns="http://www.w3.org/2000/svg" xmlns:xlink="http://www.w3.org/1999/xlink" x="0px" y="0px" width="321.026px" height="44.398px" viewBox="39.487 196.864 321.026 44.398" enable-background="new 39.487 196.864 321.026 44.398" xml:space="preserve"> <linearGradient id="SVGID_1_" gradientUnits="userSpaceOnUse" x1="280" y1="316.8115" x2="280" y2="275.375" gradientTransform="matrix(1 0 0 1 -80 -77)"> <stop offset="0" style="stop-color:#5A4A6A"/> <stop offset="0.3532" style="stop-color:#605170"/> <stop offset="0.8531" style="stop-color:#726382"/> <stop offset="1" style="stop-color:#796A89"/> </linearGradient> <path fill="url(#SVGID_1_)" d="M360,238.721c0,1.121-0.812,2.029-1.812,2.029H41.813c-1.001,0-1.813-0.908-1.813-2.029v-39.316 c0-1.119,0.812-2.027,1.813-2.027h316.375c1.002,0,1.812,0.908,1.812,2.027V238.721z"/> <path opacity="0.1" fill="#FFFFFF" enable-background="new " d="M358.188,197.376H41.813c-1.001,0-1.813,0.908-1.813,2.028 v39.316c0,1.12,0.812,2.028,1.813,2.028h316.375c1,0,1.812-0.908,1.812-2.028v-39.316C360,198.284,359.189,197.376,358.188,197.376z M358.75,238.721c0,0.415-0.264,0.779-0.562,0.779H41.813c-0.3,0-0.563-0.363-0.563-0.779v-39.316c0-0.414,0.263-0.777,0.563-0.777 h316.375c0.301,0,0.562,0.363,0.562,0.777V238.721z"/> <path opacity="0.5" fill="#FFFFFF" enable-background="new " d="M358.188,197.376H41.813c-1.001,0-1.813,0.908-1.813,2.028v1.461 c0-1.12,0.812-2.028,1.813-2.028h316.375c1.002,0,1.812,0.908,1.812,2.028v-1.461C360,198.284,359.189,197.376,358.188,197.376z"/> <g id="seperators"> <line fill="none" stroke="#000000" stroke-width="1.0259" stroke-miterlimit="10" x1="104.5" y1="197.375" x2="104.5" y2="240.75"/> <line opacity="0.1" fill="none" stroke="#FFFFFF" stroke-width="1.0259" stroke-miterlimit="10" enable-background="new " x1="103.5" y1="197.375" x2="103.5" y2="240.75"/> <line opacity="0.1" fill="none" stroke="#FFFFFF" stroke-width="1.0259" stroke-miterlimit="10" enable-background="new " x1="105.5" y1="197.375" x2="105.5" y2="240.75"/> <line fill="none" stroke="#000000" stroke-width="1.0259" stroke-miterlimit="10" x1="167.5" y1="197.375" x2="167.5" y2="240.75"/> <line opacity="0.1" fill="none" stroke="#FFFFFF" stroke-width="1.0259" stroke-miterlimit="10" enable-background="new " x1="166.5" y1="197.375" x2="166.5" y2="240.75"/> <line opacity="0.1" fill="none" stroke="#FFFFFF" stroke-width="1.0259" stroke-miterlimit="10" enable-background="new " x1="168.5" y1="197.375" x2="168.5" y2="240.75"/> <line fill="none" stroke="#000000" stroke-width="1.0259" stroke-miterlimit="10" x1="231.5" y1="197.375" x2="231.5" y2="240.75"/> <line opacity="0.1" fill="none" stroke="#FFFFFF" stroke-width="1.0259" stroke-miterlimit="10" enable-background="new " x1="232.5" y1="197.375" x2="232.5" y2="240.75"/> <line opacity="0.1" fill="none" stroke="#FFFFFF" stroke-width="1.0259" stroke-miterlimit="10" enable-background="new " x1="230.5" y1="197.375" x2="230.5" y2="240.75"/> <line fill="none" stroke="#000000" stroke-width="1.0259" stroke-miterlimit="10" x1="295.5" y1="197.375" x2="295.5" y2="240.75"/> <line opacity="0.1" fill="none" stroke="#FFFFFF" stroke-width="1.0259" stroke-miterlimit="10" enable-background="new " x1="294.5" y1="197.375" x2="294.5" y2="240.75"/> <line opacity="0.1" fill="none" stroke="#FFFFFF" stroke-width="1.0259" stroke-miterlimit="10" enable-background="new " x1="296.5" y1="197.375" x2="296.5" y2="240.75"/> </g> <path fill="none" stroke="#000000" stroke-width="1.0259" stroke-miterlimit="10" d="M360,238.721c0,1.121-0.812,2.029-1.812,2.029 H41.813c-1.001,0-1.813-0.908-1.813-2.029v-39.316c0-1.119,0.812-2.027,1.813-2.027h316.375c1.002,0,1.812,0.908,1.812,2.027 V238.721z"/> </svg> I appreciate and welcome any and all comments, help, and suggestions. Thanks in Advance!

    Read the article

  • Android 1.5 Gridview Problem,Pls help me.Thanks

    - by flybirdtt
    I used a gridview in this app.The xml file like this: When i run it at Verison 1.6 or newer. it's ok. But i can not get this gridview in 1.5 I always show this info and exception: Unable to resolve drawable "com.android.layoutlib.utils.DensityBasedResourceValue@397660" in attribute "listSelector" org.xmlpull.v1.XmlPullParserException: Binary XML file line #3: tag requires a 'drawable' attribute or child tag defining a drawable at android.graphics.drawable.StateListDrawable.inflate(StateListDrawable.java:151) at android.graphics.drawable.Drawable.createFromXmlInner(Drawable.java:779) at android.graphics.drawable.Drawable.createFromXml(Drawable.java:720) at com.android.layoutlib.bridge.ResourceHelper.getDrawable(ResourceHelper.java:150) at com.android.layoutlib.bridge.BridgeTypedArray.getDrawable(BridgeTypedArray.java:668) at android.widget.AbsListView.(AbsListView.java:514) at android.widget.GridView.(GridView.java:69) at android.widget.GridView.(GridView.java:65) at sun.reflect.NativeConstructorAccessorImpl.newInstance0(Native Method) at sun.reflect.NativeConstructorAccessorImpl.newInstance(NativeConstructorAccessorImpl.java:39) at sun.reflect.DelegatingConstructorAccessorImpl.newInstance(DelegatingConstructorAccessorImpl.java:27) at java.lang.reflect.Constructor.newInstance(Constructor.java:513) at android.view.LayoutInflater.createView(LayoutInflater.java:499) at android.view.BridgeInflater.onCreateView(BridgeInflater.java:77) at android.view.LayoutInflater.createViewFromTag(LayoutInflater.java:562) at android.view.BridgeInflater.createViewFromTag(BridgeInflater.java:122) at android.view.LayoutInflater.rInflate(LayoutInflater.java:617) at android.view.LayoutInflater.rInflate(LayoutInflater.java:620) at android.view.LayoutInflater.inflate(LayoutInflater.java:407) at android.view.LayoutInflater.inflate(LayoutInflater.java:296) at com.android.layoutlib.bridge.Bridge.computeLayout(Bridge.java:377) at com.android.ide.eclipse.adt.internal.editors.layout.gle1.GraphicalLayoutEditor.computeLayout(Unknown Source) at com.android.ide.eclipse.adt.internal.editors.layout.gle1.GraphicalLayoutEditor.recomputeLayout(Unknown Source) at com.android.ide.eclipse.adt.internal.editors.layout.gle1.GraphicalLayoutEditor.activated(Unknown Source) at com.android.ide.eclipse.adt.internal.editors.layout.LayoutEditor.pageChange(Unknown Source) at org.eclipse.ui.part.MultiPageEditorPart.setActivePage(MultiPageEditorPart.java:1076) at org.eclipse.ui.forms.editor.FormEditor.setActivePage(FormEditor.java:601) at com.android.ide.eclipse.adt.internal.editors.AndroidEditor.selectDefaultPage(Unknown Source) at com.android.ide.eclipse.adt.internal.editors.AndroidEditor.addPages(Unknown Source) at org.eclipse.ui.forms.editor.FormEditor.createPages(FormEditor.java:138) at org.eclipse.ui.part.MultiPageEditorPart.createPartControl(MultiPageEditorPart.java:357) at org.eclipse.ui.internal.EditorReference.createPartHelper(EditorReference.java:662) at org.eclipse.ui.internal.EditorReference.createPart(EditorReference.java:462) at org.eclipse.ui.internal.WorkbenchPartReference.getPart(WorkbenchPartReference.java:595) at org.eclipse.ui.internal.PartPane.setVisible(PartPane.java:313) at org.eclipse.ui.internal.presentations.PresentablePart.setVisible(PresentablePart.java:180) at org.eclipse.ui.internal.presentations.util.PresentablePartFolder.select(PresentablePartFolder.java:270) at org.eclipse.ui.internal.presentations.util.LeftToRightTabOrder.select(LeftToRightTabOrder.java:65) at org.eclipse.ui.internal.presentations.util.TabbedStackPresentation.selectPart(TabbedStackPresentation.java:473) at org.eclipse.ui.internal.PartStack.refreshPresentationSelection(PartStack.java:1256) at org.eclipse.ui.internal.PartStack.setSelection(PartStack.java:1209) at org.eclipse.ui.internal.PartStack.showPart(PartStack.java:1608) at org.eclipse.ui.internal.PartStack.add(PartStack.java:499) at org.eclipse.ui.internal.EditorStack.add(EditorStack.java:103) at org.eclipse.ui.internal.PartStack.add(PartStack.java:485) at org.eclipse.ui.internal.EditorStack.add(EditorStack.java:112) at org.eclipse.ui.internal.EditorSashContainer.addEditor(EditorSashContainer.java:63) at org.eclipse.ui.internal.EditorAreaHelper.addToLayout(EditorAreaHelper.java:225) at org.eclipse.ui.internal.EditorAreaHelper.addEditor(EditorAreaHelper.java:213) at org.eclipse.ui.internal.EditorManager.createEditorTab(EditorManager.java:778) at org.eclipse.ui.internal.EditorManager.openEditorFromDescriptor(EditorManager.java:677) at org.eclipse.ui.internal.EditorManager.openEditor(EditorManager.java:638) at org.eclipse.ui.internal.WorkbenchPage.busyOpenEditorBatched(WorkbenchPage.java:2854) at org.eclipse.ui.internal.WorkbenchPage.busyOpenEditor(WorkbenchPage.java:2762) at org.eclipse.ui.internal.WorkbenchPage.access$11(WorkbenchPage.java:2754) at org.eclipse.ui.internal.WorkbenchPage$10.run(WorkbenchPage.java:2705) at org.eclipse.swt.custom.BusyIndicator.showWhile(BusyIndicator.java:70) at org.eclipse.ui.internal.WorkbenchPage.openEditor(WorkbenchPage.java:2701) at org.eclipse.ui.internal.WorkbenchPage.openEditor(WorkbenchPage.java:2685) at org.eclipse.ui.internal.WorkbenchPage.openEditor(WorkbenchPage.java:2676) at org.eclipse.ui.ide.IDE.openEditor(IDE.java:651) at org.eclipse.ui.ide.IDE.openEditor(IDE.java:610) at org.eclipse.jdt.internal.ui.javaeditor.EditorUtility.openInEditor(EditorUtility.java:361) at org.eclipse.jdt.internal.ui.javaeditor.EditorUtility.openInEditor(EditorUtility.java:168) at org.eclipse.jdt.ui.actions.OpenAction.run(OpenAction.java:229) at org.eclipse.jdt.ui.actions.OpenAction.run(OpenAction.java:208) at org.eclipse.jdt.ui.actions.SelectionDispatchAction.dispatchRun(SelectionDispatchAction.java:274) at org.eclipse.jdt.ui.actions.SelectionDispatchAction.run(SelectionDispatchAction.java:250) at org.eclipse.jdt.internal.ui.packageview.PackageExplorerActionGroup.handleOpen(PackageExplorerActionGroup.java:373) at org.eclipse.jdt.internal.ui.packageview.PackageExplorerPart$4.open(PackageExplorerPart.java:526) at org.eclipse.ui.OpenAndLinkWithEditorHelper$InternalListener.open(OpenAndLinkWithEditorHelper.java:48) at org.eclipse.jface.viewers.StructuredViewer$2.run(StructuredViewer.java:842) at org.eclipse.core.runtime.SafeRunner.run(SafeRunner.java:42) at org.eclipse.core.runtime.Platform.run(Platform.java:888) at org.eclipse.ui.internal.JFaceUtil$1.run(JFaceUtil.java:48) at org.eclipse.jface.util.SafeRunnable.run(SafeRunnable.java:175) at org.eclipse.jface.viewers.StructuredViewer.fireOpen(StructuredViewer.java:840) at org.eclipse.jface.viewers.StructuredViewer.handleOpen(StructuredViewer.java:1101) at org.eclipse.jface.viewers.StructuredViewer$6.handleOpen(StructuredViewer.java:1205) at org.eclipse.jface.util.OpenStrategy.fireOpenEvent(OpenStrategy.java:264) at org.eclipse.jface.util.OpenStrategy.access$2(OpenStrategy.java:258) at org.eclipse.jface.util.OpenStrategy$1.handleEvent(OpenStrategy.java:298) at org.eclipse.swt.widgets.EventTable.sendEvent(EventTable.java:84) at org.eclipse.swt.widgets.Widget.sendEvent(Widget.java:1003) at org.eclipse.swt.widgets.Display.runDeferredEvents(Display.java:3910) at org.eclipse.swt.widgets.Display.readAndDispatch(Display.java:3503) at org.eclipse.ui.internal.Workbench.runEventLoop(Workbench.java:2405) at org.eclipse.ui.internal.Workbench.runUI(Workbench.java:2369) at org.eclipse.ui.internal.Workbench.access$4(Workbench.java:2221) at org.eclipse.ui.internal.Workbench$5.run(Workbench.java:500) at org.eclipse.core.databinding.observable.Realm.runWithDefault(Realm.java:332) at org.eclipse.ui.internal.Workbench.createAndRunWorkbench(Workbench.java:493) at org.eclipse.ui.PlatformUI.createAndRunWorkbench(PlatformUI.java:149) at org.eclipse.ui.internal.ide.application.IDEApplication.start(IDEApplication.java:113) at org.eclipse.equinox.internal.app.EclipseAppHandle.run(EclipseAppHandle.java:194) at org.eclipse.core.runtime.internal.adaptor.EclipseAppLauncher.runApplication(EclipseAppLauncher.java:110) at org.eclipse.core.runtime.internal.adaptor.EclipseAppLauncher.start(EclipseAppLauncher.java:79) at org.eclipse.core.runtime.adaptor.EclipseStarter.run(EclipseStarter.java:368) at org.eclipse.core.runtime.adaptor.EclipseStarter.run(EclipseStarter.java:179) at sun.reflect.NativeMethodAccessorImpl.invoke0(Native Method) at sun.reflect.NativeMethodAccessorImpl.invoke(NativeMethodAccessorImpl.java:39) at sun.reflect.DelegatingMethodAccessorImpl.invoke(DelegatingMethodAccessorImpl.java:25) at java.lang.reflect.Method.invoke(Method.java:597) at org.eclipse.equinox.launcher.Main.invokeFramework(Main.java:559) at org.eclipse.equinox.launcher.Main.basicRun(Main.java:514) at org.eclipse.equinox.launcher.Main.run(Main.java:1311) This is the layout xml: &RelativeLayout xmlns:android="http://schemas.android.com/apk/res/android" android:layout_width="fill_parent" android:layout_height="fill_parent" android:background="@drawable/menu_background2"& &LinearLayout android:id="@+id/logopanel" android:orientation="vertical" android:layout_width="fill_parent" android:layout_height="wrap_content" android:layout_alignParentTop="true" android:layout_marginTop="50dip" android:gravity="center" android:layout_marginBottom="10dip"& &ImageButton android:id="@+id/searchbar" android:layout_width="fill_parent" android:layout_height="wrap_content" android:background="@drawable/fake_search_bar"&&/ImageButton& &/LinearLayout& &LinearLayout android:id="@+id/iconpanel" android:orientation="vertical" android:layout_width="fill_parent" android:layout_height="wrap_content" android:layout_below="@+id/logopanel" android:layout_above="@+id/allbotpanel" android:layout_marginTop="10dip"& &GridView android:id="@+id/gridcontent" android:layout_width="fill_parent" android:layout_height="wrap_content" android:gravity="center" android:numColumns="3" android:layout_gravity="center" android:layout_centerInParent="true" android:background="@drawable/transparent_backgroud" android:listSelector="@drawable/gridviewselector"& &/GridView& &/LinearLayout& &RelativeLayout android:id="@+id/allbotpanel" android:layout_width="fill_parent" android:layout_height="75dip" android:background="@drawable/amex_bottom_bar" android:layout_alignParentBottom="true"& &LinearLayout android:id="@+id/noticebar" android:orientation="horizontal" android:layout_width="fill_parent" android:layout_height="29dip" android:layout_above="@+id/homebottombar"& &ImageButton android:id="@+id/infoicon" android:layout_width="wrap_content" android:layout_height="wrap_content" android:layout_gravity="left|center_vertical" android:layout_marginLeft="10dip" android:background="@drawable/amex_info_button" android:src="@drawable/infoselector"&&/ImageButton& &TextView android:id="@+id/noticeicon" android:gravity="center" android:layout_gravity="right|center_vertical" android:layout_width="wrap_content" android:layout_height="25dip" android:layout_weight="1" android:clickable="true" android:focusable="true" android:text="@string/notice_string"&&/TextView& &/LinearLayout& &LinearLayout android:id="@+id/homebottombar" android:orientation="horizontal" android:layout_width="fill_parent" android:layout_height="46dip" android:gravity="center" android:layout_alignParentBottom="true" android:background="@drawable/amex_bottom_bar" android:layout_marginBottom="3dip"&&/LinearLayout& &/RelativeLayout& &/RelativeLayout&

    Read the article

  • What is the fastest cyclic synchronization in Java (ExecutorService vs. CyclicBarrier vs. X)?

    - by Alex Dunlop
    Which Java synchronization construct is likely to provide the best performance for a concurrent, iterative processing scenario with a fixed number of threads like the one outlined below? After experimenting on my own for a while (using ExecutorService and CyclicBarrier) and being somewhat surprised by the results, I would be grateful for some expert advice and maybe some new ideas. Existing questions here do not seem to focus primarily on performance, hence this new one. Thanks in advance! The core of the app is a simple iterative data processing algorithm, parallelized to the spread the computational load across 8 cores on a Mac Pro, running OS X 10.6 and Java 1.6.0_07. The data to be processed is split into 8 blocks and each block is fed to a Runnable to be executed by one of a fixed number of threads. Parallelizing the algorithm was fairly straightforward, and it functionally works as desired, but its performance is not yet what I think it could be. The app seems to spend a lot of time in system calls synchronizing, so after some profiling I wonder whether I selected the most appropriate synchronization mechanism(s). A key requirement of the algorithm is that it needs to proceed in stages, so the threads need to sync up at the end of each stage. The main thread prepares the work (very low overhead), passes it to the threads, lets them work on it, then proceeds when all threads are done, rearranges the work (again very low overhead) and repeats the cycle. The machine is dedicated to this task, Garbage Collection is minimized by using per-thread pools of pre-allocated items, and the number of threads can be fixed (no incoming requests or the like, just one thread per CPU core). V1 - ExecutorService My first implementation used an ExecutorService with 8 worker threads. The program creates 8 tasks holding the work and then lets them work on it, roughly like this: // create one thread per CPU executorService = Executors.newFixedThreadPool( 8 ); ... // now process data in cycles while( ...) { // package data into 8 work items ... // create one Callable task per work item ... // submit the Callables to the worker threads executorService.invokeAll( taskList ); } This works well functionally (it does what it should), and for very large work items indeed all 8 CPUs become highly loaded, as much as the processing algorithm would be expected to allow (some work items will finish faster than others, then idle). However, as the work items become smaller (and this is not really under the program's control), the user CPU load shrinks dramatically: blocksize | system | user | cycles/sec 256k 1.8% 85% 1.30 64k 2.5% 77% 5.6 16k 4% 64% 22.5 4096 8% 56% 86 1024 13% 38% 227 256 17% 19% 420 64 19% 17% 948 16 19% 13% 1626 Legend: - block size = size of the work item (= computational steps) - system = system load, as shown in OS X Activity Monitor (red bar) - user = user load, as shown in OS X Activity Monitor (green bar) - cycles/sec = iterations through the main while loop, more is better The primary area of concern here is the high percentage of time spent in the system, which appears to be driven by thread synchronization calls. As expected, for smaller work items, ExecutorService.invokeAll() will require relatively more effort to sync up the threads versus the amount of work being performed in each thread. But since ExecutorService is more generic than it would need to be for this use case (it can queue tasks for threads if there are more tasks than cores), I though maybe there would be a leaner synchronization construct. V2 - CyclicBarrier The next implementation used a CyclicBarrier to sync up the threads before receiving work and after completing it, roughly as follows: main() { // create the barrier barrier = new CyclicBarrier( 8 + 1 ); // create Runable for thread, tell it about the barrier Runnable task = new WorkerThreadRunnable( barrier ); // start the threads for( int i = 0; i < 8; i++ ) { // create one thread per core new Thread( task ).start(); } while( ... ) { // tell threads about the work ... // N threads + this will call await(), then system proceeds barrier.await(); // ... now worker threads work on the work... // wait for worker threads to finish barrier.await(); } } class WorkerThreadRunnable implements Runnable { CyclicBarrier barrier; WorkerThreadRunnable( CyclicBarrier barrier ) { this.barrier = barrier; } public void run() { while( true ) { // wait for work barrier.await(); // do the work ... // wait for everyone else to finish barrier.await(); } } } Again, this works well functionally (it does what it should), and for very large work items indeed all 8 CPUs become highly loaded, as before. However, as the work items become smaller, the load still shrinks dramatically: blocksize | system | user | cycles/sec 256k 1.9% 85% 1.30 64k 2.7% 78% 6.1 16k 5.5% 52% 25 4096 9% 29% 64 1024 11% 15% 117 256 12% 8% 169 64 12% 6.5% 285 16 12% 6% 377 For large work items, synchronization is negligible and the performance is identical to V1. But unexpectedly, the results of the (highly specialized) CyclicBarrier seem MUCH WORSE than those for the (generic) ExecutorService: throughput (cycles/sec) is only about 1/4th of V1. A preliminary conclusion would be that even though this seems to be the advertised ideal use case for CyclicBarrier, it performs much worse than the generic ExecutorService. V3 - Wait/Notify + CyclicBarrier It seemed worth a try to replace the first cyclic barrier await() with a simple wait/notify mechanism: main() { // create the barrier // create Runable for thread, tell it about the barrier // start the threads while( ... ) { // tell threads about the work // for each: workerThreadRunnable.setWorkItem( ... ); // ... now worker threads work on the work... // wait for worker threads to finish barrier.await(); } } class WorkerThreadRunnable implements Runnable { CyclicBarrier barrier; @NotNull volatile private Callable<Integer> workItem; WorkerThreadRunnable( CyclicBarrier barrier ) { this.barrier = barrier; this.workItem = NO_WORK; } final protected void setWorkItem( @NotNull final Callable<Integer> callable ) { synchronized( this ) { workItem = callable; notify(); } } public void run() { while( true ) { // wait for work while( true ) { synchronized( this ) { if( workItem != NO_WORK ) break; try { wait(); } catch( InterruptedException e ) { e.printStackTrace(); } } } // do the work ... // wait for everyone else to finish barrier.await(); } } } Again, this works well functionally (it does what it should). blocksize | system | user | cycles/sec 256k 1.9% 85% 1.30 64k 2.4% 80% 6.3 16k 4.6% 60% 30.1 4096 8.6% 41% 98.5 1024 12% 23% 202 256 14% 11.6% 299 64 14% 10.0% 518 16 14.8% 8.7% 679 The throughput for small work items is still much worse than that of the ExecutorService, but about 2x that of the CyclicBarrier. Eliminating one CyclicBarrier eliminates half of the gap. V4 - Busy wait instead of wait/notify Since this app is the primary one running on the system and the cores idle anyway if they're not busy with a work item, why not try a busy wait for work items in each thread, even if that spins the CPU needlessly. The worker thread code changes as follows: class WorkerThreadRunnable implements Runnable { // as before final protected void setWorkItem( @NotNull final Callable<Integer> callable ) { workItem = callable; } public void run() { while( true ) { // busy-wait for work while( true ) { if( workItem != NO_WORK ) break; } // do the work ... // wait for everyone else to finish barrier.await(); } } } Also works well functionally (it does what it should). blocksize | system | user | cycles/sec 256k 1.9% 85% 1.30 64k 2.2% 81% 6.3 16k 4.2% 62% 33 4096 7.5% 40% 107 1024 10.4% 23% 210 256 12.0% 12.0% 310 64 11.9% 10.2% 550 16 12.2% 8.6% 741 For small work items, this increases throughput by a further 10% over the CyclicBarrier + wait/notify variant, which is not insignificant. But it is still much lower-throughput than V1 with the ExecutorService. V5 - ? So what is the best synchronization mechanism for such a (presumably not uncommon) problem? I am weary of writing my own sync mechanism to completely replace ExecutorService (assuming that it is too generic and there has to be something that can still be taken out to make it more efficient). It is not my area of expertise and I'm concerned that I'd spend a lot of time debugging it (since I'm not even sure my wait/notify and busy wait variants are correct) for uncertain gain. Any advice would be greatly appreciated.

    Read the article

  • Send email with PHP script -> How to display mutated vowels?

    - by Sebi
    I use this php script to send an email. It works well, but german mutated vowels (ö,ä,ü, etc) are not displayed correctly. Any hints how to change that? <?php /* Geben Sie hier Ihre E-Mail Adresse zwischen den beiden " an: */ $_emails[0] = "[email protected]"; // Wenn keine $_POST Daten übermittelt wurden, dann abbrechen if(!isset($_POST) OR empty($_POST)) { header("Content-type: text/plain"); echo "Es wurden keine Daten übermittelt!"; exit; } else { // Datum, Uhrzeit und Pfad zum eigenen Script feststellen $date = date("d.m.Y"); $time = date("H:i"); $host = "http://" . $_SERVER['HTTP_HOST'] . $_SERVER['PHP_SELF']; // Empfänger feststellen und auf Gültigkeit prüfen if(isset($_POST['recipient']) AND isset($_emails[ $_POST['recipient'] ]) AND preg_match("/^.*@.*\..*$/", $_emails[ $_POST['recipient'] ])) { $recipient = $_emails[ $_POST['recipient'] ]; } // Wurde kein (gültiger) Empfänger angegeben, es mit $_email[0] versuchen elseif(isset($_emails[0]) AND preg_match("/^.*@.*\..*$/", $_emails[0])) { $recipient = $_emails[0]; } // Ist auch diese Adresse ungültig, mit Fehlermeldung abbrechen else { header("Content-type: text/plain"); echo "Fehler im Script - es wurde kein Empfänger oder eine ungültige E-Mail Adresse in \ angegeben."; exit; } // Wenn Betreff übermittelt, diesen verwenden if(isset($_POST['subject'])) { $subject = $_POST['subject']; } // sonst einen Default Betreff verwenden else { $subject = "Formular Daten von {$_SERVER['HTTP_HOST']}"; } // E-Mai Kopf generieren $email = "Formular Eintrag\n" . "\n" . "Am $date um $time Uhr hast das Script auf $host Formulardaten empfangen,\n" . "welche nach Angabe des Browsers von {$_SERVER['HTTP_REFERER']} stammen.\n" . "\n" . "Der Formular Inhalt wird nachfolgend wiedergegeben.\n" . "\n"; // Alle $_POST Werte an den E-Mail Kopf anhängen foreach($_POST as $key => $value) { if($key == "redirect" OR $key == "recipient" OR $key == "subject") { continue; } $email .= "Fomular Feld '$key':\n" . "=============================\n" . "$value\n" . "\n"; } // E-Mail Fuß anfügen $email .= "=============================\n" . "Ende der automatisch generierten E-Mail."; // Versuchen E-Mail zu versenden if(!mail($recipient, $subject, $email)) { // Ist dies gescheitert, Fehlermeldung ausgeben echo "Es ist ein Fehler beim Versenden der E-Mail aufgetreten," . " eventuell liegt ein Konfigurationsfehler am Server vor.\n\n"; exit; } // Wenn gewünscht, auf Bestätigungsseite weiterleiten if(isset($_POST['redirect']) AND preg_match("=^(http|ftp)://.*\..*$=", $_POST['redirect'])) { header("Location: ".$_POST['redirect']); exit; } else { header("Content-type: text/html"); echo "Die E-Mail wurde erfolgreich versendet."; echo '<br>'; echo '<a href="http://www.ovlu.li/cms/index.php?page=kontakt">Zurueck</a>'; exit; } } ?> So i followed the hint in the first answer and the code looks now the following: <?php /* Geben Sie hier Ihre E-Mail Adresse zwischen den beiden " an: */ $_emails[0] = "[email protected]"; // Wenn keine $_POST Daten übermittelt wurden, dann abbrechen if(!isset($_POST) OR empty($_POST)) { header("Content-type: text/plain; charset=utf-8"); echo "Es wurden keine Daten übermittelt!"; exit; } else { // Datum, Uhrzeit und Pfad zum eigenen Script feststellen $date = date("d.m.Y"); $time = date("H:i"); $host = "http://" . $_SERVER['HTTP_HOST'] . $_SERVER['PHP_SELF']; // Empfänger feststellen und auf Gültigkeit prüfen if(isset($_POST['recipient']) AND isset($_emails[ $_POST['recipient'] ]) AND preg_match("/^.*@.*\..*$/", $_emails[ $_POST['recipient'] ])) { $recipient = $_emails[ $_POST['recipient'] ]; } // Wurde kein (gültiger) Empfänger angegeben, es mit $_email[0] versuchen elseif(isset($_emails[0]) AND preg_match("/^.*@.*\..*$/", $_emails[0])) { $recipient = $_emails[0]; } // Ist auch diese Adresse ungültig, mit Fehlermeldung abbrechen else { header("Content-type: text/plain"); echo "Fehler im Script - es wurde kein Empfänger oder eine ungültige E-Mail Adresse in \ angegeben."; exit; } // Wenn Betreff übermittelt, diesen verwenden if(isset($_POST['subject'])) { $subject = $_POST['subject']; } // sonst einen Default Betreff verwenden else { $subject = "Formular Daten von {$_SERVER['HTTP_HOST']}"; } // E-Mai Kopf generieren $email = "Formular Eintrag\n" . "\n" . "Am $date um $time Uhr hast das Script auf $host Formulardaten empfangen,\n" . "welche nach Angabe des Browsers von {$_SERVER['HTTP_REFERER']} stammen.\n" . "\n" . "Der Formular Inhalt wird nachfolgend wiedergegeben.\n" . "\n"; // Alle $_POST Werte an den E-Mail Kopf anhängen foreach($_POST as $key => $value) { if($key == "redirect" OR $key == "recipient" OR $key == "subject") { continue; } $email .= "Fomular Feld '$key':\n" . "=============================\n" . "$value\n" . "\n"; } // E-Mail Fuß anfügen $email .= "=============================\n" . "Ende der automatisch generierten E-Mail."; $email = htmlentities($email, ENT_QUOTES, 'uft-8'); // Versuchen E-Mail zu versenden if(!mail($recipient, $subject, $email)) { // Ist dies gescheitert, Fehlermeldung ausgeben echo "Es ist ein Fehler beim Versenden der E-Mail aufgetreten," . " eventuell liegt ein Konfigurationsfehler am Server vor.\n\n"; exit; } // Wenn gewünscht, auf Bestätigungsseite weiterleiten if(isset($_POST['redirect']) AND preg_match("=^(http|ftp)://.*\..*$=", $_POST['redirect'])) { header("Location: ".$_POST['redirect']); exit; } // sonst eine Bestätigung ausgeben else { header("Content-type: text/html"); echo "Die E-Mail wurde erfolgreich versendet."; echo '<br>'; echo '<a href="http://foto.roser.li/admin/index.php?page=kontakt">Zurueck</a>'; exit; } } ?> Now when I send the email, the following message is displayed: > Warning: htmlentities(): charset > `uft-8' not supported, assuming > iso-8859-1 in > /home/www/web21/html/roser/foto/admin/mail.php > on line 77 Die E-Mail wurde > erfolgreich versendet.

    Read the article

  • java.lang.ArrayIndexOutOfBoundsException

    - by thefonso
    Here is the code. import java.applet.Applet; import java.awt.Button; import java.awt.Color; import java.awt.Graphics; import java.awt.event.ActionEvent; import java.awt.event.ActionListener; public class GuessingGame extends Applet{ /** * */ private static final long serialVersionUID = 1L; private final int START_X = 20; private final int START_Y = 40; private final int ROWS = 4; private final int COLS = 4; private final int BOX_WIDTH = 20; private final int BOX_HEIGHT = 20; //this is used to keep track of boxes that have been matched. private boolean matchedBoxes[][]; //this is used to keep track of two boxes that have been clicked. private MaskableBox chosenBoxes[]; private MaskableBox boxes[][]; private Color boxColors[][]; private Button resetButton; public void init() { boxes = new MaskableBox[ROWS][COLS]; boxColors = new Color[ROWS][COLS]; resetButton = new Button("Reset Colors"); resetButton.addActionListener(new ActionListener() { public void actionPerformed(ActionEvent e) { randomizeColors(); buildBoxes(); repaint(); } }); add(resetButton); //separate building colors so we can add a button later //to re-randomize them. randomizeColors(); buildBoxes(); } public void paint(Graphics g) { for (int row =0; row < boxes.length; row ++) { for (int col = 0; col < boxes[row].length; col++) { if(boxes[row][col].isClicked()) { //boxes[row][col].setMaskColor(Color.black); //boxes[row][col].setMask(!boxes[row][col].isMask()); //boxes[row][col].setClicked(false); //} if (!matchedBoxes[row][col]) { gameLogic(boxes[row][col]); //boxes[row][col].draw(g); } } } } //loop through the boxes and draw them. for (int row = 0; row < boxes.length; row++) { for (int col = 0; col < boxes[row].length; col++) { boxes[row][col].draw(g); } } } public void gameLogic(MaskableBox box) { if ((chosenBoxes[0] != null)&&(chosenBoxes[1] != null)) { if(chosenBoxes[0].getBackColor() == chosenBoxes[1].getBackColor()) { for (int i=0; 0 <= chosenBoxes.length; ++i ) { for(int row = 0; row < boxes.length; row++) { for(int col = 0; col < boxes[row].length; col++) { if( boxes[row][col] == chosenBoxes[i] ) { System.out.println("boxes [row][col] == chosenBoxes[] at index: " + i ); matchedBoxes[row][col] = true; break; } } } } }else { chosenBoxes[0].setMask(true); chosenBoxes[1].setMask(true); } chosenBoxes = new MaskableBox[2]; }else { if (chosenBoxes[0] == null) { chosenBoxes[0] = box; chosenBoxes[0].setMask(false); return; }else{ if (chosenBoxes[1] == null) { chosenBoxes[1] = box; chosenBoxes[1].setMask(false); } } } } private void removeMouseListeners() { for(int row = 0; row < boxes.length; row ++) { for(int col = 0; col < boxes[row].length; col++) { removeMouseListener(boxes[row][col]); } } } private void buildBoxes() { // need to clear any chosen boxes when building new array. chosenBoxes = new MaskableBox[2]; // create a new matchedBoxes array matchedBoxes = new boolean [ROWS][COLS]; removeMouseListeners(); for(int row = 0; row < boxes.length; row++) { for(int col = 0; col < boxes[row].length; col++) { boxes[row][col] = new MaskableBox(START_X + col * BOX_WIDTH, START_Y + row * BOX_HEIGHT, BOX_WIDTH, BOX_HEIGHT, Color.gray, boxColors[row][col], true, true, this); addMouseListener(boxes[row][col]); } } } private void randomizeColors() { int[] chosenColors = {0,0,0,0,0,0,0,0}; Color[] availableColors = {Color.red, Color.blue, Color.green, Color.yellow, Color.cyan, Color.magenta, Color.pink, Color.orange }; for(int row = 0; row < boxes.length; row++) { for (int col = 0; col < boxes[row].length; col++) { for (;;) { int rnd = (int) (Math.random() * 8); if (chosenColors[rnd]< 2) { chosenColors[rnd]++; boxColors[row][col] = availableColors[rnd]; break; } } } } } } here is the second batch of code containing maskablebox import java.awt.Color; import java.awt.Container; import java.awt.Graphics; public class MaskableBox extends ClickableBox { private boolean mask; private Color maskColor; Container parent; public MaskableBox(int x, int y, int width, int height, Color borderColor, Color backColor, boolean drawBorder, boolean mask, Container parent ) { super(x, y, width, height, borderColor, backColor, drawBorder, parent); this.parent = parent; this.mask = mask; } public void draw(Graphics g) { if(mask=false) { super.draw(g); // setOldColor(g.getColor()); // g.setColor(maskColor); // g.fillRect(getX(),getY(),getWidth(), getHeight()); // if(isDrawBorder()) { // g.setColor(getBorderColor()); // g.drawRect(getX(),getY(),getWidth(),getHeight()); // } // g.setColor(getOldColor()); }else { if(mask=true) { //super.draw(g); setOldColor(g.getColor()); g.setColor(maskColor); g.fillRect(getX(),getY(),getWidth(), getHeight()); if(isDrawBorder()) { g.setColor(getBorderColor()); g.drawRect(getX(),getY(),getWidth(),getHeight()); } g.setColor(getOldColor()); } } } public boolean isMask() { return mask; } public void setMask(boolean mask) { this.mask = mask; } public Color getMaskColor() { return maskColor; } public void setMaskColor(Color maskColor) { this.maskColor = maskColor; } } I keep getting these error messages. I'm going nuts trying to figure this out. can anyone tell me what I'm doing wrong? boxes [row][col] == chosenBoxes[] at index: 0 boxes [row][col] == chosenBoxes[] at index: 1 Exception in thread "AWT-EventQueue-1" java.lang.ArrayIndexOutOfBoundsException: 2 at GuessingGame.gameLogic(GuessingGame.java:77) at GuessingGame.paint(GuessingGame.java:55) at java.awt.Container.update(Container.java:1801) at sun.awt.RepaintArea.updateComponent(RepaintArea.java:239) at sun.awt.RepaintArea.paint(RepaintArea.java:216) at sun.awt.windows.WComponentPeer.handleEvent(WComponentPeer.java:306) at java.awt.Component.dispatchEventImpl(Component.java:4706) at java.awt.Container.dispatchEventImpl(Container.java:2099) at java.awt.Component.dispatchEvent(Component.java:4460) at java.awt.EventQueue.dispatchEvent(EventQueue.java:599) at java.awt.EventDispatchThread.pumpOneEventForFilters(EventDispatchThread.java:269) at java.awt.EventDispatchThread.pumpEventsForFilter(EventDispatchThread.java:184) at java.awt.EventDispatchThread.pumpEventsForHierarchy(EventDispatchThread.java:174) at java.awt.EventDispatchThread.pumpEvents(EventDispatchThread.java:169) at java.awt.EventDispatchThread.pumpEvents(EventDispatchThread.java:161) at java.awt.EventDispatchThread.run(EventDispatchThread.java:122)

    Read the article

  • error echo id when i want to echo id for edit

    - by Prasanta Baidya
    I have a entry and edit page of a branch, I want echo id, when I mouse over into edit link in edit button, its show error,: branchedit.php?id=Note:Undefined index:id in line 101, but it work properly in localhost. error picture page link : https://www.dropbox.com/s/i1vu62lz3pezia0/id%20error.JPG My code: <?php include 'include/config.php'; include 'include/opendb.php'; include 'loginheader.php'; ?> <!DOCTYPE html PUBLIC "-//W3C//DTD XHTML 1.0 Transitional//EN" "http://www.w3.org/TR/xhtml1/DTD/xhtml1-transitional.dtd"> <html xmlns="http://www.w3.org/1999/xhtml"> <head> <meta http-equiv="Content-Type" content="text/html; charset=utf-8" /> <title>Branch</title> <!--Requered Validation --> <link rel="stylesheet" type="text/css" media="screen" href="jqueryvalidation/demo/css/screen.css" /> <script src="jqueryvalidation/jquery.js" type="text/javascript"></script> <script src="jqueryvalidation/jquery.validate.js" type="text/javascript"></script> <script type="text/javascript"> $(document).ready(function() { $("#commentForm").validate(); }); </script> <!--End Requered Validation --> <style type="text/css"> <!-- body { background-color: #cccccc; } --> </style> <style type="text/css"> <!-- --> </style> <link href="css/usercss.css" rel="stylesheet" type="text/css" /> <style type="text/css"> <!-- .style7 { color: #000000; font-weight: bold; } .style8 {color: #FFFFFF} --> </style> </head> <body> <div id="container"> <table width="453" border="0" align="left" cellpadding="0" cellspacing="1"> <tr> <td width="451"><form name="cmxform" id="commentForm" method="post" action="insert_ac.php"> <table width="100%" border="0" cellspacing="1" cellpadding="3"> <tr> <td colspan="3" class="style2">Insert Branch into Database </td> </tr> <tr> <td width="100" height="46">Branch Code</td> <td width="18">:</td> <td width="309"><input name="branch_code" type="text" id="branch_code" minlength="3" class="required"></td> </tr> <tr> <td height="51">Branch Name</td> <td>:</td> <td><input name="branch_name" type="text" id="branch_name" class="required" ></td> </tr> <tr> <td height="47" colspan="3" align="center"> <div align="right"> <input name="Submit" type="submit" class="submit_button" value="Submit" /> &nbsp;&nbsp;&nbsp;&nbsp;&nbsp;&nbsp;&nbsp;&nbsp;&nbsp; </div></td> </tr> </table> </form></td> </tr> </table> <!--Branch List --> <?php $sql="SELECT * FROM dc_master"; $result=mysql_query($sql); ?> <table width="436" border="1" cellpadding="2" cellspacing="0" class="table" id="list"> <tr> <td colspan="4"><div align="center" class="style7">List of Branches </div></td> </tr> <tr class="style4" > <td width="87" align="center"><span class="style8">Branch Code</span></td> <td width="176" align="center" ><span class="style8">Branch Name</span></td> <td width="70" align="center" ><span class="style8">Edit</span></td> <td width="77" align="center" ><span class="style8">Delete</span></td> </tr> <?php while($rows=mysql_fetch_array($result)){ ?> <tr> <td height="28"><div align="center" class="style3"><?php echo $rows['branch_code']; ?></div></td> <td class="style3">&nbsp;&nbsp;&nbsp;<?php echo $rows['dc_name']; ?></td> <!--link to update.php and send value of id --> <td align="center"><a href="branchedit.php?id=<?php echo $rows['id']; ?>" class="style3 style5 style5">Edit</a></td> <td align="center"><a href="delete.php?id=<?php echo $rows['id']; ?>" class="style3 style5 style5" onclick="return confirm('Are you sure, you want to delete? (After delete you can not undo or get it again) <?php ?>')">Delete</a></td> </tr> <?php } ?> </table> <span class="footer">Programmer : Prasanta Baidya / Mobile : 09830980840 / Email id : [email protected]</span></div> <?php mysql_close(); ?> </body> </html>

    Read the article

  • Help with java GUI- has error in main thread

    - by jan
    Hello guys, Basically im trying to do a Insurance Application form in java. And it uses multiple JPanels in a JFrame. -adding of JPanel into main program frame was done like this: //jpCenterArea to hold jp1-jp7 jpCenterArea.add(jp1); jpCenterArea.add(jp2); jpCenterArea.add(jp3); jpCenterArea.add(jp4); ...etc ********Add Jpanels to JFrame*****/ add(jpTitle, BorderLayout.NORTH); add(jpCenterArea, BorderLayout.CENTER); add(jpBottom, BorderLayout.SOUTH); However, even though program can compile, it cannot be run. error as mentioned below: Exception in thread "main" java.lang.NullPointerException at java.awt.Container.addImpl<Container.java:1045> at java.awt.Container.add<Container.java:365> at TravelInsuranceApplication.<init>TravelInsuranceApplication.java:120> at TravelInsuranceApplication.main<TravelInsuranceApplication.java:154> 1 import javax.swing.*; 2 import java.awt.*; 3 public class TravelInsuranceApplication extends JFrame 4 { 5 //declare private variables 6 private JLabel jlblTitle, jlblName, jlblNRIC, jlblAdd, jlblPostal, jlblContact, jlblDOB, 7 jlblEmail, jlblPeriod; 8 private JLabel jlblDeparture, jlblDays, jlblZone, jlblPlan; 9 private JTextField jtfName, jtfIC, jtfAdd, jtfPostal, jtfContact, jtfEmail, jtfZone; 10 private JRadioButton jrbResident, jrbOffice, jrbDeluxe, jrbClassic, jrbAsia, jrbWorldwide; 11 private ButtonGroup bgContact, bgZone, bgPlan; 12 private JComboBox jcDay, jcMonth, jcYear; 13 private JButton jbtnSubmit, jbtnCalculate, jbtnClear; 14 private JPanel jpTitle,jp1, jp2, jp3, jp4, jp5, jp6, jp7, jpBottom, jpCenterArea; 15 String[] day = {"1", "2", "3"}; 16 String[] month = {"january", "february"}; 17 String[] year = {"1981", "1985", "1990", "1995"}; 18 19 //constructor and GUI development 20 public TravelInsuranceApplication() 21 { 22 setSize(500,200); 23 setTitle("Travel Insurance Application"); 24 setDefaultCloseOperation(JFrame.EXIT_ON_CLOSE); 25 setLayout(new BorderLayout()); 26 27 //create ALL component objects/ 28 jlblTitle = new JLabel("Travel Insurance Application: "); 29 jlblName = new JLabel("Name of Insured: "); 30 jlblNRIC = new JLabel("NRIC: "); 31 jlblAdd = new JLabel("Address: "); 32 jlblPostal = new JLabel("Postal Code: "); 33 jlblContact = new JLabel("Telephone: "); 34 jlblDOB = new JLabel("Date Of Birth: "); 35 jlblEmail = new JLabel("Email Address: "); 36 jlblPeriod = new JLabel("Period Of Insurance "); 37 jlblDeparture = new JLabel("Departure Date "); 38 jlblDays = new JLabel("How Many Days To Insure "); 39 jlblZone = new JLabel("Zone: "); 40 jlblPlan = new JLabel("Plan: "); 41 42 jtfName = new JTextField(50); 43 jtfIC = new JTextField(15); 44 jtfAdd = new JTextField(50); 45 jtfPostal = new JTextField(15); 46 jtfContact = new JTextField(15); 47 jtfEmail = new JTextField(50); 48 jtfZone = new JTextField(100); 49 50 jrbResident = new JRadioButton("Rseident/Pgr"); 51 jrbOffice = new JRadioButton("Office/HP"); 52 jrbAsia = new JRadioButton("Asia"); 53 jrbAsia = new JRadioButton("Worldwide"); 54 jrbDeluxe = new JRadioButton("Deluxe"); 55 jrbClassic = new JRadioButton("Classic"); 56 57 jcDay = new JComboBox(day); 58 jcMonth = new JComboBox(month); 59 jcYear = new JComboBox(year); 60 61 jbtnSubmit = new JButton("Submit"); 62 jbtnCalculate = new JButton("Calculate"); 63 jbtnClear = new JButton("Clear"); 64 65 /****create JPanels - jpTitle, JpCenterArea & jp2-jp8 , jpBottom + setLayout 66 for ALL JPanels******/ 67 jpTitle = new JPanel(new FlowLayout(FlowLayout.CENTER)); 68 jpCenterArea = new JPanel(new FlowLayout()); 69 jp1 = new JPanel(new FlowLayout()); 70 jp2 = new JPanel(new FlowLayout(FlowLayout.CENTER)); 71 jp3 = new JPanel(new FlowLayout()); 72 jp4 = new JPanel(new FlowLayout()); 73 jp5 = new JPanel(new FlowLayout()); 74 jp6 = new JPanel(new FlowLayout(FlowLayout.CENTER)); 75 jp7 = new JPanel(new FlowLayout(FlowLayout.CENTER)); 76 jpBottom = new JPanel(new FlowLayout(FlowLayout.CENTER)); 77 78 79 80 81 //add components to JPanels 82 jpTitle.add(jlblTitle); 83 84 //jp1 85 jp1.add(jlblName); 86 jp1.add(jtfName); 87 jp1.add(jlblNRIC); 88 jp1.add(jtfIC); 89 90 //jp2 91 jp2.add(jlblAdd); 92 jp2.add(jtfAdd); 93 jp2.add(jlblPostal); 94 jp2.add(jtfPostal); 95 96 //jp3 97 jp3.add(jlblContact); 98 jp3.add(jtfContact); 99 jp3.add(jrbResident); 100 jp3.add(jrbOffice); 101 jp3.add(jlblDOB); 102 jp3.add(jcDay); 103 jp3.add(jcMonth); 104 jp3.add(jcYear); 105 106 //jp4 107 jp4.add(jlblEmail); 108 jp4.add(jtfEmail); 109 110 //jp5 111 jp5.add(jlblPeriod); 112 jp5.add(jlblDeparture); 113 jp5.add(jcDay); 114 jp5.add(jcMonth); 115 jp5.add(jcYear); 116 jp5.add(jlblDays); 117 jp5.add(jcDay); 118 119 //jp6 120 jp6.add(jlblZone); 121 jp6.add(jrbAsia); 122 jp6.add(jrbWorldwide); 123 jp6.add(jlblPlan); 124 jp6.add(jrbDeluxe); 125 jp6.add(jrbClassic); 126 127 //jp7 128 jp7.add(jtfZone); 129 130 //jpCenterArea to hold jp1-jp7 131 jpCenterArea.add(jp1); 132 jpCenterArea.add(jp2); 133 jpCenterArea.add(jp3); 134 jpCenterArea.add(jp4); 135 jpCenterArea.add(jp5); 136 jpCenterArea.add(jp6); 137 jpCenterArea.add(jp7); 138 139 //jpBottom 140 jpBottom.add(jbtnSubmit); 141 jpBottom.add(jbtnCalculate); 142 jpBottom.add(jbtnClear); 143 144 /********Add Jpanels to JFrame*****/ 145 add(jpTitle, BorderLayout.NORTH); 146 add(jpCenterArea, BorderLayout.CENTER); 147 add(jpBottom, BorderLayout.SOUTH); 148 149 setVisible(true); 150 151 152 153 }//end null constructor 154 public static void main(String[] args) 155 { 156 TravelInsuranceApplication travel = new TravelInsuranceApplication(); 157 158 }//end main 159 160 }//end class

    Read the article

  • Service Discovery in WCF 4.0 &ndash; Part 1

    - by Shaun
    When designing a service oriented architecture (SOA) system, there will be a lot of services with many service contracts, endpoints and behaviors. Besides the client calling the service, in a large distributed system a service may invoke other services. In this case, one service might need to know the endpoints it invokes. This might not be a problem in a small system. But when you have more than 10 services this might be a problem. For example in my current product, there are around 10 services, such as the user authentication service, UI integration service, location service, license service, device monitor service, event monitor service, schedule job service, accounting service, player management service, etc..   Benefit of Discovery Service Since almost all my services need to invoke at least one other service. This would be a difficult task to make sure all services endpoints are configured correctly in every service. And furthermore, it would be a nightmare when a service changed its endpoint at runtime. Hence, we need a discovery service to remove the dependency (configuration dependency). A discovery service plays as a service dictionary which stores the relationship between the contracts and the endpoints for every service. By using the discovery service, when service X wants to invoke service Y, it just need to ask the discovery service where is service Y, then the discovery service will return all proper endpoints of service Y, then service X can use the endpoint to send the request to service Y. And when some services changed their endpoint address, all need to do is to update its records in the discovery service then all others will know its new endpoint. In WCF 4.0 Discovery it supports both managed proxy discovery mode and ad-hoc discovery mode. In ad-hoc mode there is no standalone discovery service. When a client wanted to invoke a service, it will broadcast an message (normally in UDP protocol) to the entire network with the service match criteria. All services which enabled the discovery behavior will receive this message and only those matched services will send their endpoint back to the client. The managed proxy discovery service works as I described above. In this post I will only cover the managed proxy mode, where there’s a discovery service. For more information about the ad-hoc mode please refer to the MSDN.   Service Announcement and Probe The main functionality of discovery service should be return the proper endpoint addresses back to the service who is looking for. In most cases the consume service (as a client) will send the contract which it wanted to request to the discovery service. And then the discovery service will find the endpoint and respond. Sometimes the contract and endpoint are not enough. It also contains versioning, extensions attributes. This post I will only cover the case includes contract and endpoint. When a client (or sometimes a service who need to invoke another service) need to connect to a target service, it will firstly request the discovery service through the “Probe” method with the criteria. Basically the criteria contains the contract type name of the target service. Then the discovery service will search its endpoint repository by the criteria. The repository might be a database, a distributed cache or a flat XML file. If it matches, the discovery service will grab the endpoint information (it’s called discovery endpoint metadata in WCF) and send back. And this is called “Probe”. Finally the client received the discovery endpoint metadata and will use the endpoint to connect to the target service. Besides the probe, discovery service should take the responsible to know there is a new service available when it goes online, as well as stopped when it goes offline. This feature is named “Announcement”. When a service started and stopped, it will announce to the discovery service. So the basic functionality of a discovery service should includes: 1, An endpoint which receive the service online message, and add the service endpoint information in the discovery repository. 2, An endpoint which receive the service offline message, and remove the service endpoint information from the discovery repository. 3, An endpoint which receive the client probe message, and return the matches service endpoints, and return the discovery endpoint metadata. WCF 4.0 discovery service just covers all these features in it's infrastructure classes.   Discovery Service in WCF 4.0 WCF 4.0 introduced a new assembly named System.ServiceModel.Discovery which has all necessary classes and interfaces to build a WS-Discovery compliant discovery service. It supports ad-hoc and managed proxy modes. For the case mentioned in this post, what we need to build is a standalone discovery service, which is the managed proxy discovery service mode. To build a managed discovery service in WCF 4.0 just create a new class inherits from the abstract class System.ServiceModel.Discovery.DiscoveryProxy. This class implemented and abstracted the procedures of service announcement and probe. And it exposes 8 abstract methods where we can implement our own endpoint register, unregister and find logic. These 8 methods are asynchronized, which means all invokes to the discovery service are asynchronously, for better service capability and performance. 1, OnBeginOnlineAnnouncement, OnEndOnlineAnnouncement: Invoked when a service sent the online announcement message. We need to add the endpoint information to the repository in this method. 2, OnBeginOfflineAnnouncement, OnEndOfflineAnnouncement: Invoked when a service sent the offline announcement message. We need to remove the endpoint information from the repository in this method. 3, OnBeginFind, OnEndFind: Invoked when a client sent the probe message that want to find the service endpoint information. We need to look for the proper endpoints by matching the client’s criteria through the repository in this method. 4, OnBeginResolve, OnEndResolve: Invoked then a client sent the resolve message. Different from the find method, when using resolve method the discovery service will return the exactly one service endpoint metadata to the client. In our example we will NOT implement this method.   Let’s create our own discovery service, inherit the base System.ServiceModel.Discovery.DiscoveryProxy. We also need to specify the service behavior in this class. Since the build-in discovery service host class only support the singleton mode, we must set its instance context mode to single. 1: using System; 2: using System.Collections.Generic; 3: using System.Linq; 4: using System.Text; 5: using System.ServiceModel.Discovery; 6: using System.ServiceModel; 7:  8: namespace Phare.Service 9: { 10: [ServiceBehavior(InstanceContextMode = InstanceContextMode.Single, ConcurrencyMode = ConcurrencyMode.Multiple)] 11: public class ManagedProxyDiscoveryService : DiscoveryProxy 12: { 13: protected override IAsyncResult OnBeginFind(FindRequestContext findRequestContext, AsyncCallback callback, object state) 14: { 15: throw new NotImplementedException(); 16: } 17:  18: protected override IAsyncResult OnBeginOfflineAnnouncement(DiscoveryMessageSequence messageSequence, EndpointDiscoveryMetadata endpointDiscoveryMetadata, AsyncCallback callback, object state) 19: { 20: throw new NotImplementedException(); 21: } 22:  23: protected override IAsyncResult OnBeginOnlineAnnouncement(DiscoveryMessageSequence messageSequence, EndpointDiscoveryMetadata endpointDiscoveryMetadata, AsyncCallback callback, object state) 24: { 25: throw new NotImplementedException(); 26: } 27:  28: protected override IAsyncResult OnBeginResolve(ResolveCriteria resolveCriteria, AsyncCallback callback, object state) 29: { 30: throw new NotImplementedException(); 31: } 32:  33: protected override void OnEndFind(IAsyncResult result) 34: { 35: throw new NotImplementedException(); 36: } 37:  38: protected override void OnEndOfflineAnnouncement(IAsyncResult result) 39: { 40: throw new NotImplementedException(); 41: } 42:  43: protected override void OnEndOnlineAnnouncement(IAsyncResult result) 44: { 45: throw new NotImplementedException(); 46: } 47:  48: protected override EndpointDiscoveryMetadata OnEndResolve(IAsyncResult result) 49: { 50: throw new NotImplementedException(); 51: } 52: } 53: } Then let’s implement the online, offline and find methods one by one. WCF discovery service gives us full flexibility to implement the endpoint add, remove and find logic. For the demo purpose we will use an internal dictionary to store the services’ endpoint metadata. In the next post we will see how to serialize and store these information in database. Define a concurrent dictionary inside the service class since our it will be used in the multiple threads scenario. 1: [ServiceBehavior(InstanceContextMode = InstanceContextMode.Single, ConcurrencyMode = ConcurrencyMode.Multiple)] 2: public class ManagedProxyDiscoveryService : DiscoveryProxy 3: { 4: private ConcurrentDictionary<EndpointAddress, EndpointDiscoveryMetadata> _services; 5:  6: public ManagedProxyDiscoveryService() 7: { 8: _services = new ConcurrentDictionary<EndpointAddress, EndpointDiscoveryMetadata>(); 9: } 10: } Then we can simply implement the logic of service online and offline. 1: protected override IAsyncResult OnBeginOnlineAnnouncement(DiscoveryMessageSequence messageSequence, EndpointDiscoveryMetadata endpointDiscoveryMetadata, AsyncCallback callback, object state) 2: { 3: _services.AddOrUpdate(endpointDiscoveryMetadata.Address, endpointDiscoveryMetadata, (key, value) => endpointDiscoveryMetadata); 4: return new OnOnlineAnnouncementAsyncResult(callback, state); 5: } 6:  7: protected override void OnEndOnlineAnnouncement(IAsyncResult result) 8: { 9: OnOnlineAnnouncementAsyncResult.End(result); 10: } 11:  12: protected override IAsyncResult OnBeginOfflineAnnouncement(DiscoveryMessageSequence messageSequence, EndpointDiscoveryMetadata endpointDiscoveryMetadata, AsyncCallback callback, object state) 13: { 14: EndpointDiscoveryMetadata endpoint = null; 15: _services.TryRemove(endpointDiscoveryMetadata.Address, out endpoint); 16: return new OnOfflineAnnouncementAsyncResult(callback, state); 17: } 18:  19: protected override void OnEndOfflineAnnouncement(IAsyncResult result) 20: { 21: OnOfflineAnnouncementAsyncResult.End(result); 22: } Regards the find method, the parameter FindRequestContext.Criteria has a method named IsMatch, which can be use for us to evaluate which service metadata is satisfied with the criteria. So the implementation of find method would be like this. 1: protected override IAsyncResult OnBeginFind(FindRequestContext findRequestContext, AsyncCallback callback, object state) 2: { 3: _services.Where(s => findRequestContext.Criteria.IsMatch(s.Value)) 4: .Select(s => s.Value) 5: .All(meta => 6: { 7: findRequestContext.AddMatchingEndpoint(meta); 8: return true; 9: }); 10: return new OnFindAsyncResult(callback, state); 11: } 12:  13: protected override void OnEndFind(IAsyncResult result) 14: { 15: OnFindAsyncResult.End(result); 16: } As you can see, we checked all endpoints metadata in repository by invoking the IsMatch method. Then add all proper endpoints metadata into the parameter. Finally since all these methods are asynchronized we need some AsyncResult classes as well. Below are the base class and the inherited classes used in previous methods. 1: using System; 2: using System.Collections.Generic; 3: using System.Linq; 4: using System.Text; 5: using System.Threading; 6:  7: namespace Phare.Service 8: { 9: abstract internal class AsyncResult : IAsyncResult 10: { 11: AsyncCallback callback; 12: bool completedSynchronously; 13: bool endCalled; 14: Exception exception; 15: bool isCompleted; 16: ManualResetEvent manualResetEvent; 17: object state; 18: object thisLock; 19:  20: protected AsyncResult(AsyncCallback callback, object state) 21: { 22: this.callback = callback; 23: this.state = state; 24: this.thisLock = new object(); 25: } 26:  27: public object AsyncState 28: { 29: get 30: { 31: return state; 32: } 33: } 34:  35: public WaitHandle AsyncWaitHandle 36: { 37: get 38: { 39: if (manualResetEvent != null) 40: { 41: return manualResetEvent; 42: } 43: lock (ThisLock) 44: { 45: if (manualResetEvent == null) 46: { 47: manualResetEvent = new ManualResetEvent(isCompleted); 48: } 49: } 50: return manualResetEvent; 51: } 52: } 53:  54: public bool CompletedSynchronously 55: { 56: get 57: { 58: return completedSynchronously; 59: } 60: } 61:  62: public bool IsCompleted 63: { 64: get 65: { 66: return isCompleted; 67: } 68: } 69:  70: object ThisLock 71: { 72: get 73: { 74: return this.thisLock; 75: } 76: } 77:  78: protected static TAsyncResult End<TAsyncResult>(IAsyncResult result) 79: where TAsyncResult : AsyncResult 80: { 81: if (result == null) 82: { 83: throw new ArgumentNullException("result"); 84: } 85:  86: TAsyncResult asyncResult = result as TAsyncResult; 87:  88: if (asyncResult == null) 89: { 90: throw new ArgumentException("Invalid async result.", "result"); 91: } 92:  93: if (asyncResult.endCalled) 94: { 95: throw new InvalidOperationException("Async object already ended."); 96: } 97:  98: asyncResult.endCalled = true; 99:  100: if (!asyncResult.isCompleted) 101: { 102: asyncResult.AsyncWaitHandle.WaitOne(); 103: } 104:  105: if (asyncResult.manualResetEvent != null) 106: { 107: asyncResult.manualResetEvent.Close(); 108: } 109:  110: if (asyncResult.exception != null) 111: { 112: throw asyncResult.exception; 113: } 114:  115: return asyncResult; 116: } 117:  118: protected void Complete(bool completedSynchronously) 119: { 120: if (isCompleted) 121: { 122: throw new InvalidOperationException("This async result is already completed."); 123: } 124:  125: this.completedSynchronously = completedSynchronously; 126:  127: if (completedSynchronously) 128: { 129: this.isCompleted = true; 130: } 131: else 132: { 133: lock (ThisLock) 134: { 135: this.isCompleted = true; 136: if (this.manualResetEvent != null) 137: { 138: this.manualResetEvent.Set(); 139: } 140: } 141: } 142:  143: if (callback != null) 144: { 145: callback(this); 146: } 147: } 148:  149: protected void Complete(bool completedSynchronously, Exception exception) 150: { 151: this.exception = exception; 152: Complete(completedSynchronously); 153: } 154: } 155: } 1: using System; 2: using System.Collections.Generic; 3: using System.Linq; 4: using System.Text; 5: using System.ServiceModel.Discovery; 6: using Phare.Service; 7:  8: namespace Phare.Service 9: { 10: internal sealed class OnOnlineAnnouncementAsyncResult : AsyncResult 11: { 12: public OnOnlineAnnouncementAsyncResult(AsyncCallback callback, object state) 13: : base(callback, state) 14: { 15: this.Complete(true); 16: } 17:  18: public static void End(IAsyncResult result) 19: { 20: AsyncResult.End<OnOnlineAnnouncementAsyncResult>(result); 21: } 22:  23: } 24:  25: sealed class OnOfflineAnnouncementAsyncResult : AsyncResult 26: { 27: public OnOfflineAnnouncementAsyncResult(AsyncCallback callback, object state) 28: : base(callback, state) 29: { 30: this.Complete(true); 31: } 32:  33: public static void End(IAsyncResult result) 34: { 35: AsyncResult.End<OnOfflineAnnouncementAsyncResult>(result); 36: } 37: } 38:  39: sealed class OnFindAsyncResult : AsyncResult 40: { 41: public OnFindAsyncResult(AsyncCallback callback, object state) 42: : base(callback, state) 43: { 44: this.Complete(true); 45: } 46:  47: public static void End(IAsyncResult result) 48: { 49: AsyncResult.End<OnFindAsyncResult>(result); 50: } 51: } 52:  53: sealed class OnResolveAsyncResult : AsyncResult 54: { 55: EndpointDiscoveryMetadata matchingEndpoint; 56:  57: public OnResolveAsyncResult(EndpointDiscoveryMetadata matchingEndpoint, AsyncCallback callback, object state) 58: : base(callback, state) 59: { 60: this.matchingEndpoint = matchingEndpoint; 61: this.Complete(true); 62: } 63:  64: public static EndpointDiscoveryMetadata End(IAsyncResult result) 65: { 66: OnResolveAsyncResult thisPtr = AsyncResult.End<OnResolveAsyncResult>(result); 67: return thisPtr.matchingEndpoint; 68: } 69: } 70: } Now we have finished the discovery service. The next step is to host it. The discovery service is a standard WCF service. So we can use ServiceHost on a console application, windows service, or in IIS as usual. The following code is how to host the discovery service we had just created in a console application. 1: static void Main(string[] args) 2: { 3: using (var host = new ServiceHost(new ManagedProxyDiscoveryService())) 4: { 5: host.Opened += (sender, e) => 6: { 7: host.Description.Endpoints.All((ep) => 8: { 9: Console.WriteLine(ep.ListenUri); 10: return true; 11: }); 12: }; 13:  14: try 15: { 16: // retrieve the announcement, probe endpoint and binding from configuration 17: var announcementEndpointAddress = new EndpointAddress(ConfigurationManager.AppSettings["announcementEndpointAddress"]); 18: var probeEndpointAddress = new EndpointAddress(ConfigurationManager.AppSettings["probeEndpointAddress"]); 19: var binding = Activator.CreateInstance(Type.GetType(ConfigurationManager.AppSettings["bindingType"], true, true)) as Binding; 20: var announcementEndpoint = new AnnouncementEndpoint(binding, announcementEndpointAddress); 21: var probeEndpoint = new DiscoveryEndpoint(binding, probeEndpointAddress); 22: probeEndpoint.IsSystemEndpoint = false; 23: // append the service endpoint for announcement and probe 24: host.AddServiceEndpoint(announcementEndpoint); 25: host.AddServiceEndpoint(probeEndpoint); 26:  27: host.Open(); 28:  29: Console.WriteLine("Press any key to exit."); 30: Console.ReadKey(); 31: } 32: catch (Exception ex) 33: { 34: Console.WriteLine(ex.ToString()); 35: } 36: } 37:  38: Console.WriteLine("Done."); 39: Console.ReadKey(); 40: } What we need to notice is that, the discovery service needs two endpoints for announcement and probe. In this example I just retrieve them from the configuration file. I also specified the binding of these two endpoints in configuration file as well. 1: <?xml version="1.0"?> 2: <configuration> 3: <startup> 4: <supportedRuntime version="v4.0" sku=".NETFramework,Version=v4.0"/> 5: </startup> 6: <appSettings> 7: <add key="announcementEndpointAddress" value="net.tcp://localhost:10010/announcement"/> 8: <add key="probeEndpointAddress" value="net.tcp://localhost:10011/probe"/> 9: <add key="bindingType" value="System.ServiceModel.NetTcpBinding, System.ServiceModel, Version=4.0.0.0, Culture=neutral, PublicKeyToken=b77a5c561934e089"/> 10: </appSettings> 11: </configuration> And this is the console screen when I ran my discovery service. As you can see there are two endpoints listening for announcement message and probe message.   Discoverable Service and Client Next, let’s create a WCF service that is discoverable, which means it can be found by the discovery service. To do so, we need to let the service send the online announcement message to the discovery service, as well as offline message before it shutdown. Just create a simple service which can make the incoming string to upper. The service contract and implementation would be like this. 1: [ServiceContract] 2: public interface IStringService 3: { 4: [OperationContract] 5: string ToUpper(string content); 6: } 1: public class StringService : IStringService 2: { 3: public string ToUpper(string content) 4: { 5: return content.ToUpper(); 6: } 7: } Then host this service in the console application. In order to make the discovery service easy to be tested the service address will be changed each time it’s started. 1: static void Main(string[] args) 2: { 3: var baseAddress = new Uri(string.Format("net.tcp://localhost:11001/stringservice/{0}/", Guid.NewGuid().ToString())); 4:  5: using (var host = new ServiceHost(typeof(StringService), baseAddress)) 6: { 7: host.Opened += (sender, e) => 8: { 9: Console.WriteLine("Service opened at {0}", host.Description.Endpoints.First().ListenUri); 10: }; 11:  12: host.AddServiceEndpoint(typeof(IStringService), new NetTcpBinding(), string.Empty); 13:  14: host.Open(); 15:  16: Console.WriteLine("Press any key to exit."); 17: Console.ReadKey(); 18: } 19: } Currently this service is NOT discoverable. We need to add a special service behavior so that it could send the online and offline message to the discovery service announcement endpoint when the host is opened and closed. WCF 4.0 introduced a service behavior named ServiceDiscoveryBehavior. When we specified the announcement endpoint address and appended it to the service behaviors this service will be discoverable. 1: var announcementAddress = new EndpointAddress(ConfigurationManager.AppSettings["announcementEndpointAddress"]); 2: var announcementBinding = Activator.CreateInstance(Type.GetType(ConfigurationManager.AppSettings["bindingType"], true, true)) as Binding; 3: var announcementEndpoint = new AnnouncementEndpoint(announcementBinding, announcementAddress); 4: var discoveryBehavior = new ServiceDiscoveryBehavior(); 5: discoveryBehavior.AnnouncementEndpoints.Add(announcementEndpoint); 6: host.Description.Behaviors.Add(discoveryBehavior); The ServiceDiscoveryBehavior utilizes the service extension and channel dispatcher to implement the online and offline announcement logic. In short, it injected the channel open and close procedure and send the online and offline message to the announcement endpoint.   On client side, when we have the discovery service, a client can invoke a service without knowing its endpoint. WCF discovery assembly provides a class named DiscoveryClient, which can be used to find the proper service endpoint by passing the criteria. In the code below I initialized the DiscoveryClient, specified the discovery service probe endpoint address. Then I created the find criteria by specifying the service contract I wanted to use and invoke the Find method. This will send the probe message to the discovery service and it will find the endpoints back to me. The discovery service will return all endpoints that matches the find criteria, which means in the result of the find method there might be more than one endpoints. In this example I just returned the first matched one back. In the next post I will show how to extend our discovery service to make it work like a service load balancer. 1: static EndpointAddress FindServiceEndpoint() 2: { 3: var probeEndpointAddress = new EndpointAddress(ConfigurationManager.AppSettings["probeEndpointAddress"]); 4: var probeBinding = Activator.CreateInstance(Type.GetType(ConfigurationManager.AppSettings["bindingType"], true, true)) as Binding; 5: var discoveryEndpoint = new DiscoveryEndpoint(probeBinding, probeEndpointAddress); 6:  7: EndpointAddress address = null; 8: FindResponse result = null; 9: using (var discoveryClient = new DiscoveryClient(discoveryEndpoint)) 10: { 11: result = discoveryClient.Find(new FindCriteria(typeof(IStringService))); 12: } 13:  14: if (result != null && result.Endpoints.Any()) 15: { 16: var endpointMetadata = result.Endpoints.First(); 17: address = endpointMetadata.Address; 18: } 19: return address; 20: } Once we probed the discovery service we will receive the endpoint. So in the client code we can created the channel factory from the endpoint and binding, and invoke to the service. When creating the client side channel factory we need to make sure that the client side binding should be the same as the service side. WCF discovery service can be used to find the endpoint for a service contract, but the binding is NOT included. This is because the binding was not in the WS-Discovery specification. In the next post I will demonstrate how to add the binding information into the discovery service. At that moment the client don’t need to create the binding by itself. Instead it will use the binding received from the discovery service. 1: static void Main(string[] args) 2: { 3: Console.WriteLine("Say something..."); 4: var content = Console.ReadLine(); 5: while (!string.IsNullOrWhiteSpace(content)) 6: { 7: Console.WriteLine("Finding the service endpoint..."); 8: var address = FindServiceEndpoint(); 9: if (address == null) 10: { 11: Console.WriteLine("There is no endpoint matches the criteria."); 12: } 13: else 14: { 15: Console.WriteLine("Found the endpoint {0}", address.Uri); 16:  17: var factory = new ChannelFactory<IStringService>(new NetTcpBinding(), address); 18: factory.Opened += (sender, e) => 19: { 20: Console.WriteLine("Connecting to {0}.", factory.Endpoint.ListenUri); 21: }; 22: var proxy = factory.CreateChannel(); 23: using (proxy as IDisposable) 24: { 25: Console.WriteLine("ToUpper: {0} => {1}", content, proxy.ToUpper(content)); 26: } 27: } 28:  29: Console.WriteLine("Say something..."); 30: content = Console.ReadLine(); 31: } 32: } Similarly, the discovery service probe endpoint and binding were defined in the configuration file. 1: <?xml version="1.0"?> 2: <configuration> 3: <startup> 4: <supportedRuntime version="v4.0" sku=".NETFramework,Version=v4.0"/> 5: </startup> 6: <appSettings> 7: <add key="announcementEndpointAddress" value="net.tcp://localhost:10010/announcement"/> 8: <add key="probeEndpointAddress" value="net.tcp://localhost:10011/probe"/> 9: <add key="bindingType" value="System.ServiceModel.NetTcpBinding, System.ServiceModel, Version=4.0.0.0, Culture=neutral, PublicKeyToken=b77a5c561934e089"/> 10: </appSettings> 11: </configuration> OK, now let’s have a test. Firstly start the discovery service, and then start our discoverable service. When it started it will announced to the discovery service and registered its endpoint into the repository, which is the local dictionary. And then start the client and type something. As you can see the client asked the discovery service for the endpoint and then establish the connection to the discoverable service. And more interesting, do NOT close the client console but terminate the discoverable service but press the enter key. This will make the service send the offline message to the discovery service. Then start the discoverable service again. Since we made it use a different address each time it started, currently it should be hosted on another address. If we enter something in the client we could see that it asked the discovery service and retrieve the new endpoint, and connect the the service.   Summary In this post I discussed the benefit of using the discovery service and the procedures of service announcement and probe. I also demonstrated how to leverage the WCF Discovery feature in WCF 4.0 to build a simple managed discovery service. For test purpose, in this example I used the in memory dictionary as the discovery endpoint metadata repository. And when finding I also just return the first matched endpoint back. I also hard coded the bindings between the discoverable service and the client. In next post I will show you how to solve the problem mentioned above, as well as some additional feature for production usage. You can download the code here.   Hope this helps, Shaun All documents and related graphics, codes are provided "AS IS" without warranty of any kind. Copyright © Shaun Ziyan Xu. This work is licensed under the Creative Commons License.

    Read the article

  • Node.js Adventure - Storage Services and Service Runtime

    - by Shaun
    When I described on how to host a Node.js application on Windows Azure, one of questions might be raised about how to consume the vary Windows Azure services, such as the storage, service bus, access control, etc.. Interact with windows azure services is available in Node.js through the Windows Azure Node.js SDK, which is a module available in NPM. In this post I would like to describe on how to use Windows Azure Storage (a.k.a. WAS) as well as the service runtime.   Consume Windows Azure Storage Let’s firstly have a look on how to consume WAS through Node.js. As we know in the previous post we can host Node.js application on Windows Azure Web Site (a.k.a. WAWS) as well as Windows Azure Cloud Service (a.k.a. WACS). In theory, WAWS is also built on top of WACS worker roles with some more features. Hence in this post I will only demonstrate for hosting in WACS worker role. The Node.js code can be used when consuming WAS when hosted on WAWS. But since there’s no roles in WAWS, the code for consuming service runtime mentioned in the next section cannot be used for WAWS node application. We can use the solution that I created in my last post. Alternatively we can create a new windows azure project in Visual Studio with a worker role, add the “node.exe” and “index.js” and install “express” and “node-sqlserver” modules, make all files as “Copy always”. In order to use windows azure services we need to have Windows Azure Node.js SDK, as knows as a module named “azure” which can be installed through NPM. Once we downloaded and installed, we need to include them in our worker role project and make them as “Copy always”. You can use my “Copy all always” tool mentioned in my last post to update the currently worker role project file. You can also find the source code of this tool here. The source code of Windows Azure SDK for Node.js can be found in its GitHub page. It contains two parts. One is a CLI tool which provides a cross platform command line package for Mac and Linux to manage WAWS and Windows Azure Virtual Machines (a.k.a. WAVM). The other is a library for managing and consuming vary windows azure services includes tables, blobs, queues, service bus and the service runtime. I will not cover all of them but will only demonstrate on how to use tables and service runtime information in this post. You can find the full document of this SDK here. Back to Visual Studio and open the “index.js”, let’s continue our application from the last post, which was working against Windows Azure SQL Database (a.k.a. WASD). The code should looks like this. 1: var express = require("express"); 2: var sql = require("node-sqlserver"); 3:  4: var connectionString = "Driver={SQL Server Native Client 10.0};Server=tcp:ac6271ya9e.database.windows.net,1433;Database=synctile;Uid=shaunxu@ac6271ya9e;Pwd={PASSWORD};Encrypt=yes;Connection Timeout=30;"; 5: var port = 80; 6:  7: var app = express(); 8:  9: app.configure(function () { 10: app.use(express.bodyParser()); 11: }); 12:  13: app.get("/", function (req, res) { 14: sql.open(connectionString, function (err, conn) { 15: if (err) { 16: console.log(err); 17: res.send(500, "Cannot open connection."); 18: } 19: else { 20: conn.queryRaw("SELECT * FROM [Resource]", function (err, results) { 21: if (err) { 22: console.log(err); 23: res.send(500, "Cannot retrieve records."); 24: } 25: else { 26: res.json(results); 27: } 28: }); 29: } 30: }); 31: }); 32:  33: app.get("/text/:key/:culture", function (req, res) { 34: sql.open(connectionString, function (err, conn) { 35: if (err) { 36: console.log(err); 37: res.send(500, "Cannot open connection."); 38: } 39: else { 40: var key = req.params.key; 41: var culture = req.params.culture; 42: var command = "SELECT * FROM [Resource] WHERE [Key] = '" + key + "' AND [Culture] = '" + culture + "'"; 43: conn.queryRaw(command, function (err, results) { 44: if (err) { 45: console.log(err); 46: res.send(500, "Cannot retrieve records."); 47: } 48: else { 49: res.json(results); 50: } 51: }); 52: } 53: }); 54: }); 55:  56: app.get("/sproc/:key/:culture", function (req, res) { 57: sql.open(connectionString, function (err, conn) { 58: if (err) { 59: console.log(err); 60: res.send(500, "Cannot open connection."); 61: } 62: else { 63: var key = req.params.key; 64: var culture = req.params.culture; 65: var command = "EXEC GetItem '" + key + "', '" + culture + "'"; 66: conn.queryRaw(command, function (err, results) { 67: if (err) { 68: console.log(err); 69: res.send(500, "Cannot retrieve records."); 70: } 71: else { 72: res.json(results); 73: } 74: }); 75: } 76: }); 77: }); 78:  79: app.post("/new", function (req, res) { 80: var key = req.body.key; 81: var culture = req.body.culture; 82: var val = req.body.val; 83:  84: sql.open(connectionString, function (err, conn) { 85: if (err) { 86: console.log(err); 87: res.send(500, "Cannot open connection."); 88: } 89: else { 90: var command = "INSERT INTO [Resource] VALUES ('" + key + "', '" + culture + "', N'" + val + "')"; 91: conn.queryRaw(command, function (err, results) { 92: if (err) { 93: console.log(err); 94: res.send(500, "Cannot retrieve records."); 95: } 96: else { 97: res.send(200, "Inserted Successful"); 98: } 99: }); 100: } 101: }); 102: }); 103:  104: app.listen(port); Now let’s create a new function, copy the records from WASD to table service. 1. Delete the table named “resource”. 2. Create a new table named “resource”. These 2 steps ensures that we have an empty table. 3. Load all records from the “resource” table in WASD. 4. For each records loaded from WASD, insert them into the table one by one. 5. Prompt to user when finished. In order to use table service we need the storage account and key, which can be found from the developer portal. Just select the storage account and click the Manage Keys button. Then create two local variants in our Node.js application for the storage account name and key. Since we need to use WAS we need to import the azure module. Also I created another variant stored the table name. In order to work with table service I need to create the storage client for table service. This is very similar as the Windows Azure SDK for .NET. As the code below I created a new variant named “client” and use “createTableService”, specified my storage account name and key. 1: var azure = require("azure"); 2: var storageAccountName = "synctile"; 3: var storageAccountKey = "/cOy9L7xysXOgPYU9FjDvjrRAhaMX/5tnOpcjqloPNDJYucbgTy7MOrAW7CbUg6PjaDdmyl+6pkwUnKETsPVNw=="; 4: var tableName = "resource"; 5: var client = azure.createTableService(storageAccountName, storageAccountKey); Now create a new function for URL “/was/init” so that we can trigger it through browser. Then in this function we will firstly load all records from WASD. 1: app.get("/was/init", function (req, res) { 2: // load all records from windows azure sql database 3: sql.open(connectionString, function (err, conn) { 4: if (err) { 5: console.log(err); 6: res.send(500, "Cannot open connection."); 7: } 8: else { 9: conn.queryRaw("SELECT * FROM [Resource]", function (err, results) { 10: if (err) { 11: console.log(err); 12: res.send(500, "Cannot retrieve records."); 13: } 14: else { 15: if (results.rows.length > 0) { 16: // begin to transform the records into table service 17: } 18: } 19: }); 20: } 21: }); 22: }); When we succeed loaded all records we can start to transform them into table service. First I need to recreate the table in table service. This can be done by deleting and creating the table through table client I had just created previously. 1: app.get("/was/init", function (req, res) { 2: // load all records from windows azure sql database 3: sql.open(connectionString, function (err, conn) { 4: if (err) { 5: console.log(err); 6: res.send(500, "Cannot open connection."); 7: } 8: else { 9: conn.queryRaw("SELECT * FROM [Resource]", function (err, results) { 10: if (err) { 11: console.log(err); 12: res.send(500, "Cannot retrieve records."); 13: } 14: else { 15: if (results.rows.length > 0) { 16: // begin to transform the records into table service 17: // recreate the table named 'resource' 18: client.deleteTable(tableName, function (error) { 19: client.createTableIfNotExists(tableName, function (error) { 20: if (error) { 21: error["target"] = "createTableIfNotExists"; 22: res.send(500, error); 23: } 24: else { 25: // transform the records 26: } 27: }); 28: }); 29: } 30: } 31: }); 32: } 33: }); 34: }); As you can see, the azure SDK provide its methods in callback pattern. In fact, almost all modules in Node.js use the callback pattern. For example, when I deleted a table I invoked “deleteTable” method, provided the name of the table and a callback function which will be performed when the table had been deleted or failed. Underlying, the azure module will perform the table deletion operation in POSIX async threads pool asynchronously. And once it’s done the callback function will be performed. This is the reason we need to nest the table creation code inside the deletion function. If we perform the table creation code after the deletion code then they will be invoked in parallel. Next, for each records in WASD I created an entity and then insert into the table service. Finally I send the response to the browser. Can you find a bug in the code below? I will describe it later in this post. 1: app.get("/was/init", function (req, res) { 2: // load all records from windows azure sql database 3: sql.open(connectionString, function (err, conn) { 4: if (err) { 5: console.log(err); 6: res.send(500, "Cannot open connection."); 7: } 8: else { 9: conn.queryRaw("SELECT * FROM [Resource]", function (err, results) { 10: if (err) { 11: console.log(err); 12: res.send(500, "Cannot retrieve records."); 13: } 14: else { 15: if (results.rows.length > 0) { 16: // begin to transform the records into table service 17: // recreate the table named 'resource' 18: client.deleteTable(tableName, function (error) { 19: client.createTableIfNotExists(tableName, function (error) { 20: if (error) { 21: error["target"] = "createTableIfNotExists"; 22: res.send(500, error); 23: } 24: else { 25: // transform the records 26: for (var i = 0; i < results.rows.length; i++) { 27: var entity = { 28: "PartitionKey": results.rows[i][1], 29: "RowKey": results.rows[i][0], 30: "Value": results.rows[i][2] 31: }; 32: client.insertEntity(tableName, entity, function (error) { 33: if (error) { 34: error["target"] = "insertEntity"; 35: res.send(500, error); 36: } 37: else { 38: console.log("entity inserted"); 39: } 40: }); 41: } 42: // send the 43: console.log("all done"); 44: res.send(200, "All done!"); 45: } 46: }); 47: }); 48: } 49: } 50: }); 51: } 52: }); 53: }); Now we can publish it to the cloud and have a try. But normally we’d better test it at the local emulator first. In Node.js SDK there are three build-in properties which provides the account name, key and host address for local storage emulator. We can use them to initialize our table service client. We also need to change the SQL connection string to let it use my local database. The code will be changed as below. 1: // windows azure sql database 2: //var connectionString = "Driver={SQL Server Native Client 10.0};Server=tcp:ac6271ya9e.database.windows.net,1433;Database=synctile;Uid=shaunxu@ac6271ya9e;Pwd=eszqu94XZY;Encrypt=yes;Connection Timeout=30;"; 3: // sql server 4: var connectionString = "Driver={SQL Server Native Client 11.0};Server={.};Database={Caspar};Trusted_Connection={Yes};"; 5:  6: var azure = require("azure"); 7: var storageAccountName = "synctile"; 8: var storageAccountKey = "/cOy9L7xysXOgPYU9FjDvjrRAhaMX/5tnOpcjqloPNDJYucbgTy7MOrAW7CbUg6PjaDdmyl+6pkwUnKETsPVNw=="; 9: var tableName = "resource"; 10: // windows azure storage 11: //var client = azure.createTableService(storageAccountName, storageAccountKey); 12: // local storage emulator 13: var client = azure.createTableService(azure.ServiceClient.DEVSTORE_STORAGE_ACCOUNT, azure.ServiceClient.DEVSTORE_STORAGE_ACCESS_KEY, azure.ServiceClient.DEVSTORE_TABLE_HOST); Now let’s run the application and navigate to “localhost:12345/was/init” as I hosted it on port 12345. We can find it transformed the data from my local database to local table service. Everything looks fine. But there is a bug in my code. If we have a look on the Node.js command window we will find that it sent response before all records had been inserted, which is not what I expected. The reason is that, as I mentioned before, Node.js perform all IO operations in non-blocking model. When we inserted the records we executed the table service insert method in parallel, and the operation of sending response was also executed in parallel, even though I wrote it at the end of my logic. The correct logic should be, when all entities had been copied to table service with no error, then I will send response to the browser, otherwise I should send error message to the browser. To do so I need to import another module named “async”, which helps us to coordinate our asynchronous code. Install the module and import it at the beginning of the code. Then we can use its “forEach” method for the asynchronous code of inserting table entities. The first argument of “forEach” is the array that will be performed. The second argument is the operation for each items in the array. And the third argument will be invoked then all items had been performed or any errors occurred. Here we can send our response to browser. 1: app.get("/was/init", function (req, res) { 2: // load all records from windows azure sql database 3: sql.open(connectionString, function (err, conn) { 4: if (err) { 5: console.log(err); 6: res.send(500, "Cannot open connection."); 7: } 8: else { 9: conn.queryRaw("SELECT * FROM [Resource]", function (err, results) { 10: if (err) { 11: console.log(err); 12: res.send(500, "Cannot retrieve records."); 13: } 14: else { 15: if (results.rows.length > 0) { 16: // begin to transform the records into table service 17: // recreate the table named 'resource' 18: client.deleteTable(tableName, function (error) { 19: client.createTableIfNotExists(tableName, function (error) { 20: if (error) { 21: error["target"] = "createTableIfNotExists"; 22: res.send(500, error); 23: } 24: else { 25: async.forEach(results.rows, 26: // transform the records 27: function (row, callback) { 28: var entity = { 29: "PartitionKey": row[1], 30: "RowKey": row[0], 31: "Value": row[2] 32: }; 33: client.insertEntity(tableName, entity, function (error) { 34: if (error) { 35: callback(error); 36: } 37: else { 38: console.log("entity inserted."); 39: callback(null); 40: } 41: }); 42: }, 43: // send reponse 44: function (error) { 45: if (error) { 46: error["target"] = "insertEntity"; 47: res.send(500, error); 48: } 49: else { 50: console.log("all done"); 51: res.send(200, "All done!"); 52: } 53: } 54: ); 55: } 56: }); 57: }); 58: } 59: } 60: }); 61: } 62: }); 63: }); Run it locally and now we can find the response was sent after all entities had been inserted. Query entities against table service is simple as well. Just use the “queryEntity” method from the table service client and providing the partition key and row key. We can also provide a complex query criteria as well, for example the code here. In the code below I queried an entity by the partition key and row key, and return the proper localization value in response. 1: app.get("/was/:key/:culture", function (req, res) { 2: var key = req.params.key; 3: var culture = req.params.culture; 4: client.queryEntity(tableName, culture, key, function (error, entity) { 5: if (error) { 6: res.send(500, error); 7: } 8: else { 9: res.json(entity); 10: } 11: }); 12: }); And then tested it on local emulator. Finally if we want to publish this application to the cloud we should change the database connection string and storage account. For more information about how to consume blob and queue service, as well as the service bus please refer to the MSDN page.   Consume Service Runtime As I mentioned above, before we published our application to the cloud we need to change the connection string and account information in our code. But if you had played with WACS you should have known that the service runtime provides the ability to retrieve configuration settings, endpoints and local resource information at runtime. Which means we can have these values defined in CSCFG and CSDEF files and then the runtime should be able to retrieve the proper values. For example we can add some role settings though the property window of the role, specify the connection string and storage account for cloud and local. And the can also use the endpoint which defined in role environment to our Node.js application. In Node.js SDK we can get an object from “azure.RoleEnvironment”, which provides the functionalities to retrieve the configuration settings and endpoints, etc.. In the code below I defined the connection string variants and then use the SDK to retrieve and initialize the table client. 1: var connectionString = ""; 2: var storageAccountName = ""; 3: var storageAccountKey = ""; 4: var tableName = ""; 5: var client; 6:  7: azure.RoleEnvironment.getConfigurationSettings(function (error, settings) { 8: if (error) { 9: console.log("ERROR: getConfigurationSettings"); 10: console.log(JSON.stringify(error)); 11: } 12: else { 13: console.log(JSON.stringify(settings)); 14: connectionString = settings["SqlConnectionString"]; 15: storageAccountName = settings["StorageAccountName"]; 16: storageAccountKey = settings["StorageAccountKey"]; 17: tableName = settings["TableName"]; 18:  19: console.log("connectionString = %s", connectionString); 20: console.log("storageAccountName = %s", storageAccountName); 21: console.log("storageAccountKey = %s", storageAccountKey); 22: console.log("tableName = %s", tableName); 23:  24: client = azure.createTableService(storageAccountName, storageAccountKey); 25: } 26: }); In this way we don’t need to amend the code for the configurations between local and cloud environment since the service runtime will take care of it. At the end of the code we will listen the application on the port retrieved from SDK as well. 1: azure.RoleEnvironment.getCurrentRoleInstance(function (error, instance) { 2: if (error) { 3: console.log("ERROR: getCurrentRoleInstance"); 4: console.log(JSON.stringify(error)); 5: } 6: else { 7: console.log(JSON.stringify(instance)); 8: if (instance["endpoints"] && instance["endpoints"]["nodejs"]) { 9: var endpoint = instance["endpoints"]["nodejs"]; 10: app.listen(endpoint["port"]); 11: } 12: else { 13: app.listen(8080); 14: } 15: } 16: }); But if we tested the application right now we will find that it cannot retrieve any values from service runtime. This is because by default, the entry point of this role was defined to the worker role class. In windows azure environment the service runtime will open a named pipeline to the entry point instance, so that it can connect to the runtime and retrieve values. But in this case, since the entry point was worker role and the Node.js was opened inside the role, the named pipeline was established between our worker role class and service runtime, so our Node.js application cannot use it. To fix this problem we need to open the CSDEF file under the azure project, add a new element named Runtime. Then add an element named EntryPoint which specify the Node.js command line. So that the Node.js application will have the connection to service runtime, then it’s able to read the configurations. Start the Node.js at local emulator we can find it retrieved the connections, storage account for local. And if we publish our application to azure then it works with WASD and storage service through the configurations for cloud.   Summary In this post I demonstrated how to use Windows Azure SDK for Node.js to interact with storage service, especially the table service. I also demonstrated on how to use WACS service runtime, how to retrieve the configuration settings and the endpoint information. And in order to make the service runtime available to my Node.js application I need to create an entry point element in CSDEF file and set “node.exe” as the entry point. I used five posts to introduce and demonstrate on how to run a Node.js application on Windows platform, how to use Windows Azure Web Site and Windows Azure Cloud Service worker role to host our Node.js application. I also described how to work with other services provided by Windows Azure platform through Windows Azure SDK for Node.js. Node.js is a very new and young network application platform. But since it’s very simple and easy to learn and deploy, as well as, it utilizes single thread non-blocking IO model, Node.js became more and more popular on web application and web service development especially for those IO sensitive projects. And as Node.js is very good at scaling-out, it’s more useful on cloud computing platform. Use Node.js on Windows platform is new, too. The modules for SQL database and Windows Azure SDK are still under development and enhancement. It doesn’t support SQL parameter in “node-sqlserver”. It does support using storage connection string to create the storage client in “azure”. But Microsoft is working on make them easier to use, working on add more features and functionalities.   PS, you can download the source code here. You can download the source code of my “Copy all always” tool here.   Hope this helps, Shaun All documents and related graphics, codes are provided "AS IS" without warranty of any kind. Copyright © Shaun Ziyan Xu. This work is licensed under the Creative Commons License.

    Read the article

  • Error starting modern compiler

    - by saloni
    In my servlet , I m using Tomcat 5.0 and JRE is 1.5.0 but it is giving error when I click on the URL . As when I created a war file of my project and deployed in tomcat than it is working fine . It means that only problem with my eclipse configuration ERROR IS : - Apr 5, 2010 3:20:22 PM org.apache.jasper.compiler.Compiler generateClass SEVERE: Javac exception Error starting modern compiler at org.apache.tools.ant.taskdefs.compilers.Javac13.execute(Javac13.java:69) at org.apache.tools.ant.taskdefs.Javac.compile(Javac.java:942) at org.apache.tools.ant.taskdefs.Javac.execute(Javac.java:764) at org.apache.jasper.compiler.Compiler.generateClass(Compiler.java:382) at org.apache.jasper.compiler.Compiler.compile(Compiler.java:472) at org.apache.jasper.compiler.Compiler.compile(Compiler.java:451) at org.apache.jasper.compiler.Compiler.compile(Compiler.java:439) at org.apache.jasper.JspCompilationContext.compile(JspCompilationContext.java:511) at org.apache.jasper.servlet.JspServletWrapper.service(JspServletWrapper.java:295) at org.apache.jasper.servlet.JspServlet.serviceJspFile(JspServlet.java:292) at org.apache.jasper.servlet.JspServlet.service(JspServlet.java:236) at javax.servlet.http.HttpServlet.service(HttpServlet.java:802) at org.apache.catalina.core.ApplicationFilterChain.internalDoFilter(ApplicationFilterChain.java:237) at org.apache.catalina.core.ApplicationFilterChain.doFilter(ApplicationFilterChain.java:157) at org.apache.catalina.core.StandardWrapperValve.invoke(StandardWrapperValve.java:214) at org.apache.catalina.core.StandardValveContext.invokeNext(StandardValveContext.java:104) at org.apache.catalina.core.StandardPipeline.invoke(StandardPipeline.java:520) at org.apache.catalina.core.StandardContextValve.invokeInternal(StandardContextValve.java:198) at org.apache.catalina.core.StandardContextValve.invoke(StandardContextValve.java:152) at org.apache.catalina.core.StandardValveContext.invokeNext(StandardValveContext.java:104) at org.apache.catalina.core.StandardPipeline.invoke(StandardPipeline.java:520) at org.apache.catalina.core.StandardHostValve.invoke(StandardHostValve.java:137) at org.apache.catalina.core.StandardValveContext.invokeNext(StandardValveContext.java:104) at org.apache.catalina.valves.ErrorReportValve.invoke(ErrorReportValve.java:118) at org.apache.catalina.core.StandardValveContext.invokeNext(StandardValveContext.java:102) at org.apache.catalina.core.StandardPipeline.invoke(StandardPipeline.java:520) at org.apache.catalina.core.StandardEngineValve.invoke(StandardEngineValve.java:109) at org.apache.catalina.core.StandardValveContext.invokeNext(StandardValveContext.java:104) at org.apache.catalina.core.StandardPipeline.invoke(StandardPipeline.java:520) at org.apache.catalina.core.ContainerBase.invoke(ContainerBase.java:929) at org.apache.coyote.tomcat5.CoyoteAdapter.service(CoyoteAdapter.java:160) at org.apache.coyote.http11.Http11Processor.process(Http11Processor.java:799) at org.apache.coyote.http11.Http11Protocol$Http11ConnectionHandler.processConnection(Http11Protocol.java:705) at org.apache.tomcat.util.net.TcpWorkerThread.runIt(PoolTcpEndpoint.java:577) at org.apache.tomcat.util.threads.ThreadPool$ControlRunnable.run(ThreadPool.java:683) at java.lang.Thread.run(Unknown Source) Caused by: java.lang.reflect.InvocationTargetException at sun.reflect.NativeMethodAccessorImpl.invoke0(Native Method) at sun.reflect.NativeMethodAccessorImpl.invoke(Unknown Source) at sun.reflect.DelegatingMethodAccessorImpl.invoke(Unknown Source) at java.lang.reflect.Method.invoke(Unknown Source) at org.apache.tools.ant.taskdefs.compilers.Javac13.execute(Javac13.java:61) ... 35 more Caused by: java.lang.VerifyError: class com.sun.tools.javac.jvm.Target overrides final method . at java.lang.ClassLoader.defineClass1(Native Method) at java.lang.ClassLoader.defineClass(Unknown Source) at java.security.SecureClassLoader.defineClass(Unknown Source) at org.apache.catalina.loader.WebappClassLoader.findClassInternal(WebappClassLoader.java:1634) at org.apache.catalina.loader.WebappClassLoader.findClass(WebappClassLoader.java:860) at org.apache.catalina.loader.WebappClassLoader.loadClass(WebappClassLoader.java:1307) at org.apache.catalina.loader.WebappClassLoader.loadClass(WebappClassLoader.java:1189) at java.lang.ClassLoader.loadClassInternal(Unknown Source) at com.sun.tools.javac.Main.compile(Main.java:42) ... 40 more --- Nested Exception --- java.lang.reflect.InvocationTargetException at sun.reflect.NativeMethodAccessorImpl.invoke0(Native Method) at sun.reflect.NativeMethodAccessorImpl.invoke(Unknown Source) at sun.reflect.DelegatingMethodAccessorImpl.invoke(Unknown Source) at java.lang.reflect.Method.invoke(Unknown Source) at org.apache.tools.ant.taskdefs.compilers.Javac13.execute(Javac13.java:61) at org.apache.tools.ant.taskdefs.Javac.compile(Javac.java:942) at org.apache.tools.ant.taskdefs.Javac.execute(Javac.java:764) at org.apache.jasper.compiler.Compiler.generateClass(Compiler.java:382) at org.apache.jasper.compiler.Compiler.compile(Compiler.java:472) at org.apache.jasper.compiler.Compiler.compile(Compiler.java:451) at org.apache.jasper.compiler.Compiler.compile(Compiler.java:439) at org.apache.jasper.JspCompilationContext.compile(JspCompilationContext.java:511) at org.apache.jasper.servlet.JspServletWrapper.service(JspServletWrapper.java:295) at org.apache.jasper.servlet.JspServlet.serviceJspFile(JspServlet.java:292) at org.apache.jasper.servlet.JspServlet.service(JspServlet.java:236) at javax.servlet.http.HttpServlet.service(HttpServlet.java:802) at org.apache.catalina.core.ApplicationFilterChain.internalDoFilter(ApplicationFilterChain.java:237) at org.apache.catalina.core.ApplicationFilterChain.doFilter(ApplicationFilterChain.java:157) at org.apache.catalina.core.StandardWrapperValve.invoke(StandardWrapperValve.java:214) at org.apache.catalina.core.StandardValveContext.invokeNext(StandardValveContext.java:104) at org.apache.catalina.core.StandardPipeline.invoke(StandardPipeline.java:520) at org.apache.catalina.core.StandardContextValve.invokeInternal(StandardContextValve.java:198) at org.apache.catalina.core.StandardContextValve.invoke(StandardContextValve.java:152) at org.apache.catalina.core.StandardValveContext.invokeNext(StandardValveContext.java:104) at org.apache.catalina.core.StandardPipeline.invoke(StandardPipeline.java:520) at org.apache.catalina.core.StandardHostValve.invoke(StandardHostValve.java:137) at org.apache.catalina.core.StandardValveContext.invokeNext(StandardValveContext.java:104) at org.apache.catalina.valves.ErrorReportValve.invoke(ErrorReportValve.java:118) at org.apache.catalina.core.StandardValveContext.invokeNext(StandardValveContext.java:102) at org.apache.catalina.core.StandardPipeline.invoke(StandardPipeline.java:520) at org.apache.catalina.core.StandardEngineValve.invoke(StandardEngineValve.java:109) at org.apache.catalina.core.StandardValveContext.invokeNext(StandardValveContext.java:104) at org.apache.catalina.core.StandardPipeline.invoke(StandardPipeline.java:520) at org.apache.catalina.core.ContainerBase.invoke(ContainerBase.java:929) at org.apache.coyote.tomcat5.CoyoteAdapter.service(CoyoteAdapter.java:160) at org.apache.coyote.http11.Http11Processor.process(Http11Processor.java:799) at org.apache.coyote.http11.Http11Protocol$Http11ConnectionHandler.processConnection(Http11Protocol.java:705) at org.apache.tomcat.util.net.TcpWorkerThread.runIt(PoolTcpEndpoint.java:577) at org.apache.tomcat.util.threads.ThreadPool$ControlRunnable.run(ThreadPool.java:683) at java.lang.Thread.run(Unknown Source) Caused by: java.lang.VerifyError: class com.sun.tools.javac.jvm.Target overrides final method . at java.lang.ClassLoader.defineClass1(Native Method) at java.lang.ClassLoader.defineClass(Unknown Source) at java.security.SecureClassLoader.defineClass(Unknown Source) at org.apache.catalina.loader.WebappClassLoader.findClassInternal(WebappClassLoader.java:1634) at org.apache.catalina.loader.WebappClassLoader.findClass(WebappClassLoader.java:860) at org.apache.catalina.loader.WebappClassLoader.loadClass(WebappClassLoader.java:1307) at org.apache.catalina.loader.WebappClassLoader.loadClass(WebappClassLoader.java:1189) at java.lang.ClassLoader.loadClassInternal(Unknown Source) at com.sun.tools.javac.Main.compile(Main.java:42) ... 40 more Apr 5, 2010 3:20:22 PM org.apache.jasper.compiler.Compiler generateClass SEVERE: Env: Compile: javaFileName=/D:/OffViv/JAVA_IDE/workspace/.metadata/.plugins/org.eclipse.wst.server.core/tmp0/work/Catalina/localhost/SampleSaloni//org/apache/jsp/page\form_jsp.java classpath=/D:/OffViv/JAVA_IDE/workspace/.metadata/.plugins/org.eclipse.wst.server.core/tmp0/wtpwebapps/SampleSaloni/WEB-INF/classes/;/D:/OffViv/JAVA_IDE/workspace/.metadata/.plugins/org.eclipse.wst.server.core/tmp0/wtpwebapps/SampleSaloni/WEB-INF/lib/ant-launcher.jar;/D:/OffViv/JAVA_IDE/workspace/.metadata/.plugins/org.eclipse.wst.server.core/tmp0/wtpwebapps/SampleSaloni/WEB-INF/lib/ant.jar;/D:/OffViv/JAVA_IDE/workspace/.metadata/.plugins/org.eclipse.wst.server.core/tmp0/wtpwebapps/SampleSaloni/WEB-INF/lib/commons-collections-3.1.jar;/D:/OffViv/JAVA_IDE/workspace/.metadata/.plugins/org.eclipse.wst.server.core/tmp0/wtpwebapps/SampleSaloni/WEB-INF/lib/commons-dbcp-1.2.1.jar;/D:/OffViv/JAVA_IDE/workspace/.metadata/.plugins/org.eclipse.wst.server.core/tmp0/wtpwebapps/SampleSaloni/WEB-INF/lib/commons-el.jar;/D:/OffViv/JAVA_IDE/workspace/.metadata/.plugins/org.eclipse.wst.server.core/tmp0/wtpwebapps/SampleSaloni/WEB-INF/lib/commons-pool-1.2.jar;/D:/OffViv/JAVA_IDE/workspace/.metadata/.plugins/org.eclipse.wst.server.core/tmp0/wtpwebapps/SampleSaloni/WEB-INF/lib/jasper-compiler.jar;/D:/OffViv/JAVA_IDE/workspace/.metadata/.plugins/org.eclipse.wst.server.core/tmp0/wtpwebapps/SampleSaloni/WEB-INF/lib/jasper-runtime.jar;/D:/OffViv/JAVA_IDE/workspace/.metadata/.plugins/org.eclipse.wst.server.core/tmp0/wtpwebapps/SampleSaloni/WEB-INF/lib/jsp-api.jar;/D:/OffViv/JAVA_IDE/workspace/.metadata/.plugins/org.eclipse.wst.server.core/tmp0/wtpwebapps/SampleSaloni/WEB-INF/lib/naming-common.jar;/D:/OffViv/JAVA_IDE/workspace/.metadata/.plugins/org.eclipse.wst.server.core/tmp0/wtpwebapps/SampleSaloni/WEB-INF/lib/naming-factory.jar;/D:/OffViv/JAVA_IDE/workspace/.metadata/.plugins/org.eclipse.wst.server.core/tmp0/wtpwebapps/SampleSaloni/WEB-INF/lib/naming-java.jar;/D:/OffViv/JAVA_IDE/workspace/.metadata/.plugins/org.eclipse.wst.server.core/tmp0/wtpwebapps/SampleSaloni/WEB-INF/lib/naming-resources.jar;/D:/OffViv/JAVA_IDE/workspace/.metadata/.plugins/org.eclipse.wst.server.core/tmp0/wtpwebapps/SampleSaloni/WEB-INF/lib/tools.jar;D:\OffViv\JAVA_IDE\workspace\.metadata\.plugins\org.eclipse.wst.server.core\tmp0\work\Catalina\localhost\SampleSaloni;/D:/OffViv/JAVA_IDE/workspace/.metadata/.plugins/org.eclipse.wst.server.core/tmp0/wtpwebapps/SampleSaloni/WEB-INF/classes/;/D:/OffViv/JAVA_IDE/workspace/.metadata/.plugins/org.eclipse.wst.server.core/tmp0/wtpwebapps/SampleSaloni/WEB-INF/lib/ant-launcher.jar;/D:/OffViv/JAVA_IDE/workspace/.metadata/.plugins/org.eclipse.wst.server.core/tmp0/wtpwebapps/SampleSaloni/WEB-INF/lib/ant.jar;/D:/OffViv/JAVA_IDE/workspace/.metadata/.plugins/org.eclipse.wst.server.core/tmp0/wtpwebapps/SampleSaloni/WEB-INF/lib/commons-collections-3.1.jar;/D:/OffViv/JAVA_IDE/workspace/.metadata/.plugins/org.eclipse.wst.server.core/tmp0/wtpwebapps/SampleSaloni/WEB-INF/lib/commons-dbcp-1.2.1.jar;/D:/OffViv/JAVA_IDE/workspace/.metadata/.plugins/org.eclipse.wst.server.core/tmp0/wtpwebapps/SampleSaloni/WEB-INF/lib/commons-el.jar;/D:/OffViv/JAVA_IDE/workspace/.metadata/.plugins/org.eclipse.wst.server.core/tmp0/wtpwebapps/SampleSaloni/WEB-INF/lib/commons-pool-1.2.jar;/D:/OffViv/JAVA_IDE/workspace/.metadata/.plugins/org.eclipse.wst.server.core/tmp0/wtpwebapps/SampleSaloni/WEB-INF/lib/jasper-compiler.jar;/D:/OffViv/JAVA_IDE/workspace/.metadata/.plugins/org.eclipse.wst.server.core/tmp0/wtpwebapps/SampleSaloni/WEB-INF/lib/jasper-runtime.jar;/D:/OffViv/JAVA_IDE/workspace/.metadata/.plugins/org.eclipse.wst.server.core/tmp0/wtpwebapps/SampleSaloni/WEB-INF/lib/jsp-api.jar;/D:/OffViv/JAVA_IDE/workspace/.metadata/.plugins/org.eclipse.wst.server.core/tmp0/wtpwebapps/SampleSaloni/WEB-INF/lib/naming-common.jar;/D:/OffViv/JAVA_IDE/workspace/.metadata/.plugins/org.eclipse.wst.server.core/tmp0/wtpwebapps/SampleSaloni/WEB-INF/lib/naming-factory.jar;/D:/OffViv/JAVA_IDE/workspace/.metadata/.plugins/org.eclipse.wst.server.core/tmp0/wtpwebapps/SampleSaloni/WEB-INF/lib/naming-java.jar;/D:/OffViv/JAVA_IDE/workspace/.metadata/.plugins/org.eclipse.wst.server.core/tmp0/wtpwebapps/SampleSaloni/WEB-INF/lib/naming-resources.jar;/D:/OffViv/JAVA_IDE/workspace/.metadata/.plugins/org.eclipse.wst.server.core/tmp0/wtpwebapps/SampleSaloni/WEB-INF/lib/tools.jar;D:/software setups/jakarta-tomcat-5.0.28/common/classes/;D:/software setups/jakarta-tomcat-5.0.28/common/lib/ant-launcher.jar;D:/software setups/jakarta-tomcat-5.0.28/common/lib/ant.jar;D:/software setups/jakarta-tomcat-5.0.28/common/lib/commons-collections-3.1.jar;D:/software setups/jakarta-tomcat-5.0.28/common/lib/commons-dbcp-1.2.1.jar;D:/software setups/jakarta-tomcat-5.0.28/common/lib/commons-el.jar;D:/software setups/jakarta-tomcat-5.0.28/common/lib/commons-pool-1.2.jar;D:/software setups/jakarta-tomcat-5.0.28/common/lib/jasper-compiler.jar;D:/software setups/jakarta-tomcat-5.0.28/common/lib/jasper-runtime.jar;D:/software setups/jakarta-tomcat-5.0.28/common/lib/jsp-api.jar;D:/software setups/jakarta-tomcat-5.0.28/common/lib/naming-common.jar;D:/software setups/jakarta-tomcat-5.0.28/common/lib/naming-factory.jar;D:/software setups/jakarta-tomcat-5.0.28/common/lib/naming-java.jar;D:/software setups/jakarta-tomcat-5.0.28/common/lib/naming-resources.jar;D:/software setups/jakarta-tomcat-5.0.28/common/lib/servlet-api.jar;D:/software setups/jakarta-tomcat-5.0.28/common/lib/tools.jar;/D:/software%20setups/jakarta-tomcat-5.0.28/bin/bootstrap.jar;/C:/Program%20Files/Java/jre1.5.0_09/lib/ext/dnsns.jar;/C:/Program%20Files/Java/jre1.5.0_09/lib/ext/sunjce_provider.jar;/C:/Program%20Files/Java/jre1.5.0_09/lib/ext/sunpkcs11.jar cp=D:\software setups\jakarta-tomcat-5.0.28\bin\bootstrap.jar cp=D:\OffViv\JAVA_IDE\workspace\.metadata\.plugins\org.eclipse.wst.server.core\tmp0\wtpwebapps\SampleSaloni\WEB-INF\classes cp=D:\OffViv\JAVA_IDE\workspace\.metadata\.plugins\org.eclipse.wst.server.core\tmp0\wtpwebapps\SampleSaloni\WEB-INF\lib\ant-launcher.jar cp=D:\OffViv\JAVA_IDE\workspace\.metadata\.plugins\org.eclipse.wst.server.core\tmp0\wtpwebapps\SampleSaloni\WEB-INF\lib\ant.jar cp=D:\OffViv\JAVA_IDE\workspace\.metadata\.plugins\org.eclipse.wst.server.core\tmp0\wtpwebapps\SampleSaloni\WEB-INF\lib\commons-collections-3.1.jar cp=D:\OffViv\JAVA_IDE\workspace\.metadata\.plugins\org.eclipse.wst.server.core\tmp0\wtpwebapps\SampleSaloni\WEB-INF\lib\commons-dbcp-1.2.1.jar cp=D:\OffViv\JAVA_IDE\workspace\.metadata\.plugins\org.eclipse.wst.server.core\tmp0\wtpwebapps\SampleSaloni\WEB-INF\lib\commons-el.jar cp=D:\OffViv\JAVA_IDE\workspace\.metadata\.plugins\org.eclipse.wst.server.core\tmp0\wtpwebapps\SampleSaloni\WEB-INF\lib\commons-pool-1.2.jar cp=D:\OffViv\JAVA_IDE\workspace\.metadata\.plugins\org.eclipse.wst.server.core\tmp0\wtpwebapps\SampleSaloni\WEB-INF\lib\jasper-compiler.jar cp=D:\OffViv\JAVA_IDE\workspace\.metadata\.plugins\org.eclipse.wst.server.core\tmp0\wtpwebapps\SampleSaloni\WEB-INF\lib\jasper-runtime.jar cp=D:\OffViv\JAVA_IDE\workspace\.metadata\.plugins\org.eclipse.wst.server.core\tmp0\wtpwebapps\SampleSaloni\WEB-INF\lib\jsp-api.jar cp=D:\OffViv\JAVA_IDE\workspace\.metadata\.plugins\org.eclipse.wst.server.core\tmp0\wtpwebapps\SampleSaloni\WEB-INF\lib\naming-common.jar cp=D:\OffViv\JAVA_IDE\workspace\.metadata\.plugins\org.eclipse.wst.server.core\tmp0\wtpwebapps\SampleSaloni\WEB-INF\lib\naming-factory.jar cp=D:\OffViv\JAVA_IDE\workspace\.metadata\.plugins\org.eclipse.wst.server.core\tmp0\wtpwebapps\SampleSaloni\WEB-INF\lib\naming-java.jar cp=D:\OffViv\JAVA_IDE\workspace\.metadata\.plugins\org.eclipse.wst.server.core\tmp0\wtpwebapps\SampleSaloni\WEB-INF\lib\naming-resources.jar cp=D:\OffViv\JAVA_IDE\workspace\.metadata\.plugins\org.eclipse.wst.server.core\tmp0\wtpwebapps\SampleSaloni\WEB-INF\lib\tools.jar cp=D:\OffViv\JAVA_IDE\workspace\.metadata\.plugins\org.eclipse.wst.server.core\tmp0\work\Catalina\localhost\SampleSaloni cp=D:\OffViv\JAVA_IDE\workspace\.metadata\.plugins\org.eclipse.wst.server.core\tmp0\wtpwebapps\SampleSaloni\WEB-INF\classes cp=D:\OffViv\JAVA_IDE\workspace\.metadata\.plugins\org.eclipse.wst.server.core\tmp0\wtpwebapps\SampleSaloni\WEB-INF\lib\ant-launcher.jar cp=D:\OffViv\JAVA_IDE\workspace\.metadata\.plugins\org.eclipse.wst.server.core\tmp0\wtpwebapps\SampleSaloni\WEB-INF\lib\ant.jar cp=D:\OffViv\JAVA_IDE\workspace\.metadata\.plugins\org.eclipse.wst.server.core\tmp0\wtpwebapps\SampleSaloni\WEB-INF\lib\commons-collections-3.1.jar cp=D:\OffViv\JAVA_IDE\workspace\.metadata\.plugins\org.eclipse.wst.server.core\tmp0\wtpwebapps\SampleSaloni\WEB-INF\lib\commons-dbcp-1.2.1.jar cp=D:\OffViv\JAVA_IDE\workspace\.metadata\.plugins\org.eclipse.wst.server.core\tmp0\wtpwebapps\SampleSaloni\WEB-INF\lib\commons-el.jar cp=D:\OffViv\JAVA_IDE\workspace\.metadata\.plugins\org.eclipse.wst.server.core\tmp0\wtpwebapps\SampleSaloni\WEB-INF\lib\commons-pool-1.2.jar cp=D:\OffViv\JAVA_IDE\workspace\.metadata\.plugins\org.eclipse.wst.server.core\tmp0\wtpwebapps\SampleSaloni\WEB-INF\lib\jasper-compiler.jar cp=D:\OffViv\JAVA_IDE\workspace\.metadata\.plugins\org.eclipse.wst.server.core\tmp0\wtpwebapps\SampleSaloni\WEB-INF\lib\jasper-runtime.jar cp=D:\OffViv\JAVA_IDE\workspace\.metadata\.plugins\org.eclipse.wst.server.core\tmp0\wtpwebapps\SampleSaloni\WEB-INF\lib\jsp-api.jar cp=D:\OffViv\JAVA_IDE\workspace\.metadata\.plugins\org.eclipse.wst.server.core\tmp0\wtpwebapps\SampleSaloni\WEB-INF\lib\naming-common.jar cp=D:\OffViv\JAVA_IDE\workspace\.metadata\.plugins\org.eclipse.wst.server.core\tmp0\wtpwebapps\SampleSaloni\WEB-INF\lib\naming-factory.jar cp=D:\OffViv\JAVA_IDE\workspace\.metadata\.plugins\org.eclipse.wst.server.core\tmp0\wtpwebapps\SampleSaloni\WEB-INF\lib\naming-java.jar cp=D:\OffViv\JAVA_IDE\workspace\.metadata\.plugins\org.eclipse.wst.server.core\tmp0\wtpwebapps\SampleSaloni\WEB-INF\lib\naming-resources.jar cp=D:\OffViv\JAVA_IDE\workspace\.metadata\.plugins\org.eclipse.wst.server.core\tmp0\wtpwebapps\SampleSaloni\WEB-INF\lib\tools.jar cp=D:\software setups\jakarta-tomcat-5.0.28\common\classes cp=D:\software setups\jakarta-tomcat-5.0.28\common\lib\ant-launcher.jar cp=D:\software setups\jakarta-tomcat-5.0.28\common\lib\ant.jar cp=D:\software setups\jakarta-tomcat-5.0.28\common\lib\commons-collections-3.1.jar cp=D:\software setups\jakarta-tomcat-5.0.28\common\lib\commons-dbcp-1.2.1.jar cp=D:\software setups\jakarta-tomcat-5.0.28\common\lib\commons-el.jar cp=D:\software setups\jakarta-tomcat-5.0.28\common\lib\commons-pool-1.2.jar cp=D:\software setups\jakarta-tomcat-5.0.28\common\lib\jasper-compiler.jar cp=D:\software setups\jakarta-tomcat-5.0.28\common\lib\jasper-runtime.jar cp=D:\software setups\jakarta-tomcat-5.0.28\common\lib\jsp-api.jar cp=D:\software setups\jakarta-tomcat-5.0.28\common\lib\naming-common.jar cp=D:\software setups\jakarta-tomcat-5.0.28\common\lib\naming-factory.jar cp=D:\software setups\jakarta-tomcat-5.0.28\common\lib\naming-java.jar cp=D:\software setups\jakarta-tomcat-5.0.28\common\lib\naming-resources.jar cp=D:\software setups\jakarta-tomcat-5.0.28\common\lib\servlet-api.jar cp=D:\software setups\jakarta-tomcat-5.0.28\common\lib\tools.jar cp=D:\software%20setups\jakarta-tomcat-5.0.28\bin\bootstrap.jar cp=C:\Program%20Files\Java\jre1.5.0_09\lib\ext\dnsns.jar cp=C:\Program%20Files\Java\jre1.5.0_09\lib\ext\sunjce_provider.jar cp=C:\Program%20Files\Java\jre1.5.0_09\lib\ext\sunpkcs11.jar work dir=D:\OffViv\JAVA_IDE\workspace\.metadata\.plugins\org.eclipse.wst.server.core\tmp0\work\Catalina\localhost\SampleSaloni extension dir=C:\Program Files\Java\jre1.5.0_09\lib\ext srcDir=D:\OffViv\JAVA_IDE\workspace\.metadata\.plugins\org.eclipse.wst.server.core\tmp0\work\Catalina\localhost\SampleSaloni include=org/apache/jsp/page/form_jsp.java Apr 5, 2010 3:20:22 PM org.apache.jasper.compiler.Compiler generateClass SEVERE: Error compiling file: /D:/OffViv/JAVA_IDE/workspace/.metadata/.plugins/org.eclipse.wst.server.core/tmp0/work/Catalina/localhost/SampleSaloni//org/apache/jsp/page\form_jsp.java [javac] Compiling 1 source file

    Read the article

  • How can I convert XML files to one CSV file in C#?

    - by TruMan1
    I have a collection of strings that are XML content. I want to iterate thru my collection and build a CSV file to stream to the user for download (sometimes it can be hundreds in the collection). This is my loop: foreach (string response in items.Responses) { string xmlResponse = response; //BUILD CSV HERE } This is what my XML content looks like for each iteration (xmlResponse). I want to put it in a flat file including the "properties" attributes: <?xml version="1.0"?> <response> <properties id="60375c90-9dd7-400f-aafb-a8726df409a9" name="Account Request" date="Thursday, March 04, 2010 2:14:07 PM" page="http://mydomain/sitefinity/CreateAccount.aspx" ip="192.168.1.255" browser="Mozilla/5.0 (Windows; U; Windows NT 6.1; en-US; rv:1.9.1.8) Gecko/20100202 Firefox/3.5.8" referrer="http://mydomain/sitefinity/CreateAccount.aspx" confirmation="True" subject="Email from website: Account Request Form" sender="[email protected]" recipients="[email protected], , " /> <fields> <field> <label>Personal Details</label> <value>Personal Details</value> </field> <field> <label>Name</label> <value>Tim Wales</value> </field> <field> <label>Email</label> <value>[email protected]</value> </field> <field> <label>Website</label> <value></value> </field> <field> <label>Password</label> <value></value> </field> <field> <label>Phone</label> <value></value> </field> <field> <label>Years in Business</label> <value></value> </field> <field> <label>Background</label> <value>Background</value> </field> <field> <label>Place of Birth</label> <value>Earth</value> </field> <field> <label>Date of Birth</label> <value></value> </field> <field> <label>Some Label</label> <value>Some Label</value> </field> <field> <label>Industry</label> <value> Technology Other</value> </field> <field> <label>Pets</label> <value>Dog</value> </field> <field> <label>Your View</label> <value>Positive</value> </field> <field> <label>Misc</label> <value>Misc</value> </field> <field> <label>Comments</label> <value></value> </field> <field> <label>Agree to Terms?</label> <value>True</value> </field> </fields> </response> <?xml version="1.0"?> <response> <properties id="60375c90-9dd7-400f-aafb-a8726df409a9" Form="Account Request" Date="Tuesday, March 16, 2010 6:21:07 PM" Page="http://mydomain/sitefinity/Home.aspx" IP="fe80::1c0f57:9ee3%10" Browser="Mozilla/4.0 (compatible; MSIE 7.0; Windows NT 6.0; Trident/4.0; SLCC1; .NET CLR 2.0.50727; InfoPath.2; .NET CLR 3.5.21022; .NET CLR 3.5.30729; .NET CLR 3.0.30729)" Referrer="http://mydomain/sitefinity/Home.aspx" Subject="Email from website: Account Request Form" Sender="[email protected]" Recipients="[email protected]" Confirmation="True" /> <fields> <field> <label>Personal Details</label> <value>Personal Details</value> </field> <field> <label>Name</label> <value>erger</value> </field> <field> <label>Email</label> <value></value> </field> <field> <label>Website</label> <value></value> </field> <field> <label>Password</label> <value></value> </field> <field> <label>Phone</label> <value></value> </field> <field> <label>Years in Business</label> <value></value> </field> <field> <label>Background</label> <value>Background</value> </field> <field> <label>Place of Birth</label> <value>Earth</value> </field> <field> <label>Date of Birth</label> <value></value> </field> <field> <label>Some Label</label> <value>Some Label</value> </field> <field> <label>Industry</label> <value> Technology Service</value> </field> <field> <label>Pets</label> <value>Dog</value> </field> <field> <label>Your View</label> <value>Positive</value> </field> <field> <label>Misc</label> <value>Misc</value> </field> <field> <label>Comments</label> <value></value> </field> <field> <label>Agree to Terms?</label> <value>True</value> </field> </fields> </response> <?xml version="1.0"?> <response> <properties id="60375c90-9dd7-400f-aafb-a8726df409a9" Form="Account Request" Date="Tuesday, March 16, 2010 4:50:17 PM" Page="http://mydomain/sitefinity/Home.aspx" IP="fe80::1c0f:ee3%10" Browser="Mozilla/4.0 (compatible; MSIE 7.0; Windows NT 6.0; Trident/4.0; SLCC1; .NET CLR 2.0.50727; InfoPath.2; .NET CLR 3.5.21022; .NET CLR 3.5.30729; .NET CLR 3.0.30729)" Referrer="http://mydomain/sitefinity/Home.aspx" Subject="Email from website: Account Request Form" Sender="[email protected]" Recipients="[email protected]" Confirmation="True" /> <fields> <field> <label>Personal Details</label> <value>Personal Details</value> </field> <field> <label>Name</label> <value>esfs</value> </field> <field> <label>Email</label> <value></value> </field> <field> <label>Website</label> <value></value> </field> <field> <label>Password</label> <value></value> </field> <field> <label>Phone</label> <value></value> </field> <field> <label>Years in Business</label> <value></value> </field> <field> <label>Background</label> <value>Background</value> </field> <field> <label>Place of Birth</label> <value>Earth</value> </field> <field> <label>Date of Birth</label> <value></value> </field> <field> <label>Some Label</label> <value>Some Label</value> </field> <field> <label>Industry</label> <value> Technology Service</value> </field> <field> <label>Pets</label> <value>Dog</value> </field> <field> <label>Your View</label> <value>Positive</value> </field> <field> <label>Misc</label> <value>Misc</value> </field> <field> <label>Comments</label> <value></value> </field> <field> <label>Agree to Terms?</label> <value>True</value> </field> </fields> </response> Can anyone help with this?

    Read the article

  • LVM / Device Mapper maps wrong device

    - by DaDaDom
    Hi, I run a LVM setup on a raid1 created by mdadm. md2 is based on sda6 (major:minor 8:6) and sdb6 (8:22). md2 is partition 9:2. The VG on top of md2 has 4 LVs, var, home, usr, tmp. First the problem: While booting it seems as if the device mapper takes the wrong partition for the mapping! Immediately after boot the information is like ~# dmsetup table systemlvm-home: 0 4194304 linear 8:22 384 systemlvm-home: 4194304 16777216 linear 8:22 69206400 systemlvm-home: 20971520 8388608 linear 8:22 119538048 systemlvm-home: 29360128 6291456 linear 8:22 243270016 systemlvm-tmp: 0 2097152 linear 8:22 41943424 systemlvm-usr: 0 10485760 linear 8:22 20971904 systemlvm-var: 0 10485760 linear 8:22 10486144 systemlvm-var: 10485760 6291456 linear 8:22 4194688 systemlvm-var: 16777216 4194304 linear 8:22 44040576 systemlvm-var: 20971520 10485760 linear 8:22 31457664 systemlvm-var: 31457280 20971520 linear 8:22 48234880 systemlvm-var: 52428800 33554432 linear 8:22 85983616 systemlvm-var: 85983232 115343360 linear 8:22 127926656 ~# cat /proc/mdstat Personalities : [raid1] md2 : active (auto-read-only) raid1 sda6[0] 151798080 blocks [2/1] [U_] md0 : active raid1 sda1[0] sdb1[1] 96256 blocks [2/2] [UU] md1 : active raid1 sda2[0] sdb2[1] 2931776 blocks [2/2] [UU] I have to manually "lvchange -an" all LVs, add /dev/sdb6 back to the raid and reactivate the LVs, then all is fine. But it prevents me from automounting the partitions and obviously leads to a bunch of other problems. If everything works fine, the information is like ~$ cat /proc/mdstat Personalities : [raid1] md2 : active raid1 sdb6[1] sda6[0] 151798080 blocks [2/2] [UU] ... ~# dmsetup table systemlvm-home: 0 4194304 linear 9:2 384 systemlvm-home: 4194304 16777216 linear 9:2 69206400 systemlvm-home: 20971520 8388608 linear 9:2 119538048 systemlvm-home: 29360128 6291456 linear 9:2 243270016 systemlvm-tmp: 0 2097152 linear 9:2 41943424 systemlvm-usr: 0 10485760 linear 9:2 20971904 systemlvm-var: 0 10485760 linear 9:2 10486144 systemlvm-var: 10485760 6291456 linear 9:2 4194688 systemlvm-var: 16777216 4194304 linear 9:2 44040576 systemlvm-var: 20971520 10485760 linear 9:2 31457664 systemlvm-var: 31457280 20971520 linear 9:2 48234880 systemlvm-var: 52428800 33554432 linear 9:2 85983616 systemlvm-var: 85983232 115343360 linear 9:2 127926656 I think that LVM for some reason just "takes" /dev/sdb6 which is then missing in the raid. I tried almost all options in the lvm.conf but none seems to work. Below is some more information, like config files. Does anyone have any idea about what is going on here and how to prevent that? If you need any additional information, please let me know Thanks in advance! Dominik The information (off a "repaired" system): ~# cat /etc/debian_version 5.0.4 ~# uname -a Linux kermit 2.6.26-2-686 #1 SMP Wed Feb 10 08:59:21 UTC 2010 i686 GNU/Linux ~# lvm version LVM version: 2.02.39 (2008-06-27) Library version: 1.02.27 (2008-06-25) Driver version: 4.13.0 ~# cat /etc/mdadm/mdadm.conf DEVICE partitions ARRAY /dev/md1 level=raid1 num-devices=2 metadata=00.90 UUID=11e9dc6c:1da99f3f:b3088ca6:c6fe60e9 ARRAY /dev/md0 level=raid1 num-devices=2 metadata=00.90 UUID=92ed1e4b:897361d3:070682b3:3baa4fa1 ARRAY /dev/md2 level=raid1 num-devices=2 metadata=00.90 UUID=601d4642:39dc80d7:96e8bbac:649924ba ~# mount /dev/md1 on / type ext3 (rw,errors=remount-ro) tmpfs on /lib/init/rw type tmpfs (rw,nosuid,mode=0755) proc on /proc type proc (rw,noexec,nosuid,nodev) sysfs on /sys type sysfs (rw,noexec,nosuid,nodev) procbususb on /proc/bus/usb type usbfs (rw) udev on /dev type tmpfs (rw,mode=0755) tmpfs on /dev/shm type tmpfs (rw,nosuid,nodev) devpts on /dev/pts type devpts (rw,noexec,nosuid,gid=5,mode=620) /dev/md0 on /boot type ext3 (rw) /dev/mapper/systemlvm-usr on /usr type reiserfs (rw) /dev/mapper/systemlvm-tmp on /tmp type reiserfs (rw) /dev/mapper/systemlvm-home on /home type reiserfs (rw) /dev/mapper/systemlvm-var on /var type reiserfs (rw) ~# grep -v ^$ /etc/lvm/lvm.conf | grep -v "#" devices { dir = "/dev" scan = [ "/dev" ] preferred_names = [ ] filter = [ "a|/dev/md.*|", "r/.*/" ] cache_dir = "/etc/lvm/cache" cache_file_prefix = "" write_cache_state = 1 sysfs_scan = 1 md_component_detection = 1 ignore_suspended_devices = 0 } log { verbose = 0 syslog = 1 overwrite = 0 level = 0 indent = 1 command_names = 0 prefix = " " } backup { backup = 1 backup_dir = "/etc/lvm/backup" archive = 1 archive_dir = "/etc/lvm/archive" retain_min = 10 retain_days = 30 } shell { history_size = 100 } global { umask = 077 test = 0 units = "h" activation = 1 proc = "/proc" locking_type = 1 fallback_to_clustered_locking = 1 fallback_to_local_locking = 1 locking_dir = "/lib/init/rw" } activation { missing_stripe_filler = "/dev/ioerror" reserved_stack = 256 reserved_memory = 8192 process_priority = -18 mirror_region_size = 512 readahead = "auto" mirror_log_fault_policy = "allocate" mirror_device_fault_policy = "remove" } :~# vgscan -vvv Processing: vgscan -vvv O_DIRECT will be used Setting global/locking_type to 1 File-based locking selected. Setting global/locking_dir to /lib/init/rw Locking /lib/init/rw/P_global WB Wiping cache of LVM-capable devices /dev/block/1:0: Added to device cache /dev/block/1:1: Added to device cache /dev/block/1:10: Added to device cache /dev/block/1:11: Added to device cache /dev/block/1:12: Added to device cache /dev/block/1:13: Added to device cache /dev/block/1:14: Added to device cache /dev/block/1:15: Added to device cache /dev/block/1:2: Added to device cache /dev/block/1:3: Added to device cache /dev/block/1:4: Added to device cache /dev/block/1:5: Added to device cache /dev/block/1:6: Added to device cache /dev/block/1:7: Added to device cache /dev/block/1:8: Added to device cache /dev/block/1:9: Added to device cache /dev/block/253:0: Added to device cache /dev/block/253:1: Added to device cache /dev/block/253:2: Added to device cache /dev/block/253:3: Added to device cache /dev/block/8:0: Added to device cache /dev/block/8:1: Added to device cache /dev/block/8:16: Added to device cache /dev/block/8:17: Added to device cache /dev/block/8:18: Added to device cache /dev/block/8:19: Added to device cache /dev/block/8:2: Added to device cache /dev/block/8:21: Added to device cache /dev/block/8:22: Added to device cache /dev/block/8:3: Added to device cache /dev/block/8:5: Added to device cache /dev/block/8:6: Added to device cache /dev/block/9:0: Already in device cache /dev/block/9:1: Already in device cache /dev/block/9:2: Already in device cache /dev/bsg/0:0:0:0: Not a block device /dev/bsg/1:0:0:0: Not a block device /dev/bus/usb/001/001: Not a block device [... many more "not a block device"] /dev/core: Not a block device /dev/cpu_dma_latency: Not a block device /dev/disk/by-id/ata-SAMSUNG_HD160JJ_S08HJ10L507895: Aliased to /dev/block/8:16 in device cache /dev/disk/by-id/ata-SAMSUNG_HD160JJ_S08HJ10L507895-part1: Aliased to /dev/block/8:17 in device cache /dev/disk/by-id/ata-SAMSUNG_HD160JJ_S08HJ10L507895-part2: Aliased to /dev/block/8:18 in device cache /dev/disk/by-id/ata-SAMSUNG_HD160JJ_S08HJ10L507895-part3: Aliased to /dev/block/8:19 in device cache /dev/disk/by-id/ata-SAMSUNG_HD160JJ_S08HJ10L507895-part5: Aliased to /dev/block/8:21 in device cache /dev/disk/by-id/ata-SAMSUNG_HD160JJ_S08HJ10L507895-part6: Aliased to /dev/block/8:22 in device cache /dev/disk/by-id/ata-SAMSUNG_HD160JJ_S08HJ10L526800: Aliased to /dev/block/8:0 in device cache /dev/disk/by-id/ata-SAMSUNG_HD160JJ_S08HJ10L526800-part1: Aliased to /dev/block/8:1 in device cache /dev/disk/by-id/ata-SAMSUNG_HD160JJ_S08HJ10L526800-part2: Aliased to /dev/block/8:2 in device cache /dev/disk/by-id/ata-SAMSUNG_HD160JJ_S08HJ10L526800-part3: Aliased to /dev/block/8:3 in device cache /dev/disk/by-id/ata-SAMSUNG_HD160JJ_S08HJ10L526800-part5: Aliased to /dev/block/8:5 in device cache /dev/disk/by-id/ata-SAMSUNG_HD160JJ_S08HJ10L526800-part6: Aliased to /dev/block/8:6 in device cache /dev/disk/by-id/dm-name-systemlvm-home: Aliased to /dev/block/253:2 in device cache /dev/disk/by-id/dm-name-systemlvm-tmp: Aliased to /dev/block/253:3 in device cache /dev/disk/by-id/dm-name-systemlvm-usr: Aliased to /dev/block/253:1 in device cache /dev/disk/by-id/dm-name-systemlvm-var: Aliased to /dev/block/253:0 in device cache /dev/disk/by-id/dm-uuid-LVM-rL8Oq2dA7oeRYeu1orJA7Ufnb1kjOyvr25N7CRZpUMzR18NfS6zeSeAVnVT98LuU: Aliased to /dev/block/253:0 in device cache /dev/disk/by-id/dm-uuid-LVM-rL8Oq2dA7oeRYeu1orJA7Ufnb1kjOyvr3TpFXtLjYGEwn79IdXsSCZPl8AxmqbmQ: Aliased to /dev/block/253:1 in device cache /dev/disk/by-id/dm-uuid-LVM-rL8Oq2dA7oeRYeu1orJA7Ufnb1kjOyvrc5MJ4KolevMjt85PPBrQuRTkXbx6NvTi: Aliased to /dev/block/253:3 in device cache /dev/disk/by-id/dm-uuid-LVM-rL8Oq2dA7oeRYeu1orJA7Ufnb1kjOyvrYXrfdg5OSYDVkNeiQeQksgCI849Z2hx8: Aliased to /dev/block/253:2 in device cache /dev/disk/by-id/md-uuid-11e9dc6c:1da99f3f:b3088ca6:c6fe60e9: Already in device cache /dev/disk/by-id/md-uuid-601d4642:39dc80d7:96e8bbac:649924ba: Already in device cache /dev/disk/by-id/md-uuid-92ed1e4b:897361d3:070682b3:3baa4fa1: Already in device cache /dev/disk/by-id/scsi-SATA_SAMSUNG_HD160JJS08HJ10L507895: Aliased to /dev/block/8:16 in device cache /dev/disk/by-id/scsi-SATA_SAMSUNG_HD160JJS08HJ10L507895-part1: Aliased to /dev/block/8:17 in device cache /dev/disk/by-id/scsi-SATA_SAMSUNG_HD160JJS08HJ10L507895-part2: Aliased to /dev/block/8:18 in device cache /dev/disk/by-id/scsi-SATA_SAMSUNG_HD160JJS08HJ10L507895-part3: Aliased to /dev/block/8:19 in device cache /dev/disk/by-id/scsi-SATA_SAMSUNG_HD160JJS08HJ10L507895-part5: Aliased to /dev/block/8:21 in device cache /dev/disk/by-id/scsi-SATA_SAMSUNG_HD160JJS08HJ10L507895-part6: Aliased to /dev/block/8:22 in device cache /dev/disk/by-id/scsi-SATA_SAMSUNG_HD160JJS08HJ10L526800: Aliased to /dev/block/8:0 in device cache /dev/disk/by-id/scsi-SATA_SAMSUNG_HD160JJS08HJ10L526800-part1: Aliased to /dev/block/8:1 in device cache /dev/disk/by-id/scsi-SATA_SAMSUNG_HD160JJS08HJ10L526800-part2: Aliased to /dev/block/8:2 in device cache /dev/disk/by-id/scsi-SATA_SAMSUNG_HD160JJS08HJ10L526800-part3: Aliased to /dev/block/8:3 in device cache /dev/disk/by-id/scsi-SATA_SAMSUNG_HD160JJS08HJ10L526800-part5: Aliased to /dev/block/8:5 in device cache /dev/disk/by-id/scsi-SATA_SAMSUNG_HD160JJS08HJ10L526800-part6: Aliased to /dev/block/8:6 in device cache /dev/disk/by-path/pci-0000:00:0f.0-scsi-0:0:0:0: Aliased to /dev/block/8:0 in device cache /dev/disk/by-path/pci-0000:00:0f.0-scsi-0:0:0:0-part1: Aliased to /dev/block/8:1 in device cache /dev/disk/by-path/pci-0000:00:0f.0-scsi-0:0:0:0-part2: Aliased to /dev/block/8:2 in device cache /dev/disk/by-path/pci-0000:00:0f.0-scsi-0:0:0:0-part3: Aliased to /dev/block/8:3 in device cache /dev/disk/by-path/pci-0000:00:0f.0-scsi-0:0:0:0-part5: Aliased to /dev/block/8:5 in device cache /dev/disk/by-path/pci-0000:00:0f.0-scsi-0:0:0:0-part6: Aliased to /dev/block/8:6 in device cache /dev/disk/by-path/pci-0000:00:0f.0-scsi-1:0:0:0: Aliased to /dev/block/8:16 in device cache /dev/disk/by-path/pci-0000:00:0f.0-scsi-1:0:0:0-part1: Aliased to /dev/block/8:17 in device cache /dev/disk/by-path/pci-0000:00:0f.0-scsi-1:0:0:0-part2: Aliased to /dev/block/8:18 in device cache /dev/disk/by-path/pci-0000:00:0f.0-scsi-1:0:0:0-part3: Aliased to /dev/block/8:19 in device cache /dev/disk/by-path/pci-0000:00:0f.0-scsi-1:0:0:0-part5: Aliased to /dev/block/8:21 in device cache /dev/disk/by-path/pci-0000:00:0f.0-scsi-1:0:0:0-part6: Aliased to /dev/block/8:22 in device cache /dev/disk/by-uuid/13c1262b-e06f-40ce-b088-ce410640a6dc: Aliased to /dev/block/253:3 in device cache /dev/disk/by-uuid/379f57b0-2e03-414c-808a-f76160617336: Aliased to /dev/block/253:2 in device cache /dev/disk/by-uuid/4fb2d6d3-bd51-48d3-95ee-8e404faf243d: Already in device cache /dev/disk/by-uuid/5c6728ec-82c1-49c0-93c5-f6dbd5c0d659: Aliased to /dev/block/8:5 in device cache /dev/disk/by-uuid/a13cdfcd-2191-4185-a727-ffefaf7a382e: Aliased to /dev/block/253:1 in device cache /dev/disk/by-uuid/e0d5893d-ff88-412f-b753-9e3e9af3242d: Aliased to /dev/block/8:21 in device cache /dev/disk/by-uuid/e79c9da6-8533-4e55-93ec-208876671edc: Aliased to /dev/block/253:0 in device cache /dev/disk/by-uuid/f3f176f5-12f7-4af8-952a-c6ac43a6e332: Already in device cache /dev/dm-0: Aliased to /dev/block/253:0 in device cache (preferred name) /dev/dm-1: Aliased to /dev/block/253:1 in device cache (preferred name) /dev/dm-2: Aliased to /dev/block/253:2 in device cache (preferred name) /dev/dm-3: Aliased to /dev/block/253:3 in device cache (preferred name) /dev/fd: Symbolic link to directory /dev/full: Not a block device /dev/hpet: Not a block device /dev/initctl: Not a block device /dev/input/by-path/platform-i8042-serio-0-event-kbd: Not a block device /dev/input/event0: Not a block device /dev/input/mice: Not a block device /dev/kmem: Not a block device /dev/kmsg: Not a block device /dev/log: Not a block device /dev/loop/0: Added to device cache /dev/MAKEDEV: Not a block device /dev/mapper/control: Not a block device /dev/mapper/systemlvm-home: Aliased to /dev/dm-2 in device cache /dev/mapper/systemlvm-tmp: Aliased to /dev/dm-3 in device cache /dev/mapper/systemlvm-usr: Aliased to /dev/dm-1 in device cache /dev/mapper/systemlvm-var: Aliased to /dev/dm-0 in device cache /dev/md0: Already in device cache /dev/md1: Already in device cache /dev/md2: Already in device cache /dev/mem: Not a block device /dev/net/tun: Not a block device /dev/network_latency: Not a block device /dev/network_throughput: Not a block device /dev/null: Not a block device /dev/port: Not a block device /dev/ppp: Not a block device /dev/psaux: Not a block device /dev/ptmx: Not a block device /dev/pts/0: Not a block device /dev/ram0: Aliased to /dev/block/1:0 in device cache (preferred name) /dev/ram1: Aliased to /dev/block/1:1 in device cache (preferred name) /dev/ram10: Aliased to /dev/block/1:10 in device cache (preferred name) /dev/ram11: Aliased to /dev/block/1:11 in device cache (preferred name) /dev/ram12: Aliased to /dev/block/1:12 in device cache (preferred name) /dev/ram13: Aliased to /dev/block/1:13 in device cache (preferred name) /dev/ram14: Aliased to /dev/block/1:14 in device cache (preferred name) /dev/ram15: Aliased to /dev/block/1:15 in device cache (preferred name) /dev/ram2: Aliased to /dev/block/1:2 in device cache (preferred name) /dev/ram3: Aliased to /dev/block/1:3 in device cache (preferred name) /dev/ram4: Aliased to /dev/block/1:4 in device cache (preferred name) /dev/ram5: Aliased to /dev/block/1:5 in device cache (preferred name) /dev/ram6: Aliased to /dev/block/1:6 in device cache (preferred name) /dev/ram7: Aliased to /dev/block/1:7 in device cache (preferred name) /dev/ram8: Aliased to /dev/block/1:8 in device cache (preferred name) /dev/ram9: Aliased to /dev/block/1:9 in device cache (preferred name) /dev/random: Not a block device /dev/root: Already in device cache /dev/rtc: Not a block device /dev/rtc0: Not a block device /dev/sda: Aliased to /dev/block/8:0 in device cache (preferred name) /dev/sda1: Aliased to /dev/block/8:1 in device cache (preferred name) /dev/sda2: Aliased to /dev/block/8:2 in device cache (preferred name) /dev/sda3: Aliased to /dev/block/8:3 in device cache (preferred name) /dev/sda5: Aliased to /dev/block/8:5 in device cache (preferred name) /dev/sda6: Aliased to /dev/block/8:6 in device cache (preferred name) /dev/sdb: Aliased to /dev/block/8:16 in device cache (preferred name) /dev/sdb1: Aliased to /dev/block/8:17 in device cache (preferred name) /dev/sdb2: Aliased to /dev/block/8:18 in device cache (preferred name) /dev/sdb3: Aliased to /dev/block/8:19 in device cache (preferred name) /dev/sdb5: Aliased to /dev/block/8:21 in device cache (preferred name) /dev/sdb6: Aliased to /dev/block/8:22 in device cache (preferred name) /dev/shm/network/ifstate: Not a block device /dev/snapshot: Not a block device /dev/sndstat: stat failed: Datei oder Verzeichnis nicht gefunden /dev/stderr: Not a block device /dev/stdin: Not a block device /dev/stdout: Not a block device /dev/systemlvm/home: Aliased to /dev/dm-2 in device cache /dev/systemlvm/tmp: Aliased to /dev/dm-3 in device cache /dev/systemlvm/usr: Aliased to /dev/dm-1 in device cache /dev/systemlvm/var: Aliased to /dev/dm-0 in device cache /dev/tty: Not a block device /dev/tty0: Not a block device [... many more "not a block device"] /dev/vcsa6: Not a block device /dev/xconsole: Not a block device /dev/zero: Not a block device Wiping internal VG cache lvmcache: initialised VG #orphans_lvm1 lvmcache: initialised VG #orphans_pool lvmcache: initialised VG #orphans_lvm2 Reading all physical volumes. This may take a while... Finding all volume groups /dev/ram0: Skipping (regex) /dev/loop/0: Skipping (sysfs) /dev/sda: Skipping (regex) Opened /dev/md0 RO /dev/md0: size is 192512 sectors Closed /dev/md0 /dev/md0: size is 192512 sectors Opened /dev/md0 RW O_DIRECT /dev/md0: block size is 1024 bytes Closed /dev/md0 Using /dev/md0 Opened /dev/md0 RW O_DIRECT /dev/md0: block size is 1024 bytes /dev/md0: No label detected Closed /dev/md0 /dev/dm-0: Skipping (regex) /dev/ram1: Skipping (regex) /dev/sda1: Skipping (regex) Opened /dev/md1 RO /dev/md1: size is 5863552 sectors Closed /dev/md1 /dev/md1: size is 5863552 sectors Opened /dev/md1 RW O_DIRECT /dev/md1: block size is 4096 bytes Closed /dev/md1 Using /dev/md1 Opened /dev/md1 RW O_DIRECT /dev/md1: block size is 4096 bytes /dev/md1: No label detected Closed /dev/md1 /dev/dm-1: Skipping (regex) /dev/ram2: Skipping (regex) /dev/sda2: Skipping (regex) Opened /dev/md2 RO /dev/md2: size is 303596160 sectors Closed /dev/md2 /dev/md2: size is 303596160 sectors Opened /dev/md2 RW O_DIRECT /dev/md2: block size is 4096 bytes Closed /dev/md2 Using /dev/md2 Opened /dev/md2 RW O_DIRECT /dev/md2: block size is 4096 bytes /dev/md2: lvm2 label detected lvmcache: /dev/md2: now in VG #orphans_lvm2 (#orphans_lvm2) /dev/md2: Found metadata at 39936 size 2632 (in area at 2048 size 194560) for systemlvm (rL8Oq2-dA7o-eRYe-u1or-JA7U-fnb1-kjOyvr) lvmcache: /dev/md2: now in VG systemlvm with 1 mdas lvmcache: /dev/md2: setting systemlvm VGID to rL8Oq2dA7oeRYeu1orJA7Ufnb1kjOyvr lvmcache: /dev/md2: VG systemlvm: Set creation host to rescue. Closed /dev/md2 /dev/dm-2: Skipping (regex) /dev/ram3: Skipping (regex) /dev/sda3: Skipping (regex) /dev/dm-3: Skipping (regex) /dev/ram4: Skipping (regex) /dev/ram5: Skipping (regex) /dev/sda5: Skipping (regex) /dev/ram6: Skipping (regex) /dev/sda6: Skipping (regex) /dev/ram7: Skipping (regex) /dev/ram8: Skipping (regex) /dev/ram9: Skipping (regex) /dev/ram10: Skipping (regex) /dev/ram11: Skipping (regex) /dev/ram12: Skipping (regex) /dev/ram13: Skipping (regex) /dev/ram14: Skipping (regex) /dev/ram15: Skipping (regex) /dev/sdb: Skipping (regex) /dev/sdb1: Skipping (regex) /dev/sdb2: Skipping (regex) /dev/sdb3: Skipping (regex) /dev/sdb5: Skipping (regex) /dev/sdb6: Skipping (regex) Locking /lib/init/rw/V_systemlvm RB Finding volume group "systemlvm" Opened /dev/md2 RW O_DIRECT /dev/md2: block size is 4096 bytes /dev/md2: lvm2 label detected lvmcache: /dev/md2: now in VG #orphans_lvm2 (#orphans_lvm2) with 1 mdas /dev/md2: Found metadata at 39936 size 2632 (in area at 2048 size 194560) for systemlvm (rL8Oq2-dA7o-eRYe-u1or-JA7U-fnb1-kjOyvr) lvmcache: /dev/md2: now in VG systemlvm with 1 mdas lvmcache: /dev/md2: setting systemlvm VGID to rL8Oq2dA7oeRYeu1orJA7Ufnb1kjOyvr lvmcache: /dev/md2: VG systemlvm: Set creation host to rescue. Using cached label for /dev/md2 Read systemlvm metadata (19) from /dev/md2 at 39936 size 2632 /dev/md2 0: 0 16: home(0:0) /dev/md2 1: 16 24: var(40:0) /dev/md2 2: 40 40: var(0:0) /dev/md2 3: 80 40: usr(0:0) /dev/md2 4: 120 40: var(80:0) /dev/md2 5: 160 8: tmp(0:0) /dev/md2 6: 168 16: var(64:0) /dev/md2 7: 184 80: var(120:0) /dev/md2 8: 264 64: home(16:0) /dev/md2 9: 328 128: var(200:0) /dev/md2 10: 456 32: home(80:0) /dev/md2 11: 488 440: var(328:0) /dev/md2 12: 928 24: home(112:0) /dev/md2 13: 952 206: NULL(0:0) Found volume group "systemlvm" using metadata type lvm2 Read volume group systemlvm from /etc/lvm/backup/systemlvm Unlocking /lib/init/rw/V_systemlvm Closed /dev/md2 Unlocking /lib/init/rw/P_global ~# vgdisplay --- Volume group --- VG Name systemlvm System ID Format lvm2 Metadata Areas 1 Metadata Sequence No 19 VG Access read/write VG Status resizable MAX LV 0 Cur LV 4 Open LV 4 Max PV 0 Cur PV 1 Act PV 1 VG Size 144,75 GB PE Size 128,00 MB Total PE 1158 Alloc PE / Size 952 / 119,00 GB Free PE / Size 206 / 25,75 GB VG UUID rL8Oq2-dA7o-eRYe-u1or-JA7U-fnb1-kjOyvr ~# pvdisplay --- Physical volume --- PV Name /dev/md2 VG Name systemlvm PV Size 144,77 GB / not usable 16,31 MB Allocatable yes PE Size (KByte) 131072 Total PE 1158 Free PE 206 Allocated PE 952 PV UUID ZSAzP5-iBvr-L7jy-wB8T-AiWz-0g3m-HLK66Y :~# lvdisplay --- Logical volume --- LV Name /dev/systemlvm/home VG Name systemlvm LV UUID YXrfdg-5OSY-DVkN-eiQe-Qksg-CI84-9Z2hx8 LV Write Access read/write LV Status available # open 2 LV Size 17,00 GB Current LE 136 Segments 4 Allocation inherit Read ahead sectors auto - currently set to 256 Block device 253:2 --- Logical volume --- LV Name /dev/systemlvm/var VG Name systemlvm LV UUID 25N7CR-ZpUM-zR18-NfS6-zeSe-AVnV-T98LuU LV Write Access read/write LV Status available # open 2 LV Size 96,00 GB Current LE 768 Segments 7 Allocation inherit Read ahead sectors auto - currently set to 256 Block device 253:0 --- Logical volume --- LV Name /dev/systemlvm/usr VG Name systemlvm LV UUID 3TpFXt-LjYG-Ewn7-9IdX-sSCZ-Pl8A-xmqbmQ LV Write Access read/write LV Status available # open 2 LV Size 5,00 GB Current LE 40 Segments 1 Allocation inherit Read ahead sectors auto - currently set to 256 Block device 253:1 --- Logical volume --- LV Name /dev/systemlvm/tmp VG Name systemlvm LV UUID c5MJ4K-olev-Mjt8-5PPB-rQuR-TkXb-x6NvTi LV Write Access read/write LV Status available # open 2 LV Size 1,00 GB Current LE 8 Segments 1 Allocation inherit Read ahead sectors auto - currently set to 256 Block device 253:3

    Read the article

  • MySQL is running VERY slow

    - by user1032531
    I have two servers: a VPS and a laptop. I recently re-built both of them, and MySQL is running about 20 times slower on the laptop. Both servers used to run CentOS 5.8 and I think MySQL 5.1, and the laptop used to do great so I do not think it is the hardware. For the VPS, my provider installed CentOS 6.4, and then I installed MySQL 5.1.69 using yum with the CentOS repo. For the laptop, I installed CentOS 6.4 basic server and then installed MySQL 5.1.69 using yum with the CentOS repo. my.cnf for both servers are identical, and I have shown below. For both servers, I've also included below the output from SHOW VARIABLES; as well as output from sysbench, file system information, and cpu information. I have tried adding skip-name-resolve, but it didn't help. The matrix below shows the SHOW VARIABLES output from both servers which is different. Again, MySQL was installed the same way, so I do not know why it is different, but it is and I think this might be why the laptop is executing MySQL so slowly. Why is the laptop running MySQL slowly, and how do I fix it? Differences between SHOW VARIABLES on both servers +---------------------------+-----------------------+-------------------------+ | Variable | Value-VPS | Value-Laptop | +---------------------------+-----------------------+-------------------------+ | hostname | vps.site1.com | laptop.site2.com | | max_binlog_cache_size | 4294963200 | 18446744073709500000 | | max_seeks_for_key | 4294967295 | 18446744073709500000 | | max_write_lock_count | 4294967295 | 18446744073709500000 | | myisam_max_sort_file_size | 2146435072 | 9223372036853720000 | | myisam_mmap_size | 4294967295 | 18446744073709500000 | | plugin_dir | /usr/lib/mysql/plugin | /usr/lib64/mysql/plugin | | pseudo_thread_id | 7568 | 2 | | system_time_zone | EST | PDT | | thread_stack | 196608 | 262144 | | timestamp | 1372252112 | 1372252046 | | version_compile_machine | i386 | x86_64 | +---------------------------+-----------------------+-------------------------+ my.cnf for both servers [root@server1 ~]# cat /etc/my.cnf [mysqld] datadir=/var/lib/mysql socket=/var/lib/mysql/mysql.sock user=mysql # Disabling symbolic-links is recommended to prevent assorted security risks symbolic-links=0 [mysqld_safe] log-error=/var/log/mysqld.log pid-file=/var/run/mysqld/mysqld.pid innodb_strict_mode=on sql_mode=TRADITIONAL # sql_mode=STRICT_TRANS_TABLES,NO_ZERO_DATE,NO_ZERO_IN_DATE character-set-server=utf8 collation-server=utf8_general_ci log=/var/log/mysqld_all.log [root@server1 ~]# VPS SHOW VARIABLES Info Same as Laptop shown below but changes per above matrix (removed to allow me to be under the 30000 characters as required by ServerFault) Laptop SHOW VARIABLES Info auto_increment_increment 1 auto_increment_offset 1 autocommit ON automatic_sp_privileges ON back_log 50 basedir /usr/ big_tables OFF binlog_cache_size 32768 binlog_direct_non_transactional_updates OFF binlog_format STATEMENT bulk_insert_buffer_size 8388608 character_set_client utf8 character_set_connection utf8 character_set_database latin1 character_set_filesystem binary character_set_results utf8 character_set_server latin1 character_set_system utf8 character_sets_dir /usr/share/mysql/charsets/ collation_connection utf8_general_ci collation_database latin1_swedish_ci collation_server latin1_swedish_ci completion_type 0 concurrent_insert 1 connect_timeout 10 datadir /var/lib/mysql/ date_format %Y-%m-%d datetime_format %Y-%m-%d %H:%i:%s default_week_format 0 delay_key_write ON delayed_insert_limit 100 delayed_insert_timeout 300 delayed_queue_size 1000 div_precision_increment 4 engine_condition_pushdown ON error_count 0 event_scheduler OFF expire_logs_days 0 flush OFF flush_time 0 foreign_key_checks ON ft_boolean_syntax + -><()~*:""&| ft_max_word_len 84 ft_min_word_len 4 ft_query_expansion_limit 20 ft_stopword_file (built-in) general_log OFF general_log_file /var/run/mysqld/mysqld.log group_concat_max_len 1024 have_community_features YES have_compress YES have_crypt YES have_csv YES have_dynamic_loading YES have_geometry YES have_innodb YES have_ndbcluster NO have_openssl DISABLED have_partitioning YES have_query_cache YES have_rtree_keys YES have_ssl DISABLED have_symlink DISABLED hostname server1.site2.com identity 0 ignore_builtin_innodb OFF init_connect init_file init_slave innodb_adaptive_hash_index ON innodb_additional_mem_pool_size 1048576 innodb_autoextend_increment 8 innodb_autoinc_lock_mode 1 innodb_buffer_pool_size 8388608 innodb_checksums ON innodb_commit_concurrency 0 innodb_concurrency_tickets 500 innodb_data_file_path ibdata1:10M:autoextend innodb_data_home_dir innodb_doublewrite ON innodb_fast_shutdown 1 innodb_file_io_threads 4 innodb_file_per_table OFF innodb_flush_log_at_trx_commit 1 innodb_flush_method innodb_force_recovery 0 innodb_lock_wait_timeout 50 innodb_locks_unsafe_for_binlog OFF innodb_log_buffer_size 1048576 innodb_log_file_size 5242880 innodb_log_files_in_group 2 innodb_log_group_home_dir ./ innodb_max_dirty_pages_pct 90 innodb_max_purge_lag 0 innodb_mirrored_log_groups 1 innodb_open_files 300 innodb_rollback_on_timeout OFF innodb_stats_method nulls_equal innodb_stats_on_metadata ON innodb_support_xa ON innodb_sync_spin_loops 20 innodb_table_locks ON innodb_thread_concurrency 8 innodb_thread_sleep_delay 10000 innodb_use_legacy_cardinality_algorithm ON insert_id 0 interactive_timeout 28800 join_buffer_size 131072 keep_files_on_create OFF key_buffer_size 8384512 key_cache_age_threshold 300 key_cache_block_size 1024 key_cache_division_limit 100 language /usr/share/mysql/english/ large_files_support ON large_page_size 0 large_pages OFF last_insert_id 0 lc_time_names en_US license GPL local_infile ON locked_in_memory OFF log OFF log_bin OFF log_bin_trust_function_creators OFF log_bin_trust_routine_creators OFF log_error /var/log/mysqld.log log_output FILE log_queries_not_using_indexes OFF log_slave_updates OFF log_slow_queries OFF log_warnings 1 long_query_time 10.000000 low_priority_updates OFF lower_case_file_system OFF lower_case_table_names 0 max_allowed_packet 1048576 max_binlog_cache_size 18446744073709547520 max_binlog_size 1073741824 max_connect_errors 10 max_connections 151 max_delayed_threads 20 max_error_count 64 max_heap_table_size 16777216 max_insert_delayed_threads 20 max_join_size 18446744073709551615 max_length_for_sort_data 1024 max_long_data_size 1048576 max_prepared_stmt_count 16382 max_relay_log_size 0 max_seeks_for_key 18446744073709551615 max_sort_length 1024 max_sp_recursion_depth 0 max_tmp_tables 32 max_user_connections 0 max_write_lock_count 18446744073709551615 min_examined_row_limit 0 multi_range_count 256 myisam_data_pointer_size 6 myisam_max_sort_file_size 9223372036853727232 myisam_mmap_size 18446744073709551615 myisam_recover_options OFF myisam_repair_threads 1 myisam_sort_buffer_size 8388608 myisam_stats_method nulls_unequal myisam_use_mmap OFF net_buffer_length 16384 net_read_timeout 30 net_retry_count 10 net_write_timeout 60 new OFF old OFF old_alter_table OFF old_passwords OFF open_files_limit 1024 optimizer_prune_level 1 optimizer_search_depth 62 optimizer_switch index_merge=on,index_merge_union=on,index_merge_sort_union=on,index_merge_intersection=on pid_file /var/run/mysqld/mysqld.pid plugin_dir /usr/lib64/mysql/plugin port 3306 preload_buffer_size 32768 profiling OFF profiling_history_size 15 protocol_version 10 pseudo_thread_id 3 query_alloc_block_size 8192 query_cache_limit 1048576 query_cache_min_res_unit 4096 query_cache_size 0 query_cache_type ON query_cache_wlock_invalidate OFF query_prealloc_size 8192 rand_seed1 rand_seed2 range_alloc_block_size 4096 read_buffer_size 131072 read_only OFF read_rnd_buffer_size 262144 relay_log relay_log_index relay_log_info_file relay-log.info relay_log_purge ON relay_log_space_limit 0 report_host report_password report_port 3306 report_user rpl_recovery_rank 0 secure_auth OFF secure_file_priv server_id 0 skip_external_locking ON skip_name_resolve OFF skip_networking OFF skip_show_database OFF slave_compressed_protocol OFF slave_exec_mode STRICT slave_load_tmpdir /tmp slave_max_allowed_packet 1073741824 slave_net_timeout 3600 slave_skip_errors OFF slave_transaction_retries 10 slow_launch_time 2 slow_query_log OFF slow_query_log_file /var/run/mysqld/mysqld-slow.log socket /var/lib/mysql/mysql.sock sort_buffer_size 2097144 sql_auto_is_null ON sql_big_selects ON sql_big_tables OFF sql_buffer_result OFF sql_log_bin ON sql_log_off OFF sql_log_update ON sql_low_priority_updates OFF sql_max_join_size 18446744073709551615 sql_mode sql_notes ON sql_quote_show_create ON sql_safe_updates OFF sql_select_limit 18446744073709551615 sql_slave_skip_counter sql_warnings OFF ssl_ca ssl_capath ssl_cert ssl_cipher ssl_key storage_engine MyISAM sync_binlog 0 sync_frm ON system_time_zone PDT table_definition_cache 256 table_lock_wait_timeout 50 table_open_cache 64 table_type MyISAM thread_cache_size 0 thread_handling one-thread-per-connection thread_stack 262144 time_format %H:%i:%s time_zone SYSTEM timed_mutexes OFF timestamp 1372254399 tmp_table_size 16777216 tmpdir /tmp transaction_alloc_block_size 8192 transaction_prealloc_size 4096 tx_isolation REPEATABLE-READ unique_checks ON updatable_views_with_limit YES version 5.1.69 version_comment Source distribution version_compile_machine x86_64 version_compile_os redhat-linux-gnu wait_timeout 28800 warning_count 0 VPS Sysbench Info [root@vps ~]# cat sysbench.txt sysbench 0.4.12: multi-threaded system evaluation benchmark Running the test with following options: Number of threads: 8 Doing OLTP test. Running mixed OLTP test Doing read-only test Using Special distribution (12 iterations, 1 pct of values are returned in 75 pct cases) Using "BEGIN" for starting transactions Using auto_inc on the id column Threads started! Time limit exceeded, exiting... (last message repeated 7 times) Done. OLTP test statistics: queries performed: read: 1449966 write: 0 other: 207138 total: 1657104 transactions: 103569 (1726.01 per sec.) deadlocks: 0 (0.00 per sec.) read/write requests: 1449966 (24164.08 per sec.) other operations: 207138 (3452.01 per sec.) Test execution summary: total time: 60.0050s total number of events: 103569 total time taken by event execution: 479.1544 per-request statistics: min: 1.98ms avg: 4.63ms max: 330.73ms approx. 95 percentile: 8.26ms Threads fairness: events (avg/stddev): 12946.1250/381.09 execution time (avg/stddev): 59.8943/0.00 [root@vps ~]# Laptop Sysbench Info [root@server1 ~]# cat sysbench.txt sysbench 0.4.12: multi-threaded system evaluation benchmark Running the test with following options: Number of threads: 8 Doing OLTP test. Running mixed OLTP test Doing read-only test Using Special distribution (12 iterations, 1 pct of values are returned in 75 pct cases) Using "BEGIN" for starting transactions Using auto_inc on the id column Threads started! Time limit exceeded, exiting... (last message repeated 7 times) Done. OLTP test statistics: queries performed: read: 634718 write: 0 other: 90674 total: 725392 transactions: 45337 (755.56 per sec.) deadlocks: 0 (0.00 per sec.) read/write requests: 634718 (10577.78 per sec.) other operations: 90674 (1511.11 per sec.) Test execution summary: total time: 60.0048s total number of events: 45337 total time taken by event execution: 479.4912 per-request statistics: min: 2.04ms avg: 10.58ms max: 85.56ms approx. 95 percentile: 19.70ms Threads fairness: events (avg/stddev): 5667.1250/42.18 execution time (avg/stddev): 59.9364/0.00 [root@server1 ~]# VPS File Info [root@vps ~]# df -T Filesystem Type 1K-blocks Used Available Use% Mounted on /dev/simfs simfs 20971520 16187440 4784080 78% / none tmpfs 6224432 4 6224428 1% /dev none tmpfs 6224432 0 6224432 0% /dev/shm [root@vps ~]# Laptop File Info [root@server1 ~]# df -T Filesystem Type 1K-blocks Used Available Use% Mounted on /dev/mapper/vg_server1-lv_root ext4 72383800 4243964 64462860 7% / tmpfs tmpfs 956352 0 956352 0% /dev/shm /dev/sdb1 ext4 495844 60948 409296 13% /boot [root@server1 ~]# VPS CPU Info Removed to stay under the 30000 character limit required by ServerFault Laptop CPU Info [root@server1 ~]# cat /proc/cpuinfo processor : 0 vendor_id : GenuineIntel cpu family : 6 model : 15 model name : Intel(R) Core(TM)2 Duo CPU T7100 @ 1.80GHz stepping : 13 cpu MHz : 800.000 cache size : 2048 KB physical id : 0 siblings : 2 core id : 0 cpu cores : 2 apicid : 0 initial apicid : 0 fpu : yes fpu_exception : yes cpuid level : 10 wp : yes flags : fpu vme de pse tsc msr pae mce cx8 apic sep mtrr pge mca cmov pat pse36 clflush dts acpi mmx fxsr sse sse2 ss ht tm pbe syscall nx lm constant_tsc arch_perfmon pebs bts rep_good aperfmperf pni dtes64 monitor ds_cpl vmx est tm2 ssse3 cx16 xtpr pdcm lahf_lm ida dts tpr_shadow vnmi flexpriority bogomips : 3591.39 clflush size : 64 cache_alignment : 64 address sizes : 36 bits physical, 48 bits virtual power management: processor : 1 vendor_id : GenuineIntel cpu family : 6 model : 15 model name : Intel(R) Core(TM)2 Duo CPU T7100 @ 1.80GHz stepping : 13 cpu MHz : 800.000 cache size : 2048 KB physical id : 0 siblings : 2 core id : 1 cpu cores : 2 apicid : 1 initial apicid : 1 fpu : yes fpu_exception : yes cpuid level : 10 wp : yes flags : fpu vme de pse tsc msr pae mce cx8 apic sep mtrr pge mca cmov pat pse36 clflush dts acpi mmx fxsr sse sse2 ss ht tm pbe syscall nx lm constant_tsc arch_perfmon pebs bts rep_good aperfmperf pni dtes64 monitor ds_cpl vmx est tm2 ssse3 cx16 xtpr pdcm lahf_lm ida dts tpr_shadow vnmi flexpriority bogomips : 3591.39 clflush size : 64 cache_alignment : 64 address sizes : 36 bits physical, 48 bits virtual power management: [root@server1 ~]# EDIT New Info requested by shakalandy [root@localhost ~]# cat /proc/meminfo MemTotal: 2044804 kB MemFree: 761464 kB Buffers: 68868 kB Cached: 369708 kB SwapCached: 0 kB Active: 881080 kB Inactive: 246016 kB Active(anon): 688312 kB Inactive(anon): 4416 kB Active(file): 192768 kB Inactive(file): 241600 kB Unevictable: 0 kB Mlocked: 0 kB SwapTotal: 4095992 kB SwapFree: 4095992 kB Dirty: 0 kB Writeback: 0 kB AnonPages: 688428 kB Mapped: 65156 kB Shmem: 4216 kB Slab: 92428 kB SReclaimable: 31260 kB SUnreclaim: 61168 kB KernelStack: 2392 kB PageTables: 28356 kB NFS_Unstable: 0 kB Bounce: 0 kB WritebackTmp: 0 kB CommitLimit: 5118392 kB Committed_AS: 1530212 kB VmallocTotal: 34359738367 kB VmallocUsed: 343604 kB VmallocChunk: 34359372920 kB HardwareCorrupted: 0 kB AnonHugePages: 520192 kB HugePages_Total: 0 HugePages_Free: 0 HugePages_Rsvd: 0 HugePages_Surp: 0 Hugepagesize: 2048 kB DirectMap4k: 8556 kB DirectMap2M: 2078720 kB [root@localhost ~]# ps aux | grep mysql root 2227 0.0 0.0 108332 1504 ? S 07:36 0:00 /bin/sh /usr/bin/mysqld_safe --datadir=/var/lib/mysql --pid-file=/var/lib/mysql/localhost.badobe.com.pid mysql 2319 0.1 24.5 1470068 501360 ? Sl 07:36 0:57 /usr/sbin/mysqld --basedir=/usr --datadir=/var/lib/mysql --plugin-dir=/usr/lib64/mysql/plugin --user=mysql --log-error=/var/lib/mysql/localhost.badobe.com.err --pid-file=/var/lib/mysql/localhost.badobe.com.pid root 3579 0.0 0.1 201840 3028 pts/0 S+ 07:40 0:00 mysql -u root -p root 13887 0.0 0.1 201840 3036 pts/3 S+ 18:08 0:00 mysql -uroot -px xxxxxxxxxx root 14449 0.0 0.0 103248 840 pts/2 S+ 18:16 0:00 grep mysql [root@localhost ~]# ps aux | grep mysql root 2227 0.0 0.0 108332 1504 ? S 07:36 0:00 /bin/sh /usr/bin/mysqld_safe --datadir=/var/lib/mysql --pid-file=/var/lib/mysql/localhost.badobe.com.pid mysql 2319 0.1 24.5 1470068 501356 ? Sl 07:36 0:57 /usr/sbin/mysqld --basedir=/usr --datadir=/var/lib/mysql --plugin-dir=/usr/lib64/mysql/plugin --user=mysql --log-error=/var/lib/mysql/localhost.badobe.com.err --pid-file=/var/lib/mysql/localhost.badobe.com.pid root 3579 0.0 0.1 201840 3028 pts/0 S+ 07:40 0:00 mysql -u root -p root 13887 0.0 0.1 201840 3048 pts/3 S+ 18:08 0:00 mysql -uroot -px xxxxxxxxxx root 14470 0.0 0.0 103248 840 pts/2 S+ 18:16 0:00 grep mysql [root@localhost ~]# vmstat 1 procs -----------memory---------- ---swap-- -----io---- --system-- -----cpu----- r b swpd free buff cache si so bi bo in cs us sy id wa st 0 0 0 742172 76376 371064 0 0 6 6 78 202 2 1 97 1 0 0 0 0 742164 76380 371060 0 0 0 16 191 467 2 1 93 5 0 0 0 0 742164 76380 371064 0 0 0 0 148 388 2 1 98 0 0 0 0 0 742164 76380 371064 0 0 0 0 159 418 2 1 98 0 0 0 0 0 742164 76380 371064 0 0 0 0 145 380 2 1 98 0 0 0 0 0 742164 76380 371064 0 0 0 0 166 429 2 1 97 0 0 1 0 0 742164 76380 371064 0 0 0 0 148 373 2 1 98 0 0 0 0 0 742164 76380 371064 0 0 0 0 149 382 2 1 98 0 0 0 0 0 742164 76380 371064 0 0 0 0 168 408 2 0 97 0 0 0 0 0 742164 76380 371064 0 0 0 0 165 394 2 1 98 0 0 0 0 0 742164 76380 371064 0 0 0 0 159 354 2 1 98 0 0 0 0 0 742164 76388 371060 0 0 0 16 180 447 2 0 91 6 0 0 0 0 742164 76388 371064 0 0 0 0 143 344 2 1 98 0 0 0 1 0 742784 76416 370044 0 0 28 580 360 678 3 1 74 23 0 1 0 0 744768 76496 367772 0 0 40 1036 437 865 3 1 53 43 0 0 1 0 747248 76596 365412 0 0 48 1224 561 923 3 2 53 43 0 0 1 0 749232 76696 363092 0 0 32 1132 512 883 3 2 52 44 0 0 1 0 751340 76772 361020 0 0 32 1008 472 872 2 1 52 45 0 0 1 0 753448 76840 358540 0 0 36 1088 512 860 2 1 51 46 0 0 1 0 755060 76936 357636 0 0 28 1012 481 922 2 2 52 45 0 0 1 0 755060 77064 357988 0 0 12 896 444 902 2 1 53 45 0 0 1 0 754688 77148 358448 0 0 16 1096 506 1007 1 1 56 42 0 0 2 0 754192 77268 358932 0 0 12 1060 481 957 1 2 53 44 0 0 1 0 753696 77380 359392 0 0 12 1052 512 1025 2 1 55 42 0 0 1 0 751028 77480 359828 0 0 8 984 423 909 2 2 52 45 0 0 1 0 750524 77620 360200 0 0 8 788 367 869 1 2 54 44 0 0 1 0 749904 77700 360664 0 0 8 928 439 924 2 2 55 43 0 0 1 0 749408 77796 361084 0 0 12 976 468 967 1 1 56 43 0 0 1 0 748788 77896 361464 0 0 12 992 453 944 1 2 54 43 0 1 1 0 748416 77992 361996 0 0 12 784 392 868 2 1 52 46 0 0 1 0 747920 78092 362336 0 0 4 896 382 874 1 1 52 46 0 0 1 0 745252 78172 362780 0 0 12 1040 444 923 1 1 56 42 0 0 1 0 744764 78288 363220 0 0 8 1024 448 934 2 1 55 43 0 0 1 0 744144 78408 363668 0 0 8 1000 461 982 2 1 53 44 0 0 1 0 743648 78488 364148 0 0 8 872 443 888 2 1 54 43 0 0 1 0 743152 78548 364468 0 0 16 1020 511 995 2 1 55 43 0 0 1 0 742656 78632 365024 0 0 12 928 431 913 1 2 53 44 0 0 1 0 742160 78728 365468 0 0 12 996 470 955 2 2 54 44 0 1 1 0 739492 78840 365896 0 0 8 988 447 939 1 2 52 46 0 0 1 0 738872 78996 366352 0 0 12 972 442 928 1 1 55 44 0 1 1 0 738244 79148 366812 0 0 8 948 549 1126 2 2 54 43 0 0 1 0 737624 79312 367188 0 0 12 996 456 953 2 2 54 43 0 0 1 0 736880 79456 367660 0 0 12 960 444 918 1 1 53 46 0 0 1 0 736260 79584 368124 0 0 8 884 414 921 1 1 54 44 0 0 1 0 735648 79716 368488 0 0 12 976 450 955 2 1 56 41 0 0 1 0 733104 79840 368988 0 0 12 932 453 918 1 2 55 43 0 0 1 0 732608 79996 369356 0 0 16 916 444 889 1 2 54 43 0 1 1 0 731476 80128 369800 0 0 16 852 514 978 2 2 54 43 0 0 1 0 731244 80252 370200 0 0 8 904 398 870 2 1 55 43 0 1 1 0 730624 80384 370612 0 0 12 1032 447 977 1 2 57 41 0 0 1 0 730004 80524 371096 0 0 12 984 469 941 2 2 52 45 0 0 1 0 729508 80636 371544 0 0 12 928 438 922 2 1 52 46 0 0 1 0 728888 80756 371948 0 0 16 972 439 943 2 1 55 43 0 0 1 0 726468 80900 372272 0 0 8 960 545 1024 2 1 54 43 0 1 1 0 726344 81024 372272 0 0 8 464 490 1057 1 2 53 44 0 0 1 0 726096 81148 372276 0 0 4 328 441 1063 2 1 53 45 0 1 1 0 726096 81256 372292 0 0 0 296 387 975 1 1 53 45 0 0 1 0 725848 81380 372284 0 0 4 332 425 1034 2 1 54 44 0 1 1 0 725848 81496 372300 0 0 4 308 386 992 2 1 54 43 0 0 1 0 725600 81616 372296 0 0 4 328 404 1060 1 1 54 44 0 procs -----------memory---------- ---swap-- -----io---- --system-- -----cpu----- r b swpd free buff cache si so bi bo in cs us sy id wa st 0 1 0 725600 81732 372296 0 0 4 328 439 1011 1 1 53 44 0 0 1 0 725476 81848 372308 0 0 0 316 441 1023 2 2 52 46 0 1 1 0 725352 81972 372300 0 0 4 344 451 1021 1 1 55 43 0 2 1 0 725228 82088 372320 0 0 0 328 427 1058 1 1 54 44 0 1 1 0 724980 82220 372300 0 0 4 336 419 999 2 1 54 44 0 1 1 0 724980 82328 372320 0 0 4 320 430 1019 1 1 54 44 0 1 1 0 724732 82436 372328 0 0 0 388 363 942 2 1 54 44 0 1 1 0 724608 82560 372312 0 0 4 308 419 993 1 2 54 44 0 1 0 0 724360 82684 372320 0 0 0 304 421 1028 2 1 55 42 0 1 0 0 724360 82684 372388 0 0 0 0 158 416 2 1 98 0 0 1 1 0 724236 82720 372360 0 0 0 6464 243 855 3 2 84 12 0 1 0 0 724112 82748 372360 0 0 0 5356 266 895 3 1 84 12 0 2 1 0 724112 82764 372380 0 0 0 3052 221 511 2 2 93 4 0 1 0 0 724112 82796 372372 0 0 0 4548 325 1067 2 2 81 16 0 1 0 0 724112 82816 372368 0 0 0 3240 259 829 3 1 90 6 0 1 0 0 724112 82836 372380 0 0 0 3260 309 822 3 2 88 8 0 1 1 0 724112 82876 372364 0 0 0 4680 326 978 3 1 77 19 0 1 0 0 724112 82884 372380 0 0 0 512 207 508 2 1 95 2 0 1 0 0 724112 82884 372388 0 0 0 0 138 361 2 1 98 0 0 1 0 0 724112 82884 372388 0 0 0 0 158 397 2 1 98 0 0 1 0 0 724112 82884 372388 0 0 0 0 146 395 2 1 98 0 0 2 0 0 724112 82884 372388 0 0 0 0 160 395 2 1 98 0 0 1 0 0 724112 82884 372388 0 0 0 0 163 382 1 1 98 0 0 1 0 0 724112 82884 372388 0 0 0 0 176 422 2 1 98 0 0 1 0 0 724112 82884 372388 0 0 0 0 134 351 2 1 98 0 0 0 0 0 724112 82884 372388 0 0 0 0 190 429 2 1 97 0 0 0 0 0 724104 82884 372392 0 0 0 0 139 358 2 1 98 0 0 0 0 0 724848 82884 372392 0 0 0 4 211 432 2 1 97 0 0 1 0 0 724980 82884 372392 0 0 0 0 166 370 2 1 98 0 0 0 0 0 724980 82884 372392 0 0 0 0 164 397 2 1 98 0 0 ^C [root@localhost ~]#

    Read the article

  • MySQL is running VERY slow on CentOS 6x (not 5x)

    - by user1032531
    I have two servers: a VPS and a laptop. I recently re-built both of them, and MySQL is running about 20 times slower on the laptop. Both servers used to run CentOS 5.8 and I think MySQL 5.1, and the laptop used to do great so I do not think it is the hardware. For the VPS, my provider installed CentOS 6.4, and then I installed MySQL 5.1.69 using yum with the CentOS repo. For the laptop, I installed CentOS 6.4 basic server and then installed MySQL 5.1.69 using yum with the CentOS repo. my.cnf for both servers are identical, and I have shown below. For both servers, I've also included below the output from SHOW VARIABLES; as well as output from sysbench, file system information, and cpu information. I have tried adding skip-name-resolve, but it didn't help. The matrix below shows the SHOW VARIABLES output from both servers which is different. Again, MySQL was installed the same way, so I do not know why it is different, but it is and I think this might be why the laptop is executing MySQL so slowly. Why is the laptop running MySQL slowly, and how do I fix it? Differences between SHOW VARIABLES on both servers +---------------------------+-----------------------+-------------------------+ | Variable | Value-VPS | Value-Laptop | +---------------------------+-----------------------+-------------------------+ | hostname | vps.site1.com | laptop.site2.com | | max_binlog_cache_size | 4294963200 | 18446744073709500000 | | max_seeks_for_key | 4294967295 | 18446744073709500000 | | max_write_lock_count | 4294967295 | 18446744073709500000 | | myisam_max_sort_file_size | 2146435072 | 9223372036853720000 | | myisam_mmap_size | 4294967295 | 18446744073709500000 | | plugin_dir | /usr/lib/mysql/plugin | /usr/lib64/mysql/plugin | | pseudo_thread_id | 7568 | 2 | | system_time_zone | EST | PDT | | thread_stack | 196608 | 262144 | | timestamp | 1372252112 | 1372252046 | | version_compile_machine | i386 | x86_64 | +---------------------------+-----------------------+-------------------------+ my.cnf for both servers [root@server1 ~]# cat /etc/my.cnf [mysqld] datadir=/var/lib/mysql socket=/var/lib/mysql/mysql.sock user=mysql # Disabling symbolic-links is recommended to prevent assorted security risks symbolic-links=0 [mysqld_safe] log-error=/var/log/mysqld.log pid-file=/var/run/mysqld/mysqld.pid innodb_strict_mode=on sql_mode=TRADITIONAL # sql_mode=STRICT_TRANS_TABLES,NO_ZERO_DATE,NO_ZERO_IN_DATE character-set-server=utf8 collation-server=utf8_general_ci log=/var/log/mysqld_all.log [root@server1 ~]# VPS SHOW VARIABLES Info Same as Laptop shown below but changes per above matrix (removed to allow me to be under the 30000 characters as required by ServerFault) Laptop SHOW VARIABLES Info auto_increment_increment 1 auto_increment_offset 1 autocommit ON automatic_sp_privileges ON back_log 50 basedir /usr/ big_tables OFF binlog_cache_size 32768 binlog_direct_non_transactional_updates OFF binlog_format STATEMENT bulk_insert_buffer_size 8388608 character_set_client utf8 character_set_connection utf8 character_set_database latin1 character_set_filesystem binary character_set_results utf8 character_set_server latin1 character_set_system utf8 character_sets_dir /usr/share/mysql/charsets/ collation_connection utf8_general_ci collation_database latin1_swedish_ci collation_server latin1_swedish_ci completion_type 0 concurrent_insert 1 connect_timeout 10 datadir /var/lib/mysql/ date_format %Y-%m-%d datetime_format %Y-%m-%d %H:%i:%s default_week_format 0 delay_key_write ON delayed_insert_limit 100 delayed_insert_timeout 300 delayed_queue_size 1000 div_precision_increment 4 engine_condition_pushdown ON error_count 0 event_scheduler OFF expire_logs_days 0 flush OFF flush_time 0 foreign_key_checks ON ft_boolean_syntax + -><()~*:""&| ft_max_word_len 84 ft_min_word_len 4 ft_query_expansion_limit 20 ft_stopword_file (built-in) general_log OFF general_log_file /var/run/mysqld/mysqld.log group_concat_max_len 1024 have_community_features YES have_compress YES have_crypt YES have_csv YES have_dynamic_loading YES have_geometry YES have_innodb YES have_ndbcluster NO have_openssl DISABLED have_partitioning YES have_query_cache YES have_rtree_keys YES have_ssl DISABLED have_symlink DISABLED hostname server1.site2.com identity 0 ignore_builtin_innodb OFF init_connect init_file init_slave innodb_adaptive_hash_index ON innodb_additional_mem_pool_size 1048576 innodb_autoextend_increment 8 innodb_autoinc_lock_mode 1 innodb_buffer_pool_size 8388608 innodb_checksums ON innodb_commit_concurrency 0 innodb_concurrency_tickets 500 innodb_data_file_path ibdata1:10M:autoextend innodb_data_home_dir innodb_doublewrite ON innodb_fast_shutdown 1 innodb_file_io_threads 4 innodb_file_per_table OFF innodb_flush_log_at_trx_commit 1 innodb_flush_method innodb_force_recovery 0 innodb_lock_wait_timeout 50 innodb_locks_unsafe_for_binlog OFF innodb_log_buffer_size 1048576 innodb_log_file_size 5242880 innodb_log_files_in_group 2 innodb_log_group_home_dir ./ innodb_max_dirty_pages_pct 90 innodb_max_purge_lag 0 innodb_mirrored_log_groups 1 innodb_open_files 300 innodb_rollback_on_timeout OFF innodb_stats_method nulls_equal innodb_stats_on_metadata ON innodb_support_xa ON innodb_sync_spin_loops 20 innodb_table_locks ON innodb_thread_concurrency 8 innodb_thread_sleep_delay 10000 innodb_use_legacy_cardinality_algorithm ON insert_id 0 interactive_timeout 28800 join_buffer_size 131072 keep_files_on_create OFF key_buffer_size 8384512 key_cache_age_threshold 300 key_cache_block_size 1024 key_cache_division_limit 100 language /usr/share/mysql/english/ large_files_support ON large_page_size 0 large_pages OFF last_insert_id 0 lc_time_names en_US license GPL local_infile ON locked_in_memory OFF log OFF log_bin OFF log_bin_trust_function_creators OFF log_bin_trust_routine_creators OFF log_error /var/log/mysqld.log log_output FILE log_queries_not_using_indexes OFF log_slave_updates OFF log_slow_queries OFF log_warnings 1 long_query_time 10.000000 low_priority_updates OFF lower_case_file_system OFF lower_case_table_names 0 max_allowed_packet 1048576 max_binlog_cache_size 18446744073709547520 max_binlog_size 1073741824 max_connect_errors 10 max_connections 151 max_delayed_threads 20 max_error_count 64 max_heap_table_size 16777216 max_insert_delayed_threads 20 max_join_size 18446744073709551615 max_length_for_sort_data 1024 max_long_data_size 1048576 max_prepared_stmt_count 16382 max_relay_log_size 0 max_seeks_for_key 18446744073709551615 max_sort_length 1024 max_sp_recursion_depth 0 max_tmp_tables 32 max_user_connections 0 max_write_lock_count 18446744073709551615 min_examined_row_limit 0 multi_range_count 256 myisam_data_pointer_size 6 myisam_max_sort_file_size 9223372036853727232 myisam_mmap_size 18446744073709551615 myisam_recover_options OFF myisam_repair_threads 1 myisam_sort_buffer_size 8388608 myisam_stats_method nulls_unequal myisam_use_mmap OFF net_buffer_length 16384 net_read_timeout 30 net_retry_count 10 net_write_timeout 60 new OFF old OFF old_alter_table OFF old_passwords OFF open_files_limit 1024 optimizer_prune_level 1 optimizer_search_depth 62 optimizer_switch index_merge=on,index_merge_union=on,index_merge_sort_union=on,index_merge_intersection=on pid_file /var/run/mysqld/mysqld.pid plugin_dir /usr/lib64/mysql/plugin port 3306 preload_buffer_size 32768 profiling OFF profiling_history_size 15 protocol_version 10 pseudo_thread_id 3 query_alloc_block_size 8192 query_cache_limit 1048576 query_cache_min_res_unit 4096 query_cache_size 0 query_cache_type ON query_cache_wlock_invalidate OFF query_prealloc_size 8192 rand_seed1 rand_seed2 range_alloc_block_size 4096 read_buffer_size 131072 read_only OFF read_rnd_buffer_size 262144 relay_log relay_log_index relay_log_info_file relay-log.info relay_log_purge ON relay_log_space_limit 0 report_host report_password report_port 3306 report_user rpl_recovery_rank 0 secure_auth OFF secure_file_priv server_id 0 skip_external_locking ON skip_name_resolve OFF skip_networking OFF skip_show_database OFF slave_compressed_protocol OFF slave_exec_mode STRICT slave_load_tmpdir /tmp slave_max_allowed_packet 1073741824 slave_net_timeout 3600 slave_skip_errors OFF slave_transaction_retries 10 slow_launch_time 2 slow_query_log OFF slow_query_log_file /var/run/mysqld/mysqld-slow.log socket /var/lib/mysql/mysql.sock sort_buffer_size 2097144 sql_auto_is_null ON sql_big_selects ON sql_big_tables OFF sql_buffer_result OFF sql_log_bin ON sql_log_off OFF sql_log_update ON sql_low_priority_updates OFF sql_max_join_size 18446744073709551615 sql_mode sql_notes ON sql_quote_show_create ON sql_safe_updates OFF sql_select_limit 18446744073709551615 sql_slave_skip_counter sql_warnings OFF ssl_ca ssl_capath ssl_cert ssl_cipher ssl_key storage_engine MyISAM sync_binlog 0 sync_frm ON system_time_zone PDT table_definition_cache 256 table_lock_wait_timeout 50 table_open_cache 64 table_type MyISAM thread_cache_size 0 thread_handling one-thread-per-connection thread_stack 262144 time_format %H:%i:%s time_zone SYSTEM timed_mutexes OFF timestamp 1372254399 tmp_table_size 16777216 tmpdir /tmp transaction_alloc_block_size 8192 transaction_prealloc_size 4096 tx_isolation REPEATABLE-READ unique_checks ON updatable_views_with_limit YES version 5.1.69 version_comment Source distribution version_compile_machine x86_64 version_compile_os redhat-linux-gnu wait_timeout 28800 warning_count 0 VPS Sysbench Info Deleted to stay under 30000 characters. Laptop Sysbench Info [root@server1 ~]# cat sysbench.txt sysbench 0.4.12: multi-threaded system evaluation benchmark Running the test with following options: Number of threads: 8 Doing OLTP test. Running mixed OLTP test Doing read-only test Using Special distribution (12 iterations, 1 pct of values are returned in 75 pct cases) Using "BEGIN" for starting transactions Using auto_inc on the id column Threads started! Time limit exceeded, exiting... (last message repeated 7 times) Done. OLTP test statistics: queries performed: read: 634718 write: 0 other: 90674 total: 725392 transactions: 45337 (755.56 per sec.) deadlocks: 0 (0.00 per sec.) read/write requests: 634718 (10577.78 per sec.) other operations: 90674 (1511.11 per sec.) Test execution summary: total time: 60.0048s total number of events: 45337 total time taken by event execution: 479.4912 per-request statistics: min: 2.04ms avg: 10.58ms max: 85.56ms approx. 95 percentile: 19.70ms Threads fairness: events (avg/stddev): 5667.1250/42.18 execution time (avg/stddev): 59.9364/0.00 [root@server1 ~]# VPS File Info [root@vps ~]# df -T Filesystem Type 1K-blocks Used Available Use% Mounted on /dev/simfs simfs 20971520 16187440 4784080 78% / none tmpfs 6224432 4 6224428 1% /dev none tmpfs 6224432 0 6224432 0% /dev/shm [root@vps ~]# Laptop File Info [root@server1 ~]# df -T Filesystem Type 1K-blocks Used Available Use% Mounted on /dev/mapper/vg_server1-lv_root ext4 72383800 4243964 64462860 7% / tmpfs tmpfs 956352 0 956352 0% /dev/shm /dev/sdb1 ext4 495844 60948 409296 13% /boot [root@server1 ~]# VPS CPU Info Removed to stay under the 30000 character limit required by ServerFault Laptop CPU Info [root@server1 ~]# cat /proc/cpuinfo processor : 0 vendor_id : GenuineIntel cpu family : 6 model : 15 model name : Intel(R) Core(TM)2 Duo CPU T7100 @ 1.80GHz stepping : 13 cpu MHz : 800.000 cache size : 2048 KB physical id : 0 siblings : 2 core id : 0 cpu cores : 2 apicid : 0 initial apicid : 0 fpu : yes fpu_exception : yes cpuid level : 10 wp : yes flags : fpu vme de pse tsc msr pae mce cx8 apic sep mtrr pge mca cmov pat pse36 clflush dts acpi mmx fxsr sse sse2 ss ht tm pbe syscall nx lm constant_tsc arch_perfmon pebs bts rep_good aperfmperf pni dtes64 monitor ds_cpl vmx est tm2 ssse3 cx16 xtpr pdcm lahf_lm ida dts tpr_shadow vnmi flexpriority bogomips : 3591.39 clflush size : 64 cache_alignment : 64 address sizes : 36 bits physical, 48 bits virtual power management: processor : 1 vendor_id : GenuineIntel cpu family : 6 model : 15 model name : Intel(R) Core(TM)2 Duo CPU T7100 @ 1.80GHz stepping : 13 cpu MHz : 800.000 cache size : 2048 KB physical id : 0 siblings : 2 core id : 1 cpu cores : 2 apicid : 1 initial apicid : 1 fpu : yes fpu_exception : yes cpuid level : 10 wp : yes flags : fpu vme de pse tsc msr pae mce cx8 apic sep mtrr pge mca cmov pat pse36 clflush dts acpi mmx fxsr sse sse2 ss ht tm pbe syscall nx lm constant_tsc arch_perfmon pebs bts rep_good aperfmperf pni dtes64 monitor ds_cpl vmx est tm2 ssse3 cx16 xtpr pdcm lahf_lm ida dts tpr_shadow vnmi flexpriority bogomips : 3591.39 clflush size : 64 cache_alignment : 64 address sizes : 36 bits physical, 48 bits virtual power management: [root@server1 ~]# EDIT New Info requested by shakalandy [root@localhost ~]# cat /proc/meminfo MemTotal: 2044804 kB MemFree: 761464 kB Buffers: 68868 kB Cached: 369708 kB SwapCached: 0 kB Active: 881080 kB Inactive: 246016 kB Active(anon): 688312 kB Inactive(anon): 4416 kB Active(file): 192768 kB Inactive(file): 241600 kB Unevictable: 0 kB Mlocked: 0 kB SwapTotal: 4095992 kB SwapFree: 4095992 kB Dirty: 0 kB Writeback: 0 kB AnonPages: 688428 kB Mapped: 65156 kB Shmem: 4216 kB Slab: 92428 kB SReclaimable: 31260 kB SUnreclaim: 61168 kB KernelStack: 2392 kB PageTables: 28356 kB NFS_Unstable: 0 kB Bounce: 0 kB WritebackTmp: 0 kB CommitLimit: 5118392 kB Committed_AS: 1530212 kB VmallocTotal: 34359738367 kB VmallocUsed: 343604 kB VmallocChunk: 34359372920 kB HardwareCorrupted: 0 kB AnonHugePages: 520192 kB HugePages_Total: 0 HugePages_Free: 0 HugePages_Rsvd: 0 HugePages_Surp: 0 Hugepagesize: 2048 kB DirectMap4k: 8556 kB DirectMap2M: 2078720 kB [root@localhost ~]# ps aux | grep mysql root 2227 0.0 0.0 108332 1504 ? S 07:36 0:00 /bin/sh /usr/bin/mysqld_safe --datadir=/var/lib/mysql --pid-file=/var/lib/mysql/localhost.badobe.com.pid mysql 2319 0.1 24.5 1470068 501360 ? Sl 07:36 0:57 /usr/sbin/mysqld --basedir=/usr --datadir=/var/lib/mysql --plugin-dir=/usr/lib64/mysql/plugin --user=mysql --log-error=/var/lib/mysql/localhost.badobe.com.err --pid-file=/var/lib/mysql/localhost.badobe.com.pid root 3579 0.0 0.1 201840 3028 pts/0 S+ 07:40 0:00 mysql -u root -p root 13887 0.0 0.1 201840 3036 pts/3 S+ 18:08 0:00 mysql -uroot -px xxxxxxxxxx root 14449 0.0 0.0 103248 840 pts/2 S+ 18:16 0:00 grep mysql [root@localhost ~]# ps aux | grep mysql root 2227 0.0 0.0 108332 1504 ? S 07:36 0:00 /bin/sh /usr/bin/mysqld_safe --datadir=/var/lib/mysql --pid-file=/var/lib/mysql/localhost.badobe.com.pid mysql 2319 0.1 24.5 1470068 501356 ? Sl 07:36 0:57 /usr/sbin/mysqld --basedir=/usr --datadir=/var/lib/mysql --plugin-dir=/usr/lib64/mysql/plugin --user=mysql --log-error=/var/lib/mysql/localhost.badobe.com.err --pid-file=/var/lib/mysql/localhost.badobe.com.pid root 3579 0.0 0.1 201840 3028 pts/0 S+ 07:40 0:00 mysql -u root -p root 13887 0.0 0.1 201840 3048 pts/3 S+ 18:08 0:00 mysql -uroot -px xxxxxxxxxx root 14470 0.0 0.0 103248 840 pts/2 S+ 18:16 0:00 grep mysql [root@localhost ~]# vmstat 1 procs -----------memory---------- ---swap-- -----io---- --system-- -----cpu----- r b swpd free buff cache si so bi bo in cs us sy id wa st 0 0 0 742172 76376 371064 0 0 6 6 78 202 2 1 97 1 0 0 0 0 742164 76380 371060 0 0 0 16 191 467 2 1 93 5 0 0 0 0 742164 76380 371064 0 0 0 0 148 388 2 1 98 0 0 0 0 0 742164 76380 371064 0 0 0 0 159 418 2 1 98 0 0 0 0 0 742164 76380 371064 0 0 0 0 145 380 2 1 98 0 0 0 0 0 742164 76380 371064 0 0 0 0 166 429 2 1 97 0 0 1 0 0 742164 76380 371064 0 0 0 0 148 373 2 1 98 0 0 0 0 0 742164 76380 371064 0 0 0 0 149 382 2 1 98 0 0 0 0 0 742164 76380 371064 0 0 0 0 168 408 2 0 97 0 0 0 0 0 742164 76380 371064 0 0 0 0 165 394 2 1 98 0 0 0 0 0 742164 76380 371064 0 0 0 0 159 354 2 1 98 0 0 0 0 0 742164 76388 371060 0 0 0 16 180 447 2 0 91 6 0 0 0 0 742164 76388 371064 0 0 0 0 143 344 2 1 98 0 0 0 1 0 742784 76416 370044 0 0 28 580 360 678 3 1 74 23 0 1 0 0 744768 76496 367772 0 0 40 1036 437 865 3 1 53 43 0 0 1 0 747248 76596 365412 0 0 48 1224 561 923 3 2 53 43 0 0 1 0 749232 76696 363092 0 0 32 1132 512 883 3 2 52 44 0 0 1 0 751340 76772 361020 0 0 32 1008 472 872 2 1 52 45 0 0 1 0 753448 76840 358540 0 0 36 1088 512 860 2 1 51 46 0 0 1 0 755060 76936 357636 0 0 28 1012 481 922 2 2 52 45 0 0 1 0 755060 77064 357988 0 0 12 896 444 902 2 1 53 45 0 0 1 0 754688 77148 358448 0 0 16 1096 506 1007 1 1 56 42 0 0 2 0 754192 77268 358932 0 0 12 1060 481 957 1 2 53 44 0 0 1 0 753696 77380 359392 0 0 12 1052 512 1025 2 1 55 42 0 0 1 0 751028 77480 359828 0 0 8 984 423 909 2 2 52 45 0 0 1 0 750524 77620 360200 0 0 8 788 367 869 1 2 54 44 0 0 1 0 749904 77700 360664 0 0 8 928 439 924 2 2 55 43 0 0 1 0 749408 77796 361084 0 0 12 976 468 967 1 1 56 43 0 0 1 0 748788 77896 361464 0 0 12 992 453 944 1 2 54 43 0 1 1 0 748416 77992 361996 0 0 12 784 392 868 2 1 52 46 0 0 1 0 747920 78092 362336 0 0 4 896 382 874 1 1 52 46 0 0 1 0 745252 78172 362780 0 0 12 1040 444 923 1 1 56 42 0 0 1 0 744764 78288 363220 0 0 8 1024 448 934 2 1 55 43 0 0 1 0 744144 78408 363668 0 0 8 1000 461 982 2 1 53 44 0 0 1 0 743648 78488 364148 0 0 8 872 443 888 2 1 54 43 0 0 1 0 743152 78548 364468 0 0 16 1020 511 995 2 1 55 43 0 0 1 0 742656 78632 365024 0 0 12 928 431 913 1 2 53 44 0 0 1 0 742160 78728 365468 0 0 12 996 470 955 2 2 54 44 0 1 1 0 739492 78840 365896 0 0 8 988 447 939 1 2 52 46 0 0 1 0 738872 78996 366352 0 0 12 972 442 928 1 1 55 44 0 1 1 0 738244 79148 366812 0 0 8 948 549 1126 2 2 54 43 0 0 1 0 737624 79312 367188 0 0 12 996 456 953 2 2 54 43 0 0 1 0 736880 79456 367660 0 0 12 960 444 918 1 1 53 46 0 0 1 0 736260 79584 368124 0 0 8 884 414 921 1 1 54 44 0 0 1 0 735648 79716 368488 0 0 12 976 450 955 2 1 56 41 0 0 1 0 733104 79840 368988 0 0 12 932 453 918 1 2 55 43 0 0 1 0 732608 79996 369356 0 0 16 916 444 889 1 2 54 43 0 1 1 0 731476 80128 369800 0 0 16 852 514 978 2 2 54 43 0 0 1 0 731244 80252 370200 0 0 8 904 398 870 2 1 55 43 0 1 1 0 730624 80384 370612 0 0 12 1032 447 977 1 2 57 41 0 0 1 0 730004 80524 371096 0 0 12 984 469 941 2 2 52 45 0 0 1 0 729508 80636 371544 0 0 12 928 438 922 2 1 52 46 0 0 1 0 728888 80756 371948 0 0 16 972 439 943 2 1 55 43 0 0 1 0 726468 80900 372272 0 0 8 960 545 1024 2 1 54 43 0 1 1 0 726344 81024 372272 0 0 8 464 490 1057 1 2 53 44 0 0 1 0 726096 81148 372276 0 0 4 328 441 1063 2 1 53 45 0 1 1 0 726096 81256 372292 0 0 0 296 387 975 1 1 53 45 0 0 1 0 725848 81380 372284 0 0 4 332 425 1034 2 1 54 44 0 1 1 0 725848 81496 372300 0 0 4 308 386 992 2 1 54 43 0 0 1 0 725600 81616 372296 0 0 4 328 404 1060 1 1 54 44 0 procs -----------memory---------- ---swap-- -----io---- --system-- -----cpu----- r b swpd free buff cache si so bi bo in cs us sy id wa st 0 1 0 725600 81732 372296 0 0 4 328 439 1011 1 1 53 44 0 0 1 0 725476 81848 372308 0 0 0 316 441 1023 2 2 52 46 0 1 1 0 725352 81972 372300 0 0 4 344 451 1021 1 1 55 43 0 2 1 0 725228 82088 372320 0 0 0 328 427 1058 1 1 54 44 0 1 1 0 724980 82220 372300 0 0 4 336 419 999 2 1 54 44 0 1 1 0 724980 82328 372320 0 0 4 320 430 1019 1 1 54 44 0 1 1 0 724732 82436 372328 0 0 0 388 363 942 2 1 54 44 0 1 1 0 724608 82560 372312 0 0 4 308 419 993 1 2 54 44 0 1 0 0 724360 82684 372320 0 0 0 304 421 1028 2 1 55 42 0 1 0 0 724360 82684 372388 0 0 0 0 158 416 2 1 98 0 0 1 1 0 724236 82720 372360 0 0 0 6464 243 855 3 2 84 12 0 1 0 0 724112 82748 372360 0 0 0 5356 266 895 3 1 84 12 0 2 1 0 724112 82764 372380 0 0 0 3052 221 511 2 2 93 4 0 1 0 0 724112 82796 372372 0 0 0 4548 325 1067 2 2 81 16 0 1 0 0 724112 82816 372368 0 0 0 3240 259 829 3 1 90 6 0 1 0 0 724112 82836 372380 0 0 0 3260 309 822 3 2 88 8 0 1 1 0 724112 82876 372364 0 0 0 4680 326 978 3 1 77 19 0 1 0 0 724112 82884 372380 0 0 0 512 207 508 2 1 95 2 0 1 0 0 724112 82884 372388 0 0 0 0 138 361 2 1 98 0 0 1 0 0 724112 82884 372388 0 0 0 0 158 397 2 1 98 0 0 1 0 0 724112 82884 372388 0 0 0 0 146 395 2 1 98 0 0 2 0 0 724112 82884 372388 0 0 0 0 160 395 2 1 98 0 0 1 0 0 724112 82884 372388 0 0 0 0 163 382 1 1 98 0 0 1 0 0 724112 82884 372388 0 0 0 0 176 422 2 1 98 0 0 1 0 0 724112 82884 372388 0 0 0 0 134 351 2 1 98 0 0 0 0 0 724112 82884 372388 0 0 0 0 190 429 2 1 97 0 0 0 0 0 724104 82884 372392 0 0 0 0 139 358 2 1 98 0 0 0 0 0 724848 82884 372392 0 0 0 4 211 432 2 1 97 0 0 1 0 0 724980 82884 372392 0 0 0 0 166 370 2 1 98 0 0 0 0 0 724980 82884 372392 0 0 0 0 164 397 2 1 98 0 0 ^C [root@localhost ~]# Database size mysql> SELECT table_schema "Data Base Name", sum( data_length + index_length ) / 1024 / 1024 "Data Base Size in MB", sum( data_free )/ 1024 / 1024 "Free Space in MB" FROM information_schema.TABLES GROUP BY table_schema; +--------------------+----------------------+------------------+ | Data Base Name | Data Base Size in MB | Free Space in MB | +--------------------+----------------------+------------------+ | bidjunction | 4.68750000 | 0.00000000 | | information_schema | 0.00976563 | 0.00000000 | | mysql | 0.63899899 | 0.00105286 | +--------------------+----------------------+------------------+ 3 rows in set (0.01 sec) mysql> Before Query mysql> SHOW SESSION STATUS like '%Tmp%'; +-------------------------+-------+ | Variable_name | Value | +-------------------------+-------+ | Created_tmp_disk_tables | 0 | | Created_tmp_files | 6 | | Created_tmp_tables | 0 | +-------------------------+-------+ 3 rows in set (0.00 sec) mysql> After Query mysql> SHOW SESSION STATUS like '%Tmp%'; +-------------------------+-------+ | Variable_name | Value | +-------------------------+-------+ | Created_tmp_disk_tables | 0 | | Created_tmp_files | 6 | | Created_tmp_tables | 2 | +-------------------------+-------+ 3 rows in set (0.00 sec) mysql>

    Read the article

  • Linux router: ping doesn't route back

    - by El Barto
    I have a Debian box which I'm trying to set up as a router and an Ubuntu box which I'm using as a client. My problem is that when the Ubuntu client tries to ping a server on the Internet, all the packets are lost (though, as you can see below, they seem to go to the server and back without problem). I'm doing this in the Ubuntu Box: # ping -I eth1 my.remote-server.com PING my.remote-server.com (X.X.X.X) from 10.1.1.12 eth1: 56(84) bytes of data. ^C --- my.remote-server.com ping statistics --- 13 packets transmitted, 0 received, 100% packet loss, time 12094ms (I changed the name and IP of the remote server for privacy). From the Debian Router I see this: # tcpdump -i eth1 -qtln icmp tcpdump: verbose output suppressed, use -v or -vv for full protocol decode listening on eth1, link-type EN10MB (Ethernet), capture size 65535 bytes IP X.X.X.X > 10.1.1.12: ICMP echo reply, id 305, seq 7, length 64 IP 10.1.1.12 > X.X.X.X: ICMP echo request, id 305, seq 8, length 64 IP X.X.X.X > 10.1.1.12: ICMP echo reply, id 305, seq 8, length 64 IP 10.1.1.12 > X.X.X.X: ICMP echo request, id 305, seq 9, length 64 IP X.X.X.X > 10.1.1.12: ICMP echo reply, id 305, seq 9, length 64 IP 10.1.1.12 > X.X.X.X: ICMP echo request, id 305, seq 10, length 64 IP X.X.X.X > 10.1.1.12: ICMP echo reply, id 305, seq 10, length 64 IP 10.1.1.12 > X.X.X.X: ICMP echo request, id 305, seq 11, length 64 IP X.X.X.X > 10.1.1.12: ICMP echo reply, id 305, seq 11, length 64 ^C 9 packets captured 9 packets received by filter 0 packets dropped by kernel # tcpdump -i eth2 -qtln icmp tcpdump: verbose output suppressed, use -v or -vv for full protocol decode listening on eth2, link-type EN10MB (Ethernet), capture size 65535 bytes IP 192.168.1.10 > X.X.X.X: ICMP echo request, id 360, seq 213, length 64 IP X.X.X.X > 192.168.1.10: ICMP echo reply, id 360, seq 213, length 64 IP 192.168.1.10 > X.X.X.X: ICMP echo request, id 360, seq 214, length 64 IP X.X.X.X > 192.168.1.10: ICMP echo reply, id 360, seq 214, length 64 IP 192.168.1.10 > X.X.X.X: ICMP echo request, id 360, seq 215, length 64 IP X.X.X.X > 192.168.1.10: ICMP echo reply, id 360, seq 215, length 64 IP 192.168.1.10 > X.X.X.X: ICMP echo request, id 360, seq 216, length 64 IP X.X.X.X > 192.168.1.10: ICMP echo reply, id 360, seq 216, length 64 IP 192.168.1.10 > X.X.X.X: ICMP echo request, id 360, seq 217, length 64 IP X.X.X.X > 192.168.1.10: ICMP echo reply, id 360, seq 217, length 64 ^C 10 packets captured 10 packets received by filter 0 packets dropped by kernel And at the remote server I see this: # tcpdump -i eth0 -qtln icmp tcpdump: verbose output suppressed, use -v or -vv for full protocol decode listening on eth0, link-type EN10MB (Ethernet), capture size 96 bytes IP Y.Y.Y.Y > X.X.X.X: ICMP echo request, id 360, seq 1, length 64 IP X.X.X.X > Y.Y.Y.Y: ICMP echo reply, id 360, seq 1, length 64 IP Y.Y.Y.Y > X.X.X.X: ICMP echo request, id 360, seq 2, length 64 IP X.X.X.X > Y.Y.Y.Y: ICMP echo reply, id 360, seq 2, length 64 IP Y.Y.Y.Y > X.X.X.X: ICMP echo request, id 360, seq 3, length 64 IP X.X.X.X > Y.Y.Y.Y: ICMP echo reply, id 360, seq 3, length 64 IP Y.Y.Y.Y > X.X.X.X: ICMP echo request, id 360, seq 4, length 64 IP X.X.X.X > Y.Y.Y.Y: ICMP echo reply, id 360, seq 4, length 64 IP Y.Y.Y.Y > X.X.X.X: ICMP echo request, id 360, seq 5, length 64 IP X.X.X.X > Y.Y.Y.Y: ICMP echo reply, id 360, seq 5, length 64 IP Y.Y.Y.Y > X.X.X.X: ICMP echo request, id 360, seq 6, length 64 IP X.X.X.X > Y.Y.Y.Y: ICMP echo reply, id 360, seq 6, length 64 IP Y.Y.Y.Y > X.X.X.X: ICMP echo request, id 360, seq 7, length 64 IP X.X.X.X > Y.Y.Y.Y: ICMP echo reply, id 360, seq 7, length 64 IP Y.Y.Y.Y > X.X.X.X: ICMP echo request, id 360, seq 8, length 64 IP X.X.X.X > Y.Y.Y.Y: ICMP echo reply, id 360, seq 8, length 64 IP Y.Y.Y.Y > X.X.X.X: ICMP echo request, id 360, seq 9, length 64 IP X.X.X.X > Y.Y.Y.Y: ICMP echo reply, id 360, seq 9, length 64 18 packets captured 228 packets received by filter 92 packets dropped by kernel Here "X.X.X.X" is my remote server's IP and "Y.Y.Y.Y" is my local network's public IP. So, what I understand is that the ping packets are coming out of the Ubuntu box (10.1.1.12), to the router (10.1.1.1), from there to the next router (192.168.1.1) and reaching the remote server (X.X.X.X). Then they come back all the way to the Debian router, but they never reach the Ubuntu box back. What am I missing? Here's the Debian router setup: # ifconfig eth1 Link encap:Ethernet HWaddr 94:0c:6d:82:0d:98 inet addr:10.1.1.1 Bcast:10.1.1.255 Mask:255.255.255.0 inet6 addr: fe80::960c:6dff:fe82:d98/64 Scope:Link UP BROADCAST RUNNING MULTICAST MTU:1500 Metric:1 RX packets:105761 errors:0 dropped:0 overruns:0 frame:0 TX packets:48944 errors:0 dropped:0 overruns:0 carrier:0 collisions:0 txqueuelen:1000 RX bytes:40298768 (38.4 MiB) TX bytes:44831595 (42.7 MiB) Interrupt:19 Base address:0x6000 eth2 Link encap:Ethernet HWaddr 6c:f0:49:a4:47:38 inet addr:192.168.1.10 Bcast:192.168.1.255 Mask:255.255.255.0 inet6 addr: fe80::6ef0:49ff:fea4:4738/64 Scope:Link UP BROADCAST RUNNING MULTICAST MTU:1500 Metric:1 RX packets:38335992 errors:0 dropped:0 overruns:0 frame:0 TX packets:37097705 errors:0 dropped:0 overruns:0 carrier:1 collisions:0 txqueuelen:1000 RX bytes:4260680226 (3.9 GiB) TX bytes:3759806551 (3.5 GiB) Interrupt:27 eth3 Link encap:Ethernet HWaddr 94:0c:6d:82:c8:72 UP BROADCAST MULTICAST MTU:1500 Metric:1 RX packets:0 errors:0 dropped:0 overruns:0 frame:0 TX packets:0 errors:0 dropped:0 overruns:0 carrier:0 collisions:0 txqueuelen:1000 RX bytes:0 (0.0 B) TX bytes:0 (0.0 B) Interrupt:20 Base address:0x2000 lo Link encap:Local Loopback inet addr:127.0.0.1 Mask:255.0.0.0 inet6 addr: ::1/128 Scope:Host UP LOOPBACK RUNNING MTU:16436 Metric:1 RX packets:3408 errors:0 dropped:0 overruns:0 frame:0 TX packets:3408 errors:0 dropped:0 overruns:0 carrier:0 collisions:0 txqueuelen:0 RX bytes:358445 (350.0 KiB) TX bytes:358445 (350.0 KiB) tun0 Link encap:UNSPEC HWaddr 00-00-00-00-00-00-00-00-00-00-00-00-00-00-00-00 inet addr:10.8.0.1 P-t-P:10.8.0.2 Mask:255.255.255.255 UP POINTOPOINT RUNNING NOARP MULTICAST MTU:1500 Metric:1 RX packets:2767779 errors:0 dropped:0 overruns:0 frame:0 TX packets:1569477 errors:0 dropped:0 overruns:0 carrier:0 collisions:0 txqueuelen:100 RX bytes:3609469393 (3.3 GiB) TX bytes:96113978 (91.6 MiB) # route -n Kernel IP routing table Destination Gateway Genmask Flags Metric Ref Use Iface 10.8.0.2 0.0.0.0 255.255.255.255 UH 0 0 0 tun0 127.0.0.1 0.0.0.0 255.255.255.255 UH 0 0 0 lo 10.8.0.0 10.8.0.2 255.255.255.0 UG 0 0 0 tun0 192.168.1.0 0.0.0.0 255.255.255.0 U 1 0 0 eth2 10.1.1.0 0.0.0.0 255.255.255.0 U 0 0 0 eth1 0.0.0.0 192.168.1.1 0.0.0.0 UG 0 0 0 eth2 # arp -n # Note: Here I have changed all the different MACs except the ones corresponding to the Ubuntu box (on 10.1.1.12 and 192.168.1.12) Address HWtype HWaddress Flags Mask Iface 192.168.1.118 ether NN:NN:NN:NN:NN:NN C eth2 192.168.1.72 ether NN:NN:NN:NN:NN:NN C eth2 192.168.1.94 ether NN:NN:NN:NN:NN:NN C eth2 192.168.1.102 ether NN:NN:NN:NN:NN:NN C eth2 10.1.1.12 ether 00:1e:67:15:2b:f0 C eth1 192.168.1.86 ether NN:NN:NN:NN:NN:NN C eth2 192.168.1.2 ether NN:NN:NN:NN:NN:NN C eth2 192.168.1.61 ether NN:NN:NN:NN:NN:NN C eth2 192.168.1.64 ether NN:NN:NN:NN:NN:NN C eth2 192.168.1.116 ether NN:NN:NN:NN:NN:NN C eth2 192.168.1.91 ether NN:NN:NN:NN:NN:NN C eth2 192.168.1.52 ether NN:NN:NN:NN:NN:NN C eth2 192.168.1.93 ether NN:NN:NN:NN:NN:NN C eth2 192.168.1.87 ether NN:NN:NN:NN:NN:NN C eth2 192.168.1.92 ether NN:NN:NN:NN:NN:NN C eth2 192.168.1.100 ether NN:NN:NN:NN:NN:NN C eth2 192.168.1.40 ether NN:NN:NN:NN:NN:NN C eth2 192.168.1.53 ether NN:NN:NN:NN:NN:NN C eth2 192.168.1.1 ether NN:NN:NN:NN:NN:NN C eth2 192.168.1.83 ether NN:NN:NN:NN:NN:NN C eth2 192.168.1.89 ether NN:NN:NN:NN:NN:NN C eth2 192.168.1.12 ether 00:1e:67:15:2b:f1 C eth2 192.168.1.77 ether NN:NN:NN:NN:NN:NN C eth2 192.168.1.66 ether NN:NN:NN:NN:NN:NN C eth2 192.168.1.90 ether NN:NN:NN:NN:NN:NN C eth2 192.168.1.65 ether NN:NN:NN:NN:NN:NN C eth2 192.168.1.41 ether NN:NN:NN:NN:NN:NN C eth2 192.168.1.78 ether NN:NN:NN:NN:NN:NN C eth2 192.168.1.123 ether NN:NN:NN:NN:NN:NN C eth2 # iptables -L -n Chain INPUT (policy ACCEPT) target prot opt source destination Chain FORWARD (policy ACCEPT) target prot opt source destination Chain OUTPUT (policy ACCEPT) target prot opt source destination # iptables -L -n -t nat Chain PREROUTING (policy ACCEPT) target prot opt source destination Chain POSTROUTING (policy ACCEPT) target prot opt source destination MASQUERADE all -- 10.1.1.0/24 !10.1.1.0/24 MASQUERADE all -- !10.1.1.0/24 10.1.1.0/24 Chain OUTPUT (policy ACCEPT) target prot opt source destination And here's the Ubuntu box: # ifconfig eth0 Link encap:Ethernet HWaddr 00:1e:67:15:2b:f1 inet addr:192.168.1.12 Bcast:192.168.1.255 Mask:255.255.255.0 inet6 addr: fe80::21e:67ff:fe15:2bf1/64 Scope:Link UP BROADCAST RUNNING MULTICAST MTU:1500 Metric:1 RX packets:28785139 errors:0 dropped:0 overruns:0 frame:0 TX packets:19050735 errors:0 dropped:0 overruns:0 carrier:0 collisions:0 txqueuelen:1000 RX bytes:32068182803 (32.0 GB) TX bytes:6061333280 (6.0 GB) Interrupt:16 Memory:b1a00000-b1a20000 eth1 Link encap:Ethernet HWaddr 00:1e:67:15:2b:f0 inet addr:10.1.1.12 Bcast:10.1.1.255 Mask:255.255.255.0 inet6 addr: fe80::21e:67ff:fe15:2bf0/64 Scope:Link UP BROADCAST RUNNING MULTICAST MTU:1500 Metric:1 RX packets:285086 errors:0 dropped:0 overruns:0 frame:0 TX packets:12719 errors:0 dropped:0 overruns:0 carrier:0 collisions:0 txqueuelen:1000 RX bytes:30817249 (30.8 MB) TX bytes:2153228 (2.1 MB) Interrupt:16 Memory:b1900000-b1920000 lo Link encap:Local Loopback inet addr:127.0.0.1 Mask:255.0.0.0 inet6 addr: ::1/128 Scope:Host UP LOOPBACK RUNNING MTU:16436 Metric:1 RX packets:86048 errors:0 dropped:0 overruns:0 frame:0 TX packets:86048 errors:0 dropped:0 overruns:0 carrier:0 collisions:0 txqueuelen:0 RX bytes:11426538 (11.4 MB) TX bytes:11426538 (11.4 MB) # route -n Kernel IP routing table Destination Gateway Genmask Flags Metric Ref Use Iface 0.0.0.0 192.168.1.1 0.0.0.0 UG 0 0 0 eth0 0.0.0.0 10.1.1.1 0.0.0.0 UG 100 0 0 eth1 10.1.1.0 0.0.0.0 255.255.255.0 U 0 0 0 eth1 10.8.0.0 192.168.1.10 255.255.255.0 UG 0 0 0 eth0 169.254.0.0 0.0.0.0 255.255.0.0 U 1000 0 0 eth0 192.168.1.0 0.0.0.0 255.255.255.0 U 1 0 0 eth0 # arp -n # Note: Here I have changed all the different MACs except the ones corresponding to the Debian box (on 10.1.1.1 and 192.168.1.10) Address HWtype HWaddress Flags Mask Iface 192.168.1.70 ether NN:NN:NN:NN:NN:NN C eth0 192.168.1.90 ether NN:NN:NN:NN:NN:NN C eth0 192.168.1.97 ether NN:NN:NN:NN:NN:NN C eth0 192.168.1.103 ether NN:NN:NN:NN:NN:NN C eth0 192.168.1.13 ether NN:NN:NN:NN:NN:NN C eth0 192.168.1.120 (incomplete) eth0 192.168.1.111 ether NN:NN:NN:NN:NN:NN C eth0 192.168.1.118 ether NN:NN:NN:NN:NN:NN C eth0 192.168.1.51 ether NN:NN:NN:NN:NN:NN C eth0 192.168.1.102 (incomplete) eth0 192.168.1.64 ether NN:NN:NN:NN:NN:NN C eth0 192.168.1.52 ether NN:NN:NN:NN:NN:NN C eth0 192.168.1.74 (incomplete) eth0 192.168.1.94 ether NN:NN:NN:NN:NN:NN C eth0 192.168.1.121 ether NN:NN:NN:NN:NN:NN C eth0 192.168.1.72 ether NN:NN:NN:NN:NN:NN C eth0 192.168.1.87 ether NN:NN:NN:NN:NN:NN C eth0 192.168.1.91 ether NN:NN:NN:NN:NN:NN C eth0 192.168.1.71 ether NN:NN:NN:NN:NN:NN C eth0 192.168.1.78 ether NN:NN:NN:NN:NN:NN C eth0 192.168.1.83 ether NN:NN:NN:NN:NN:NN C eth0 192.168.1.88 (incomplete) eth0 192.168.1.82 ether NN:NN:NN:NN:NN:NN C eth0 192.168.1.98 ether NN:NN:NN:NN:NN:NN C eth0 192.168.1.100 ether NN:NN:NN:NN:NN:NN C eth0 192.168.1.93 ether NN:NN:NN:NN:NN:NN C eth0 192.168.1.73 ether NN:NN:NN:NN:NN:NN C eth0 192.168.1.11 ether NN:NN:NN:NN:NN:NN C eth0 192.168.1.85 (incomplete) eth0 192.168.1.112 ether NN:NN:NN:NN:NN:NN C eth0 192.168.1.89 ether NN:NN:NN:NN:NN:NN C eth0 192.168.1.65 ether NN:NN:NN:NN:NN:NN C eth0 192.168.1.81 ether NN:NN:NN:NN:NN:NN C eth0 10.1.1.1 ether 94:0c:6d:82:0d:98 C eth1 192.168.1.53 ether NN:NN:NN:NN:NN:NN C eth0 192.168.1.116 ether NN:NN:NN:NN:NN:NN C eth0 192.168.1.61 ether NN:NN:NN:NN:NN:NN C eth0 192.168.1.10 ether 6c:f0:49:a4:47:38 C eth0 192.168.1.86 (incomplete) eth0 192.168.1.119 ether NN:NN:NN:NN:NN:NN C eth0 192.168.1.66 ether NN:NN:NN:NN:NN:NN C eth0 192.168.1.1 ether NN:NN:NN:NN:NN:NN C eth0 192.168.1.1 ether NN:NN:NN:NN:NN:NN C eth1 192.168.1.92 ether NN:NN:NN:NN:NN:NN C eth0 # iptables -L -n Chain INPUT (policy ACCEPT) target prot opt source destination Chain FORWARD (policy ACCEPT) target prot opt source destination Chain OUTPUT (policy ACCEPT) target prot opt source destination # iptables -L -n -t nat Chain PREROUTING (policy ACCEPT) target prot opt source destination Chain INPUT (policy ACCEPT) target prot opt source destination Chain OUTPUT (policy ACCEPT) target prot opt source destination Chain POSTROUTING (policy ACCEPT) target prot opt source destination Edit: Following Patrick's suggestion, I did a tcpdump con the Ubuntu box and I see this: # tcpdump -i eth1 -qtln icmp tcpdump: verbose output suppressed, use -v or -vv for full protocol decode listening on eth1, link-type EN10MB (Ethernet), capture size 65535 bytes IP 10.1.1.12 > X.X.X.X: ICMP echo request, id 21967, seq 1, length 64 IP X.X.X.X > 10.1.1.12: ICMP echo reply, id 21967, seq 1, length 64 IP 10.1.1.12 > X.X.X.X: ICMP echo request, id 21967, seq 2, length 64 IP X.X.X.X > 10.1.1.12: ICMP echo reply, id 21967, seq 2, length 64 IP 10.1.1.12 > X.X.X.X: ICMP echo request, id 21967, seq 3, length 64 IP X.X.X.X > 10.1.1.12: ICMP echo reply, id 21967, seq 3, length 64 IP 10.1.1.12 > X.X.X.X: ICMP echo request, id 21967, seq 4, length 64 IP X.X.X.X > 10.1.1.12: ICMP echo reply, id 21967, seq 4, length 64 IP 10.1.1.12 > X.X.X.X: ICMP echo request, id 21967, seq 5, length 64 IP X.X.X.X > 10.1.1.12: ICMP echo reply, id 21967, seq 5, length 64 IP 10.1.1.12 > X.X.X.X: ICMP echo request, id 21967, seq 6, length 64 IP X.X.X.X > 10.1.1.12: ICMP echo reply, id 21967, seq 6, length 64 ^C 12 packets captured 12 packets received by filter 0 packets dropped by kernel So the question is: if all packets seem to be coming and going, why does ping report 100% packet loss?

    Read the article

  • How to reduce RAM consumption when my server is idle

    - by Julien Genestoux
    We use Slicehost, with 512MB instances. We run Ubuntu 9.10 on them. I installed a few packages, and I'm now trying to optimize RAM consumption before running anything on there. A simple ps gives me the list of running processes : # ps faux USER PID %CPU %MEM VSZ RSS TTY STAT START TIME COMMAND root 2 0.0 0.0 0 0 ? S< Jan04 0:00 [kthreadd] root 3 0.0 0.0 0 0 ? S< Jan04 0:15 \_ [migration/0] root 4 0.0 0.0 0 0 ? S< Jan04 0:01 \_ [ksoftirqd/0] root 5 0.0 0.0 0 0 ? S< Jan04 0:00 \_ [watchdog/0] root 6 0.0 0.0 0 0 ? S< Jan04 0:04 \_ [events/0] root 7 0.0 0.0 0 0 ? S< Jan04 0:00 \_ [cpuset] root 8 0.0 0.0 0 0 ? S< Jan04 0:00 \_ [khelper] root 9 0.0 0.0 0 0 ? S< Jan04 0:00 \_ [async/mgr] root 10 0.0 0.0 0 0 ? S< Jan04 0:00 \_ [xenwatch] root 11 0.0 0.0 0 0 ? S< Jan04 0:00 \_ [xenbus] root 13 0.0 0.0 0 0 ? S< Jan04 0:02 \_ [migration/1] root 14 0.0 0.0 0 0 ? S< Jan04 0:00 \_ [ksoftirqd/1] root 15 0.0 0.0 0 0 ? S< Jan04 0:00 \_ [watchdog/1] root 16 0.0 0.0 0 0 ? S< Jan04 0:07 \_ [events/1] root 17 0.0 0.0 0 0 ? S< Jan04 0:02 \_ [migration/2] root 18 0.0 0.0 0 0 ? S< Jan04 0:00 \_ [ksoftirqd/2] root 19 0.0 0.0 0 0 ? S< Jan04 0:00 \_ [watchdog/2] root 20 0.0 0.0 0 0 ? R< Jan04 0:07 \_ [events/2] root 21 0.0 0.0 0 0 ? S< Jan04 0:04 \_ [migration/3] root 22 0.0 0.0 0 0 ? S< Jan04 0:00 \_ [ksoftirqd/3] root 23 0.0 0.0 0 0 ? S< Jan04 0:00 \_ [watchdog/3] root 24 0.0 0.0 0 0 ? S< Jan04 0:00 \_ [events/3] root 25 0.0 0.0 0 0 ? S< Jan04 0:00 \_ [kintegrityd/0] root 26 0.0 0.0 0 0 ? S< Jan04 0:00 \_ [kintegrityd/1] root 27 0.0 0.0 0 0 ? S< Jan04 0:00 \_ [kintegrityd/2] root 28 0.0 0.0 0 0 ? S< Jan04 0:00 \_ [kintegrityd/3] root 29 0.0 0.0 0 0 ? S< Jan04 0:01 \_ [kblockd/0] root 30 0.0 0.0 0 0 ? S< Jan04 0:00 \_ [kblockd/1] root 31 0.0 0.0 0 0 ? S< Jan04 0:00 \_ [kblockd/2] root 32 0.0 0.0 0 0 ? S< Jan04 0:00 \_ [kblockd/3] root 33 0.0 0.0 0 0 ? S< Jan04 0:00 \_ [kseriod] root 34 0.0 0.0 0 0 ? S Jan04 0:00 \_ [khungtaskd] root 35 0.0 0.0 0 0 ? S Jan04 0:05 \_ [pdflush] root 36 0.0 0.0 0 0 ? S Jan04 0:06 \_ [pdflush] root 37 0.0 0.0 0 0 ? S< Jan04 1:02 \_ [kswapd0] root 38 0.0 0.0 0 0 ? S< Jan04 0:00 \_ [aio/0] root 39 0.0 0.0 0 0 ? S< Jan04 0:00 \_ [aio/1] root 40 0.0 0.0 0 0 ? S< Jan04 0:00 \_ [aio/2] root 41 0.0 0.0 0 0 ? S< Jan04 0:00 \_ [aio/3] root 42 0.0 0.0 0 0 ? S< Jan04 0:00 \_ [jfsIO] root 43 0.0 0.0 0 0 ? S< Jan04 0:00 \_ [jfsCommit] root 44 0.0 0.0 0 0 ? S< Jan04 0:00 \_ [jfsCommit] root 45 0.0 0.0 0 0 ? S< Jan04 0:00 \_ [jfsCommit] root 46 0.0 0.0 0 0 ? S< Jan04 0:00 \_ [jfsCommit] root 47 0.0 0.0 0 0 ? S< Jan04 0:00 \_ [jfsSync] root 48 0.0 0.0 0 0 ? S< Jan04 0:00 \_ [xfs_mru_cache] root 49 0.0 0.0 0 0 ? S< Jan04 0:00 \_ [xfslogd/0] root 50 0.0 0.0 0 0 ? S< Jan04 0:00 \_ [xfslogd/1] root 51 0.0 0.0 0 0 ? S< Jan04 0:00 \_ [xfslogd/2] root 52 0.0 0.0 0 0 ? S< Jan04 0:00 \_ [xfslogd/3] root 53 0.0 0.0 0 0 ? S< Jan04 0:00 \_ [xfsdatad/0] root 54 0.0 0.0 0 0 ? S< Jan04 0:00 \_ [xfsdatad/1] root 55 0.0 0.0 0 0 ? S< Jan04 0:00 \_ [xfsdatad/2] root 56 0.0 0.0 0 0 ? S< Jan04 0:00 \_ [xfsdatad/3] root 57 0.0 0.0 0 0 ? S< Jan04 0:00 \_ [xfsconvertd/0] root 58 0.0 0.0 0 0 ? S< Jan04 0:00 \_ [xfsconvertd/1] root 59 0.0 0.0 0 0 ? S< Jan04 0:00 \_ [xfsconvertd/2] root 60 0.0 0.0 0 0 ? S< Jan04 0:00 \_ [xfsconvertd/3] root 61 0.0 0.0 0 0 ? S< Jan04 0:00 \_ [glock_workqueue] root 62 0.0 0.0 0 0 ? S< Jan04 0:00 \_ [glock_workqueue] root 63 0.0 0.0 0 0 ? S< Jan04 0:00 \_ [glock_workqueue] root 64 0.0 0.0 0 0 ? S< Jan04 0:00 \_ [glock_workqueue] root 65 0.0 0.0 0 0 ? S< Jan04 0:00 \_ [delete_workqueu] root 66 0.0 0.0 0 0 ? S< Jan04 0:00 \_ [delete_workqueu] root 67 0.0 0.0 0 0 ? S< Jan04 0:00 \_ [delete_workqueu] root 68 0.0 0.0 0 0 ? S< Jan04 0:00 \_ [delete_workqueu] root 69 0.0 0.0 0 0 ? S< Jan04 0:00 \_ [kslowd] root 70 0.0 0.0 0 0 ? S< Jan04 0:00 \_ [kslowd] root 71 0.0 0.0 0 0 ? S< Jan04 0:00 \_ [crypto/0] root 72 0.0 0.0 0 0 ? S< Jan04 0:00 \_ [crypto/1] root 73 0.0 0.0 0 0 ? S< Jan04 0:00 \_ [crypto/2] root 74 0.0 0.0 0 0 ? S< Jan04 0:00 \_ [crypto/3] root 77 0.0 0.0 0 0 ? S< Jan04 0:00 \_ [net_accel/0] root 78 0.0 0.0 0 0 ? S< Jan04 0:00 \_ [net_accel/1] root 79 0.0 0.0 0 0 ? S< Jan04 0:00 \_ [net_accel/2] root 80 0.0 0.0 0 0 ? S< Jan04 0:00 \_ [net_accel/3] root 81 0.0 0.0 0 0 ? S< Jan04 0:00 \_ [sfc_netfront/0] root 82 0.0 0.0 0 0 ? S< Jan04 0:00 \_ [sfc_netfront/1] root 83 0.0 0.0 0 0 ? S< Jan04 0:00 \_ [sfc_netfront/2] root 84 0.0 0.0 0 0 ? S< Jan04 0:00 \_ [sfc_netfront/3] root 310 0.0 0.0 0 0 ? S< Jan04 0:00 \_ [kstriped] root 315 0.0 0.0 0 0 ? S< Jan04 0:00 \_ [ksnapd] root 1452 0.0 0.0 0 0 ? S< Jan04 4:31 \_ [kjournald] root 1 0.0 0.1 19292 948 ? Ss Jan04 0:15 /sbin/init root 1545 0.0 0.1 13164 1064 ? S Jan04 0:00 upstart-udev-bridge --daemon root 1547 0.0 0.1 17196 996 ? S<s Jan04 0:00 udevd --daemon root 1728 0.0 0.2 20284 1468 ? S< Jan04 0:00 \_ udevd --daemon root 1729 0.0 0.1 17192 792 ? S< Jan04 0:00 \_ udevd --daemon root 1881 0.0 0.0 8192 152 ? Ss Jan04 0:00 dd bs=1 if=/proc/kmsg of=/var/run/rsyslog/kmsg syslog 1884 0.0 0.2 185252 1200 ? Sl Jan04 1:00 rsyslogd -c4 103 1894 0.0 0.1 23328 700 ? Ss Jan04 1:08 dbus-daemon --system --fork root 2046 0.0 0.0 136 32 ? Ss Jan04 4:05 runsvdir -P /etc/service log: gems/custom_require.rb:31:in `require'??from /mnt/app/superfeedr-firehoser/current/script/component:52?/opt/ruby-enterprise/lib/ruby/si root 2055 0.0 0.0 112 32 ? Ss Jan04 0:00 \_ runsv chef-client root 2060 0.0 0.0 132 40 ? S Jan04 0:02 | \_ svlogd -tt ./main root 2056 0.0 0.0 112 28 ? Ss Jan04 0:20 \_ runsv superfeedr-firehoser_2 root 2059 0.0 0.0 132 40 ? S Jan04 0:29 | \_ svlogd /var/log/superfeedr-firehoser_2 root 2057 0.0 0.0 112 28 ? Ss Jan04 0:20 \_ runsv superfeedr-firehoser_1 root 2062 0.0 0.0 132 44 ? S Jan04 0:26 \_ svlogd /var/log/superfeedr-firehoser_1 root 2058 0.0 0.0 18708 316 ? Ss Jan04 0:01 cron root 2095 0.0 0.1 49072 764 ? Ss Jan04 0:06 /usr/sbin/sshd root 9832 0.0 0.5 78916 3500 ? Ss 00:37 0:00 \_ sshd: root@pts/0 root 9846 0.0 0.3 17900 2036 pts/0 Ss 00:37 0:00 \_ -bash root 10132 0.0 0.1 15020 1064 pts/0 R+ 09:51 0:00 \_ ps faux root 2180 0.0 0.0 5988 140 tty1 Ss+ Jan04 0:00 /sbin/getty -8 38400 tty1 root 27610 0.0 1.4 47060 8436 ? S Apr04 2:21 python /usr/sbin/denyhosts --daemon --purge --config=/etc/denyhosts.conf --config=/etc/denyhosts.conf root 22640 0.0 0.7 119244 4164 ? Ssl Apr05 0:05 /usr/sbin/console-kit-daemon root 10113 0.0 0.0 3904 316 ? Ss 09:46 0:00 /usr/sbin/collectdmon -P /var/run/collectdmon.pid -- -C /etc/collectd/collectd.conf root 10114 0.0 0.2 201084 1464 ? Sl 09:46 0:00 \_ collectd -C /etc/collectd/collectd.conf -f As you can see there is nothing serious here. If I sum up the RSS line on all this, I get the following : # ps -aeo rss | awk '{sum+=$1} END {print sum}' 30096 Which makes sense. However, I have a pretty big surprise when I do a free: # free total used free shared buffers cached Mem: 591180 343684 247496 0 25432 161256 -/+ buffers/cache: 156996 434184 Swap: 1048568 0 1048568 As you can see 60% of the available memory is already consumed... which leaves me with only 40% to run my own applications if I want to avoid swapping. Quite disapointing! 2 questions arise : Where is all this memory? How to take some of it back for my own apps?

    Read the article

  • SOA Suite 11g Native Format Builder Complex Format Example

    - by bob.webster
    This rather long posting details the steps required to process a grouping of fixed length records using Format Builder.   If it’s 10 pm and you’re feeling beat you might want to leave this until tomorrow.  But if it’s 10 pm and you need to get a Format Builder Complex template done, read on… The goal is to process individual orders from a file using the 11g File Adapter and Format Builder Sample Data =========== 001Square Widget            0245.98 102Triagular Widget         1120.00 403Circular Widget           0099.45 ORD8898302/01/2011 301Hexagon Widget         1150.98 ORD6735502/01/2011 The records are fixed length records representing a number of logical Order records. Each order record consists of a number of item records starting with a 3 digit number, followed by a single Summary Record which starts with the constant ORD. How can this file be processed so that the first poll returns the first order? 001Square Widget            0245.98 102Triagular Widget         1120.00 403Circular Widget           0099.45 ORD8898302/01/2011 And the second poll returns the second order? 301Hexagon Widget           1150.98 ORD6735502/01/2011 Note: if you need more than one order per poll, that’s also possible, see the “Multiple Messages” field in the “File Adapter Step 6 of 9” snapshot further down.   To follow along with this example you will need - Studio Edition Version 11.1.1.4.0    with the   - SOA Extension for JDeveloper 11.1.1.4.0 installed Both can be downloaded from here:  http://www.oracle.com/technetwork/middleware/soasuite/downloads/index.html You will not need a running WebLogic Server domain to complete the steps and Format Builder tests in this article.     Start with a SOA Composite containing a File Adapter The Format Builder is part of the File Adapter so start by creating a new SOA Project and Composite. Here is a quick summary for those not familiar with these steps - Start JDeveloper - From the Main Menu choose File->New - In the New Gallery window that opens Expand the “General” category and Select the Applications node.   Then choose SOA Application from the Items section on the right.  Finally press the OK button. - In Step 1 of the “Create SOA Application wizard” that appears enter an Application Name and an Directory of your     choice,   then press the Next button. - In Step 2 of the “Create SOA Application wizard”, press the Next button leaving all entries as defaulted. - In Step 3 of the “Create SOA Application wizard”, Enter a composite name of your choice and Press the Finish   Button These steps result in a new Application and SOA Project. The SOA Project contains a composite.xml file which is opened and shown below. For our example we have not defined a Mediator or a BPEL process to minimize the steps, but one or the other would eventually be needed to use the File Adapter we are about to create. Drag and drop the File Adapter icon from the Component Pallette onto either the LEFT side of the diagram under “Exposed Services” or the right side under “External References”.  (See the Green Circle in the image below).  Placing the adapter on the left side would indicate the file being processed is inbound to the composite, if the adapter is placed on the right side then the data is outbound to a file.     Note that the same Format Builder definition can be used in both directions.  For example we could use the format with a File Adapter on the left side of the composite to parse fixed data into XML, modify the data in our Composite or BPEL process and then use the same Format Builder definition with a File adapter on the right side of the composite to write the data back out in the same fixed data format When the File Adapter is dropped on the Composite the File Adapter Wizard Appears. Skip Past the first page, Step 1 of 9 by pressing the Next button. In Step 2 enter a service name of your choice as shown below, then press Next   When the Native Format Builder appears, skip the welcome page by pressing next. Also press the Next button to accept the settings on Step 3 of 9 On Step 4, select Read File and press the Next button as shown below.   On Step 5 enter a directory that will contain a file with the input data, then  Press the Next button as shown below. In step 6, enter *.txt or another file format to select input files from the input directory mentioned in step 5. ALSO check the “Files contain Multiple Messages” checkbox and set the “Publish Messages in Batches of” field to 1.  The value can be set higher to increase the number of logical order group records returned on each poll of the file adapter.  In other words, it determines the number of Orders that will be sent to each instance of a Mediator or Composite processing using the File Adapter.   Skip Step 7 by pressing the Next button In Step 8 press the Gear Icon on the right side to load the Native Format Builder.       Native Format Builder  appears Before diving into the format, here is an overview of the process. Approach - Bottom up Assuming an Order is a grouping of item records and a summary record…. - Define a separate  Complex Type for each Record Type found in the group.    (One for itemRecord and one for summaryRecord) - Define a Complex Type to contain the Group of Record types defined above   (LogicalOrderRecord) - Define a top level element to represent an order.  (order)   The order element will be of type LogicalOrderRecord   Defining the Format In Step 1 select   “Create new”  and  “Complex Type” and “Next”   In Step two browse to and select a file containing the test data shown at the start of this article. A link is provided at the end of this article to download a file containing the test data. Press the Next button     In Step 3 Complex types must be define for each type of input record. Select the Root-Element and Click on the Add Complex Type icon This creates a new empty complex type definition shown below. The fastest way to create the definition is to highlight the first line of the Sample File data and drag the line onto the  <new_complex_type> Format Builder introspects the data and provides a grid to define additional fields. Change the “Complex Type Name” to  “itemRecord” Then click on the ruler to indicate the position of fixed columns.  Drag the red triangle icons to the exact columns if necessary. Double click on an existing red triangle to remove an unwanted entry. In the case below fields are define in columns 0-3, 4-28, 29-eol When the field definitions are correct, press the “Generate Fields” button. Field entries named C1, C2 and C3 will be created as shown below. Click on the field names and rename them from C1->itemNum, C2->itemDesc and C3->itemCost  When all the fields are correctly defined press OK to save the complex type.        Next, the process is repeated to define a Complex Type for the SummaryRecord. Select the Root-Element in the schema tree and press the new complex type icon Then highlight and drag the Summary Record from the sample data onto the <new_complex_type>   Change the complex type name to “summaryRecord” Mark the fixed fields for Order Number and Order Date. Press the Generate Fields button and rename C1 and C2 to itemNum and orderDate respectively.   The last complex type to be defined is a type to hold the group of items and the summary record. Select the Root-Element in the schema tree and click the new complex type icon Select the “<new_complex_type>” entry and click the pencil icon   On the Complex Type Details page change the name and type of each input field. Change line 1 to be named item and set the Type  to “itemRecord” Change line 2 to be named summary and set the Type to “summaryRecord” We also need to indicate that itemRecords repeat in the input file. Click the pencil icon at the right side of the item line. On the Edit Details page change the “Max Occurs” entry from 1 to UNBOUNDED. We also need to indicate how to identify an itemRecord.  Since each item record has “.” in column 32 we can use this fact to differentiate an item record from a summary record. Change the “Look Ahead” field to value 32 and enter a period in the “Look For” field Press the OK button to save entry.     Finally, its time to create a top level element to represent an order. Select the “Root-Element” in the schema tree and press the New element icon Click on the <new_element> and press the pencil icon.   Set the Element Name to “order” and change the Data Type to “logicalOrderRecord” Press the OK button to save the element definition.   The final definition should match the screenshot below. Press the Next Button to view the definition source.     Press the Test Button to test the definition   Press the Green Triangle Icon to run the test.   And we are presented with an unwelcome error. The error states that the processor ran out of data while working through the definition. The processor was unable to differentiate between itemRecords and summaryRecords and therefore treated the entire file as a list of itemRecords.  At end of file, the “summary” portion of the logicalOrderRecord remained unprocessed but mandatory.   This root cause of this error is the loss of our “lookAhead” definition used to identify itemRecords. This appears to be a bug in the  Native Format Builder 11.1.1.4.0 Luckily, a simple workaround exists. Press the Cancel button and return to the “Step 4 of 4” Window. Manually add    nxsd:lookAhead="32" nxsd:lookFor="."   attributes after the maxOccurs attribute of the item element. as shown in the highlighted text below.   When the lookAhead and lookFor attributes have been added Press the Test button and on the Test page press the Green Triangle. The test is now successful, the first order in the file is returned by the File Adapter.     Below is a complete listing of the Result XML from the right column of the screen above   Try running it The downloaded input test file and completed schema file can be used for testing without following all the Native Format Builder steps in this example. Use the following link to download a file containing the sample data. Download Sample Input Data This is the best approach rather than cutting and pasting the input data at the top of the article.  Since the data is fixed length it’s very important to watch out for trailing spaces in the data and to ensure an eol character at the end of every line. The download file is correctly formatted. The final schema definition can be downloaded at the following link Download Completed Schema Definition   - Save the inputData.txt file to a known location like the xsd folder in your project. - Save the inputData_6.xsd file to the xsd folder in your project. - At step 1 in the Native Format Builder wizard  (as shown above) check the “Edit existing” radio button,    then browse and select the inputData_6.xsd file - At step 2 of the Format Builder configuration Wizard (as shown above) supply the path and filename for    the inputData.txt file. - You can then proceed to the test page and run a test. - Remember the wizard bug will drop the lookAhead and lookFor attributes,  you will need to manually add   nxsd:lookAhead="32" nxsd:lookFor="."    after the maxOccurs attribute of the item element in the   LogicalOrderRecord Complex Type.  (as shown above)   Good Luck with your Format Project

    Read the article

  • Acer aspire 5735z wireless not working after upgrade to 11.10

    - by Jon
    I cant get my wifi card to work at all after upgrading to 11.10 Oneiric. I'm not sure where to start to fix this. Ive tried using the additional drivers tool but this shows that no additional drivers are needed. Before my upgrade I had a drivers working for the Rt2860 chipset. Any help on this would be much appreciated.... thanks Jon jon@ubuntu:~$ ifconfig eth0 Link encap:Ethernet HWaddr 00:1d:72:ec:76:d5 inet addr:192.168.1.134 Bcast:192.168.1.255 Mask:255.255.255.0 inet6 addr: fe80::21d:72ff:feec:76d5/64 Scope:Link UP BROADCAST RUNNING MULTICAST MTU:1500 Metric:1 RX packets:7846 errors:0 dropped:0 overruns:0 frame:0 TX packets:7213 errors:0 dropped:0 overruns:0 carrier:0 collisions:0 txqueuelen:1000 RX bytes:8046624 (8.0 MB) TX bytes:1329442 (1.3 MB) Interrupt:16 lo Link encap:Local Loopback inet addr:127.0.0.1 Mask:255.0.0.0 inet6 addr: ::1/128 Scope:Host UP LOOPBACK RUNNING MTU:16436 Metric:1 RX packets:91 errors:0 dropped:0 overruns:0 frame:0 TX packets:91 errors:0 dropped:0 overruns:0 carrier:0 collisions:0 txqueuelen:0 RX bytes:34497 (34.4 KB) TX bytes:34497 (34.4 KB) Ive included by dmesg output below [ 0.428818] NET: Registered protocol family 2 [ 0.429003] IP route cache hash table entries: 131072 (order: 8, 1048576 bytes) [ 0.430562] TCP established hash table entries: 524288 (order: 11, 8388608 bytes) [ 0.436614] TCP bind hash table entries: 65536 (order: 8, 1048576 bytes) [ 0.437409] TCP: Hash tables configured (established 524288 bind 65536) [ 0.437412] TCP reno registered [ 0.437431] UDP hash table entries: 2048 (order: 4, 65536 bytes) [ 0.437482] UDP-Lite hash table entries: 2048 (order: 4, 65536 bytes) [ 0.437678] NET: Registered protocol family 1 [ 0.437705] pci 0000:00:02.0: Boot video device [ 0.437892] PCI: CLS 64 bytes, default 64 [ 0.437916] Simple Boot Flag at 0x57 set to 0x1 [ 0.438294] audit: initializing netlink socket (disabled) [ 0.438309] type=2000 audit(1319243447.432:1): initialized [ 0.440763] Freeing initrd memory: 13416k freed [ 0.468362] HugeTLB registered 2 MB page size, pre-allocated 0 pages [ 0.488192] VFS: Disk quotas dquot_6.5.2 [ 0.488254] Dquot-cache hash table entries: 512 (order 0, 4096 bytes) [ 0.488888] fuse init (API version 7.16) [ 0.488985] msgmni has been set to 5890 [ 0.489381] Block layer SCSI generic (bsg) driver version 0.4 loaded (major 253) [ 0.489413] io scheduler noop registered [ 0.489415] io scheduler deadline registered [ 0.489460] io scheduler cfq registered (default) [ 0.489583] pcieport 0000:00:1c.0: setting latency timer to 64 [ 0.489633] pcieport 0000:00:1c.0: irq 40 for MSI/MSI-X [ 0.489699] pcieport 0000:00:1c.1: setting latency timer to 64 [ 0.489741] pcieport 0000:00:1c.1: irq 41 for MSI/MSI-X [ 0.489800] pcieport 0000:00:1c.2: setting latency timer to 64 [ 0.489841] pcieport 0000:00:1c.2: irq 42 for MSI/MSI-X [ 0.489904] pcieport 0000:00:1c.3: setting latency timer to 64 [ 0.489944] pcieport 0000:00:1c.3: irq 43 for MSI/MSI-X [ 0.490006] pcieport 0000:00:1c.4: setting latency timer to 64 [ 0.490047] pcieport 0000:00:1c.4: irq 44 for MSI/MSI-X [ 0.490126] pci_hotplug: PCI Hot Plug PCI Core version: 0.5 [ 0.490149] pciehp: PCI Express Hot Plug Controller Driver version: 0.4 [ 0.490196] intel_idle: MWAIT substates: 0x1110 [ 0.490198] intel_idle: does not run on family 6 model 15 [ 0.491240] ACPI: Deprecated procfs I/F for AC is loaded, please retry with CONFIG_ACPI_PROCFS_POWER cleared [ 0.493473] ACPI: AC Adapter [ADP1] (on-line) [ 0.493590] input: Lid Switch as /devices/LNXSYSTM:00/device:00/PNP0C0D:00/input/input0 [ 0.496771] ACPI: Lid Switch [LID0] [ 0.496818] input: Sleep Button as /devices/LNXSYSTM:00/device:00/PNP0C0E:00/input/input1 [ 0.496823] ACPI: Sleep Button [SLPB] [ 0.496865] input: Power Button as /devices/LNXSYSTM:00/LNXPWRBN:00/input/input2 [ 0.496869] ACPI: Power Button [PWRF] [ 0.496900] ACPI: acpi_idle registered with cpuidle [ 0.498719] Monitor-Mwait will be used to enter C-1 state [ 0.498753] Monitor-Mwait will be used to enter C-2 state [ 0.498761] Marking TSC unstable due to TSC halts in idle [ 0.517627] thermal LNXTHERM:00: registered as thermal_zone0 [ 0.517630] ACPI: Thermal Zone [TZS0] (67 C) [ 0.524796] thermal LNXTHERM:01: registered as thermal_zone1 [ 0.524799] ACPI: Thermal Zone [TZS1] (67 C) [ 0.524823] ACPI: Deprecated procfs I/F for battery is loaded, please retry with CONFIG_ACPI_PROCFS_POWER cleared [ 0.524852] ERST: Table is not found! [ 0.524948] Serial: 8250/16550 driver, 32 ports, IRQ sharing enabled [ 0.680991] ACPI: Battery Slot [BAT0] (battery present) [ 0.688567] Linux agpgart interface v0.103 [ 0.688672] agpgart-intel 0000:00:00.0: Intel GM45 Chipset [ 0.688865] agpgart-intel 0000:00:00.0: detected gtt size: 2097152K total, 262144K mappable [ 0.689786] agpgart-intel 0000:00:00.0: detected 65536K stolen memory [ 0.689912] agpgart-intel 0000:00:00.0: AGP aperture is 256M @ 0xd0000000 [ 0.691006] brd: module loaded [ 0.691510] loop: module loaded [ 0.691967] Fixed MDIO Bus: probed [ 0.691990] PPP generic driver version 2.4.2 [ 0.692065] tun: Universal TUN/TAP device driver, 1.6 [ 0.692067] tun: (C) 1999-2004 Max Krasnyansky <[email protected]> [ 0.692146] ehci_hcd: USB 2.0 'Enhanced' Host Controller (EHCI) Driver [ 0.692181] ehci_hcd 0000:00:1a.7: PCI INT C -> GSI 20 (level, low) -> IRQ 20 [ 0.692206] ehci_hcd 0000:00:1a.7: setting latency timer to 64 [ 0.692210] ehci_hcd 0000:00:1a.7: EHCI Host Controller [ 0.692255] ehci_hcd 0000:00:1a.7: new USB bus registered, assigned bus number 1 [ 0.692289] ehci_hcd 0000:00:1a.7: debug port 1 [ 0.696181] ehci_hcd 0000:00:1a.7: cache line size of 64 is not supported [ 0.696202] ehci_hcd 0000:00:1a.7: irq 20, io mem 0xf8904800 [ 0.712014] ehci_hcd 0000:00:1a.7: USB 2.0 started, EHCI 1.00 [ 0.712131] hub 1-0:1.0: USB hub found [ 0.712136] hub 1-0:1.0: 6 ports detected [ 0.712230] ehci_hcd 0000:00:1d.7: PCI INT A -> GSI 23 (level, low) -> IRQ 23 [ 0.712243] ehci_hcd 0000:00:1d.7: setting latency timer to 64 [ 0.712247] ehci_hcd 0000:00:1d.7: EHCI Host Controller [ 0.712287] ehci_hcd 0000:00:1d.7: new USB bus registered, assigned bus number 2 [ 0.712315] ehci_hcd 0000:00:1d.7: debug port 1 [ 0.716201] ehci_hcd 0000:00:1d.7: cache line size of 64 is not supported [ 0.716216] ehci_hcd 0000:00:1d.7: irq 23, io mem 0xf8904c00 [ 0.732014] ehci_hcd 0000:00:1d.7: USB 2.0 started, EHCI 1.00 [ 0.732130] hub 2-0:1.0: USB hub found [ 0.732135] hub 2-0:1.0: 6 ports detected [ 0.732209] ohci_hcd: USB 1.1 'Open' Host Controller (OHCI) Driver [ 0.732223] uhci_hcd: USB Universal Host Controller Interface driver [ 0.732254] uhci_hcd 0000:00:1a.0: PCI INT A -> GSI 20 (level, low) -> IRQ 20 [ 0.732262] uhci_hcd 0000:00:1a.0: setting latency timer to 64 [ 0.732265] uhci_hcd 0000:00:1a.0: UHCI Host Controller [ 0.732298] uhci_hcd 0000:00:1a.0: new USB bus registered, assigned bus number 3 [ 0.732325] uhci_hcd 0000:00:1a.0: irq 20, io base 0x00001820 [ 0.732441] hub 3-0:1.0: USB hub found [ 0.732445] hub 3-0:1.0: 2 ports detected [ 0.732508] uhci_hcd 0000:00:1a.1: PCI INT B -> GSI 20 (level, low) -> IRQ 20 [ 0.732514] uhci_hcd 0000:00:1a.1: setting latency timer to 64 [ 0.732518] uhci_hcd 0000:00:1a.1: UHCI Host Controller [ 0.732553] uhci_hcd 0000:00:1a.1: new USB bus registered, assigned bus number 4 [ 0.732577] uhci_hcd 0000:00:1a.1: irq 20, io base 0x00001840 [ 0.732696] hub 4-0:1.0: USB hub found [ 0.732700] hub 4-0:1.0: 2 ports detected [ 0.732762] uhci_hcd 0000:00:1a.2: PCI INT C -> GSI 20 (level, low) -> IRQ 20 [ 0.732768] uhci_hcd 0000:00:1a.2: setting latency timer to 64 [ 0.732772] uhci_hcd 0000:00:1a.2: UHCI Host Controller [ 0.732805] uhci_hcd 0000:00:1a.2: new USB bus registered, assigned bus number 5 [ 0.732829] uhci_hcd 0000:00:1a.2: irq 20, io base 0x00001860 [ 0.732942] hub 5-0:1.0: USB hub found [ 0.732946] hub 5-0:1.0: 2 ports detected [ 0.733007] uhci_hcd 0000:00:1d.0: PCI INT A -> GSI 23 (level, low) -> IRQ 23 [ 0.733014] uhci_hcd 0000:00:1d.0: setting latency timer to 64 [ 0.733017] uhci_hcd 0000:00:1d.0: UHCI Host Controller [ 0.733057] uhci_hcd 0000:00:1d.0: new USB bus registered, assigned bus number 6 [ 0.733082] uhci_hcd 0000:00:1d.0: irq 23, io base 0x00001880 [ 0.733202] hub 6-0:1.0: USB hub found [ 0.733206] hub 6-0:1.0: 2 ports detected [ 0.733265] uhci_hcd 0000:00:1d.1: PCI INT B -> GSI 17 (level, low) -> IRQ 17 [ 0.733273] uhci_hcd 0000:00:1d.1: setting latency timer to 64 [ 0.733276] uhci_hcd 0000:00:1d.1: UHCI Host Controller [ 0.733313] uhci_hcd 0000:00:1d.1: new USB bus registered, assigned bus number 7 [ 0.733351] uhci_hcd 0000:00:1d.1: irq 17, io base 0x000018a0 [ 0.733466] hub 7-0:1.0: USB hub found [ 0.733470] hub 7-0:1.0: 2 ports detected [ 0.733532] uhci_hcd 0000:00:1d.2: PCI INT C -> GSI 18 (level, low) -> IRQ 18 [ 0.733539] uhci_hcd 0000:00:1d.2: setting latency timer to 64 [ 0.733542] uhci_hcd 0000:00:1d.2: UHCI Host Controller [ 0.733578] uhci_hcd 0000:00:1d.2: new USB bus registered, assigned bus number 8 [ 0.733610] uhci_hcd 0000:00:1d.2: irq 18, io base 0x000018c0 [ 0.733730] hub 8-0:1.0: USB hub found [ 0.733736] hub 8-0:1.0: 2 ports detected [ 0.733843] i8042: PNP: PS/2 Controller [PNP0303:KBD0,PNP0f13:PS2M] at 0x60,0x64 irq 1,12 [ 0.751594] serio: i8042 KBD port at 0x60,0x64 irq 1 [ 0.751605] serio: i8042 AUX port at 0x60,0x64 irq 12 [ 0.751732] mousedev: PS/2 mouse device common for all mice [ 0.752670] rtc_cmos 00:08: RTC can wake from S4 [ 0.752770] rtc_cmos 00:08: rtc core: registered rtc_cmos as rtc0 [ 0.752796] rtc0: alarms up to one month, y3k, 242 bytes nvram, hpet irqs [ 0.752907] device-mapper: uevent: version 1.0.3 [ 0.752976] device-mapper: ioctl: 4.20.0-ioctl (2011-02-02) initialised: [email protected] [ 0.753028] cpuidle: using governor ladder [ 0.753093] cpuidle: using governor menu [ 0.753096] EFI Variables Facility v0.08 2004-May-17 [ 0.753361] TCP cubic registered [ 0.753482] NET: Registered protocol family 10 [ 0.753966] NET: Registered protocol family 17 [ 0.753992] Registering the dns_resolver key type [ 0.754113] PM: Hibernation image not present or could not be loaded. [ 0.754131] registered taskstats version 1 [ 0.771553] Magic number: 15:152:507 [ 0.771667] rtc_cmos 00:08: setting system clock to 2011-10-22 00:30:48 UTC (1319243448) [ 0.772238] BIOS EDD facility v0.16 2004-Jun-25, 0 devices found [ 0.772240] EDD information not available. [ 0.774165] Freeing unused kernel memory: 984k freed [ 0.774504] Write protecting the kernel read-only data: 10240k [ 0.774755] Freeing unused kernel memory: 20k freed [ 0.775093] input: AT Translated Set 2 keyboard as /devices/platform/i8042/serio0/input/input3 [ 0.779727] Freeing unused kernel memory: 1400k freed [ 0.801946] udevd[84]: starting version 173 [ 0.880950] sky2: driver version 1.28 [ 0.881046] sky2 0000:02:00.0: PCI INT A -> GSI 16 (level, low) -> IRQ 16 [ 0.881096] sky2 0000:02:00.0: setting latency timer to 64 [ 0.881197] sky2 0000:02:00.0: Yukon-2 Extreme chip revision 2 [ 0.881871] sky2 0000:02:00.0: irq 45 for MSI/MSI-X [ 0.896273] sky2 0000:02:00.0: eth0: addr 00:1d:72:ec:76:d5 [ 0.910630] ahci 0000:00:1f.2: version 3.0 [ 0.910647] ahci 0000:00:1f.2: PCI INT B -> GSI 19 (level, low) -> IRQ 19 [ 0.910710] ahci 0000:00:1f.2: irq 46 for MSI/MSI-X [ 0.910775] ahci: SSS flag set, parallel bus scan disabled [ 0.910812] ahci 0000:00:1f.2: AHCI 0001.0200 32 slots 4 ports 3 Gbps 0x33 impl SATA mode [ 0.910816] ahci 0000:00:1f.2: flags: 64bit ncq sntf stag pm led clo pio slum part ccc ems sxs [ 0.910821] ahci 0000:00:1f.2: setting latency timer to 64 [ 0.941773] scsi0 : ahci [ 0.941954] scsi1 : ahci [ 0.942038] scsi2 : ahci [ 0.942118] scsi3 : ahci [ 0.942196] scsi4 : ahci [ 0.942268] scsi5 : ahci [ 0.942332] ata1: SATA max UDMA/133 abar m2048@0xf8904000 port 0xf8904100 irq 46 [ 0.942336] ata2: SATA max UDMA/133 abar m2048@0xf8904000 port 0xf8904180 irq 46 [ 0.942339] ata3: DUMMY [ 0.942340] ata4: DUMMY [ 0.942344] ata5: SATA max UDMA/133 abar m2048@0xf8904000 port 0xf8904300 irq 46 [ 0.942347] ata6: SATA max UDMA/133 abar m2048@0xf8904000 port 0xf8904380 irq 46 [ 1.028061] usb 1-5: new high speed USB device number 2 using ehci_hcd [ 1.181775] usbcore: registered new interface driver uas [ 1.260062] ata1: SATA link up 3.0 Gbps (SStatus 123 SControl 300) [ 1.261126] ata1.00: ATA-8: Hitachi HTS543225L9A300, FBEOC40C, max UDMA/133 [ 1.261129] ata1.00: 488397168 sectors, multi 16: LBA48 NCQ (depth 31/32), AA [ 1.262360] ata1.00: configured for UDMA/133 [ 1.262518] scsi 0:0:0:0: Direct-Access ATA Hitachi HTS54322 FBEO PQ: 0 ANSI: 5 [ 1.262716] sd 0:0:0:0: Attached scsi generic sg0 type 0 [ 1.262762] sd 0:0:0:0: [sda] 488397168 512-byte logical blocks: (250 GB/232 GiB) [ 1.262824] sd 0:0:0:0: [sda] Write Protect is off [ 1.262827] sd 0:0:0:0: [sda] Mode Sense: 00 3a 00 00 [ 1.262851] sd 0:0:0:0: [sda] Write cache: enabled, read cache: enabled, doesn't support DPO or FUA [ 1.287277] sda: sda1 sda2 sda3 [ 1.287693] sd 0:0:0:0: [sda] Attached SCSI disk [ 1.580059] ata2: SATA link up 1.5 Gbps (SStatus 113 SControl 300) [ 1.581188] ata2.00: ATAPI: HL-DT-STDVDRAM GT10N, 1.00, max UDMA/100 [ 1.582663] ata2.00: configured for UDMA/100 [ 1.584162] scsi 1:0:0:0: CD-ROM HL-DT-ST DVDRAM GT10N 1.00 PQ: 0 ANSI: 5 [ 1.585821] sr0: scsi3-mmc drive: 24x/24x writer dvd-ram cd/rw xa/form2 cdda tray [ 1.585824] cdrom: Uniform CD-ROM driver Revision: 3.20 [ 1.585953] sr 1:0:0:0: Attached scsi CD-ROM sr0 [ 1.586038] sr 1:0:0:0: Attached scsi generic sg1 type 5 [ 1.632061] usb 6-1: new low speed USB device number 2 using uhci_hcd [ 1.908056] ata5: SATA link down (SStatus 0 SControl 300) [ 2.228065] ata6: SATA link down (SStatus 0 SControl 300) [ 2.228955] Initializing USB Mass Storage driver... [ 2.229052] usbcore: registered new interface driver usb-storage [ 2.229054] USB Mass Storage support registered. [ 2.235827] scsi6 : usb-storage 1-5:1.0 [ 2.235987] usbcore: registered new interface driver ums-realtek [ 2.244451] input: B16_b_02 USB-PS/2 Optical Mouse as /devices/pci0000:00/0000:00:1d.0/usb6/6-1/6-1:1.0/input/input4 [ 2.244598] generic-usb 0003:046D:C025.0001: input,hidraw0: USB HID v1.10 Mouse [B16_b_02 USB-PS/2 Optical Mouse] on usb-0000:00:1d.0-1/input0 [ 2.244620] usbcore: registered new interface driver usbhid [ 2.244622] usbhid: USB HID core driver [ 3.091083] EXT4-fs (loop0): mounted filesystem with ordered data mode. Opts: (null) [ 3.238275] scsi 6:0:0:0: Direct-Access Generic- Multi-Card 1.00 PQ: 0 ANSI: 0 CCS [ 3.348261] sd 6:0:0:0: Attached scsi generic sg2 type 0 [ 3.351897] sd 6:0:0:0: [sdb] Attached SCSI removable disk [ 47.138012] udevd[334]: starting version 173 [ 47.177678] lp: driver loaded but no devices found [ 47.197084] wmi: Mapper loaded [ 47.197526] acer_wmi: Acer Laptop ACPI-WMI Extras [ 47.210227] acer_wmi: Brightness must be controlled by generic video driver [ 47.566578] Disabling lock debugging due to kernel taint [ 47.584050] ndiswrapper version 1.56 loaded (smp=yes, preempt=no) [ 47.620666] type=1400 audit(1319239895.347:2): apparmor="STATUS" operation="profile_load" name="/sbin/dhclient" pid=624 comm="apparmor_parser" [ 47.620934] type=1400 audit(1319239895.347:3): apparmor="STATUS" operation="profile_load" name="/usr/lib/NetworkManager/nm-dhcp-client.action" pid=624 comm="apparmor_parser" [ 47.621108] type=1400 audit(1319239895.347:4): apparmor="STATUS" operation="profile_load" name="/usr/lib/connman/scripts/dhclient-script" pid=624 comm="apparmor_parser" [ 47.633056] [drm] Initialized drm 1.1.0 20060810 [ 47.722594] i915 0000:00:02.0: PCI INT A -> GSI 16 (level, low) -> IRQ 16 [ 47.722602] i915 0000:00:02.0: setting latency timer to 64 [ 47.807152] ndiswrapper (check_nt_hdr:141): kernel is 64-bit, but Windows driver is not 64-bit;bad magic: 010B [ 47.807159] ndiswrapper (load_sys_files:206): couldn't prepare driver 'rt2860' [ 47.807930] ndiswrapper (load_wrap_driver:108): couldn't load driver rt2860; check system log for messages from 'loadndisdriver' [ 47.856250] usbcore: registered new interface driver ndiswrapper [ 47.861772] i915 0000:00:02.0: irq 47 for MSI/MSI-X [ 47.861781] [drm] Supports vblank timestamp caching Rev 1 (10.10.2010). [ 47.861783] [drm] Driver supports precise vblank timestamp query. [ 47.861842] vgaarb: device changed decodes: PCI:0000:00:02.0,olddecodes=io+mem,decodes=io+mem:owns=io+mem [ 47.980620] fixme: max PWM is zero. [ 48.286153] fbcon: inteldrmfb (fb0) is primary device [ 48.287033] Console: switching to colour frame buffer device 170x48 [ 48.287062] fb0: inteldrmfb frame buffer device [ 48.287064] drm: registered panic notifier [ 48.333883] acpi device:02: registered as cooling_device2 [ 48.334053] input: Video Bus as /devices/LNXSYSTM:00/device:00/PNP0A08:00/LNXVIDEO:00/input/input5 [ 48.334128] ACPI: Video Device [GFX0] (multi-head: yes rom: no post: no) [ 48.334203] [drm] Initialized i915 1.6.0 20080730 for 0000:00:02.0 on minor 0 [ 48.334644] HDA Intel 0000:00:1b.0: power state changed by ACPI to D0 [ 48.334652] HDA Intel 0000:00:1b.0: power state changed by ACPI to D0 [ 48.334673] HDA Intel 0000:00:1b.0: PCI INT A -> GSI 21 (level, low) -> IRQ 21 [ 48.334737] HDA Intel 0000:00:1b.0: irq 48 for MSI/MSI-X [ 48.334772] HDA Intel 0000:00:1b.0: setting latency timer to 64 [ 48.356107] Adding 261116k swap on /host/ubuntu/disks/swap.disk. Priority:-1 extents:1 across:261116k [ 48.380946] hda_codec: ALC268: BIOS auto-probing. [ 48.390242] input: HDA Intel Mic as /devices/pci0000:00/0000:00:1b.0/sound/card0/input6 [ 48.390365] input: HDA Intel Headphone as /devices/pci0000:00/0000:00:1b.0/sound/card0/input7 [ 48.490870] EXT4-fs (loop0): re-mounted. Opts: errors=remount-ro,user_xattr [ 48.917990] ppdev: user-space parallel port driver [ 48.950729] type=1400 audit(1319239896.675:5): apparmor="STATUS" operation="profile_load" name="/usr/lib/cups/backend/cups-pdf" pid=941 comm="apparmor_parser" [ 48.951114] type=1400 audit(1319239896.675:6): apparmor="STATUS" operation="profile_load" name="/usr/sbin/cupsd" pid=941 comm="apparmor_parser" [ 48.977706] Synaptics Touchpad, model: 1, fw: 7.2, id: 0x1c0b1, caps: 0xd04733/0xa44000/0xa0000 [ 49.048871] input: SynPS/2 Synaptics TouchPad as /devices/platform/i8042/serio1/input/input8 [ 49.078713] sky2 0000:02:00.0: eth0: enabling interface [ 49.079462] ADDRCONF(NETDEV_UP): eth0: link is not ready [ 50.762266] sky2 0000:02:00.0: eth0: Link is up at 100 Mbps, full duplex, flow control rx [ 50.762702] ADDRCONF(NETDEV_CHANGE): eth0: link becomes ready [ 54.751478] type=1400 audit(1319239902.475:7): apparmor="STATUS" operation="profile_load" name="/usr/lib/lightdm/lightdm-guest-session-wrapper" pid=1039 comm="apparmor_parser" [ 54.755907] type=1400 audit(1319239902.479:8): apparmor="STATUS" operation="profile_replace" name="/sbin/dhclient" pid=1040 comm="apparmor_parser" [ 54.756237] type=1400 audit(1319239902.483:9): apparmor="STATUS" operation="profile_replace" name="/usr/lib/NetworkManager/nm-dhcp-client.action" pid=1040 comm="apparmor_parser" [ 54.756417] type=1400 audit(1319239902.483:10): apparmor="STATUS" operation="profile_replace" name="/usr/lib/connman/scripts/dhclient-script" pid=1040 comm="apparmor_parser" [ 54.764825] type=1400 audit(1319239902.491:11): apparmor="STATUS" operation="profile_load" name="/usr/bin/evince" pid=1041 comm="apparmor_parser" [ 54.768365] type=1400 audit(1319239902.495:12): apparmor="STATUS" operation="profile_load" name="/usr/bin/evince-previewer" pid=1041 comm="apparmor_parser" [ 54.770601] type=1400 audit(1319239902.495:13): apparmor="STATUS" operation="profile_load" name="/usr/bin/evince-thumbnailer" pid=1041 comm="apparmor_parser" [ 54.770729] type=1400 audit(1319239902.495:14): apparmor="STATUS" operation="profile_load" name="/usr/share/gdm/guest-session/Xsession" pid=1038 comm="apparmor_parser" [ 54.775181] type=1400 audit(1319239902.499:15): apparmor="STATUS" operation="profile_load" name="/usr/lib/telepathy/mission-control-5" pid=1043 comm="apparmor_parser" [ 54.775533] type=1400 audit(1319239902.499:16): apparmor="STATUS" operation="profile_load" name="/usr/lib/telepathy/telepathy-*" pid=1043 comm="apparmor_parser" [ 54.936691] init: failsafe main process (891) killed by TERM signal [ 54.944583] init: apport pre-start process (1096) terminated with status 1 [ 55.000373] init: apport post-stop process (1160) terminated with status 1 [ 55.005291] init: gdm main process (1159) killed by TERM signal [ 59.782579] EXT4-fs (loop0): re-mounted. Opts: errors=remount-ro,user_xattr,commit=0 [ 60.992021] eth0: no IPv6 routers present [ 61.936072] device eth0 entered promiscuous mode [ 62.053949] Bluetooth: Core ver 2.16 [ 62.054005] NET: Registered protocol family 31 [ 62.054007] Bluetooth: HCI device and connection manager initialized [ 62.054010] Bluetooth: HCI socket layer initialized [ 62.054012] Bluetooth: L2CAP socket layer initialized [ 62.054993] Bluetooth: SCO socket layer initialized [ 62.058750] Bluetooth: RFCOMM TTY layer initialized [ 62.058758] Bluetooth: RFCOMM socket layer initialized [ 62.058760] Bluetooth: RFCOMM ver 1.11 [ 62.059428] Bluetooth: BNEP (Ethernet Emulation) ver 1.3 [ 62.059432] Bluetooth: BNEP filters: protocol multicast [ 62.460389] init: plymouth-stop pre-start process (1662) terminated with status 1 '

    Read the article

  • EM12c: Using the LIST verb in emcli

    - by SubinDaniVarughese
    Many of us who use EM CLI to write scripts and automate our daily tasks should not miss out on the new list verb released with Oracle Enterprise Manager 12.1.0.3.0. The combination of list and Jython based scripting support in EM CLI makes it easier to achieve automation for complex tasks with just a few lines of code. Before I jump into a script, let me highlight the key attributes of the list verb and why it’s simply excellent! 1. Multiple resources under a single verb:A resource can be set of users or targets, etc. Using the list verb, you can retrieve information about a resource from the repository database.Here is an example which retrieves the list of administrators within EM.Standard mode$ emcli list -resource="Administrators" Interactive modeemcli>list(resource="Administrators")The output will be the same as standard mode.Standard mode$ emcli @myAdmin.pyEnter password :  ******The output will be the same as standard mode.Contents of myAdmin.py scriptlogin()print list(resource="Administrators",jsonout=False).out()To get a list of all available resources use$ emcli list -helpWith every release of EM, more resources are being added to the list verb. If you have a resource which you feel would be valuable then go ahead and contact Oracle Support to log an enhancement request with product development. Be sure to say how the resource is going to help improve your daily tasks. 2. Consistent Formatting:It is possible to format the output of any resource consistently using these options:  –column  This option is used to specify which columns should be shown in the output. Here is an example which shows the list of administrators and their account status$ emcli list -resource="Administrators" -columns="USER_NAME,REPOS_ACCOUNT_STATUS" To get a list of columns in a resource use:$ emcli list -resource="Administrators" -help You can also specify the width of the each column. For example, here the column width of user_type is set to 20 and department to 30. $ emcli list -resource=Administrators -columns="USER_NAME,USER_TYPE:20,COST_CENTER,CONTACT,DEPARTMENT:30"This is useful if your terminal is too small or you need to fine tune a list of specific columns for your quick use or improved readability.  –colsize  This option is used to resize column widths.Here is the same example as above, but using -colsize to define the width of user_type to 20 and department to 30.$ emcli list -resource=Administrators -columns="USER_NAME,USER_TYPE,COST_CENTER,CONTACT,DEPARTMENT" -colsize="USER_TYPE:20,DEPARTMENT:30" The existing standard EMCLI formatting options are also available in list verb. They are: -format="name:pretty" | -format="name:script” | -format="name:csv" | -noheader | -scriptThere are so many uses depending on your needs. Have a look at the resources and columns in each resource. Refer to the EMCLI book in EM documentation for more information.3. Search:Using the -search option in the list verb makes it is possible to search for a specific row in a specific column within a resource. This is similar to the sqlplus where clause. The following operators are supported:           =           !=           >           <           >=           <=           like           is (Must be followed by null or not null)Here is an example which searches for all EM administrators in the marketing department located in the USA.$emcli list -resource="Administrators" -search="DEPARTMENT ='Marketing'" -search="LOCATION='USA'" Here is another example which shows all the named credentials created since a specific date.  $emcli list -resource=NamedCredentials -search="CredCreatedDate > '11-Nov-2013 12:37:20 PM'"Note that the timestamp has to be in the format DD-MON-YYYY HH:MI:SS AM/PM Some resources need a bind variable to be passed to get output. A bind variable is created in the resource and then referenced in the command. For example, this command will list all the default preferred credentials for target type oracle_database.Here is an example$ emcli list -resource="PreferredCredentialsDefault" -bind="TargetType='oracle_database'" -colsize="SetName:15,TargetType:15" You can provide multiple bind variables. To verify if a column is searchable or requires a bind variable, use the –help option. Here is an example:$ emcli list -resource="PreferredCredentialsDefault" -help 4. Secure accessWhen list verb collects the data, it only displays content for which the administrator currently logged into emcli, has access. For example consider this usecase:AdminA has access only to TargetA. AdminA logs into EM CLIExecuting the list verb to get the list of all targets will only show TargetA.5. User defined SQLUsing the –sql option, user defined sql can be executed. The SQL provided in the -sql option is executed as the EM user MGMT_VIEW, which has read-only access to the EM published MGMT$ database views in the SYSMAN schema. To get the list of EM published MGMT$ database views, go to the Extensibility Programmer's Reference book in EM documentation. There is a chapter about Using Management Repository Views. It’s always recommended to reference the documentation for the supported MGMT$ database views.  Consider you are using the MGMT$ABC view which is not in the chapter. During upgrade, it is possible, since the view was not in the book and not supported, it is likely the view might undergo a change in its structure or the data in it. Using a supported view ensures that your scripts using -sql will continue working after upgrade.Here’s an example  $ emcli list -sql='select * from mgmt$target' 6. JSON output support    JSON (JavaScript Object Notation) enables data to be displayed in a collection of name/value pairs. There is lot of reading material about JSON on line for more information.As an example, we had a requirement where an EM administrator had many 11.2 databases in their test environment and the developers had requested an Administrator to change the lifecycle status from Test to Production which meant the admin had to go to the EM “All targets” page and identify the set of 11.2 databases and then to go into each target database page and manually changes the property to Production. Sounds easy to say, but this Administrator had numerous targets and this task is repeated for every release cycle.We told him there is an easier way to do this with a script and he can reuse the script whenever anyone wanted to change a set of targets to a different Lifecycle status. Here is a jython script which uses list and JSON to change all 11.2 database target’s LifeCycle Property value.If you are new to scripting and Jython, I would suggest visiting the basic chapters in any Jython tutorials. Understanding Jython is important to write the logic depending on your usecase.If you are already writing scripts like perl or shell or know a programming language like java, then you can easily understand the logic.Disclaimer: The scripts in this post are subject to the Oracle Terms of Use located here.  1 from emcli import *  2  search_list = ['PROPERTY_NAME=\'DBVersion\'','TARGET_TYPE= \'oracle_database\'','PROPERTY_VALUE LIKE \'11.2%\'']  3 if len(sys.argv) == 2:  4    print login(username=sys.argv[0])  5    l_prop_val_to_set = sys.argv[1]  6      l_targets = list(resource="TargetProperties", search=search_list,   columns="TARGET_NAME,TARGET_TYPE,PROPERTY_NAME")  7    for target in l_targets.out()['data']:  8       t_pn = 'LifeCycle Status'  9      print "INFO: Setting Property name " + t_pn + " to value " +       l_prop_val_to_set + " for " + target['TARGET_NAME']  10      print  set_target_property_value(property_records=      target['TARGET_NAME']+":"+target['TARGET_TYPE']+":"+      t_pn+":"+l_prop_val_to_set)  11  else:  12   print "\n ERROR: Property value argument is missing"  13   print "\n INFO: Format to run this file is filename.py <username>   <Database Target LifeCycle Status Property Value>" You can download the script from here. I could not upload the file with .py extension so you need to rename the file to myScript.py before executing it using emcli.A line by line explanation for beginners: Line  1 Imports the emcli verbs as functions  2 search_list is a variable to pass to the search option in list verb. I am using escape character for the single quotes. In list verb to pass more than one value for the same option, you should define as above comma separated values, surrounded by square brackets.  3 This is an “if” condition to ensure the user does provide two arguments with the script, else in line #15, it prints an error message.  4 Logging into EM. You can remove this if you have setup emcli with autologin. For more details about setup and autologin, please go the EM CLI book in EM documentation.  5 l_prop_val_to_set is another variable. This is the property value to be set. Remember we are changing the value from Test to Production. The benefit of this variable is you can reuse the script to change the property value from and to any other values.  6 Here the output of the list verb is stored in l_targets. In the list verb I am passing the resource as TargetProperties, search as the search_list variable and I only need these three columns – target_name, target_type and property_name. I don’t need the other columns for my task.  7 This is a for loop. The data in l_targets is available in JSON format. Using the for loop, each pair will now be available in the ‘target’ variable.  8 t_pn is the “LifeCycle Status” variable. If required, I can have this also as an input and then use my script to change any target property. In this example, I just wanted to change the “LifeCycle Status”.  9 This a message informing the user the script is setting the property value for dbxyz.  10 This line shows the set_target_property_value verb which sets the value using the property_records option. Once it is set for a target pair, it moves to the next one. In my example, I am just showing three dbs, but the real use is when you have 20 or 50 targets. The script is executed as:$ emcli @myScript.py subin Production The recommendation is to first test the scripts before running it on a production system. We tested on a small set of targets and optimizing the script for fewer lines of code and better messaging.For your quick reference, the resources available in Enterprise Manager 12.1.0.4.0 with list verb are:$ emcli list -helpWatch this space for more blog posts using the list verb and EM CLI Scripting use cases. I hope you enjoyed reading this blog post and it has helped you gain more information about the list verb. Happy Scripting!!Disclaimer: The scripts in this post are subject to the Oracle Terms of Use located here. Stay Connected: Twitter | Facebook | YouTube | Linkedin | Newsletter mt=8">Download the Oracle Enterprise Manager 12c Mobile app

    Read the article

  • MSDeploy: error using runCommand provider to call remote .cmd file (timeout)

    - by mjw.
    We are running into an error when trying to use the MSDeploy "runCommand" provider to execute a .cmd file on a remote machine. The expected run time should be about 10 seconds, but MSDeploy only runs it for about 2-3 seconds, after which time error details are returned. Here is the complete MSDeploy "runCommand" command line text I am using: msdeploy.exe -verb:sync -source:runCommand="D:\web deploy tester\test_cmd.cmd",dontUseCommandExe=false,waitAttempts=5,waitInterval=1000 -dest:auto,computername=http://test-machine:89/MsDeployAgentService/,userName=aaa,password=bbb Here are the error details returned: Error 'Error: (4/21/2010 12:19:25 PM) An error occurred when the request was processed on the remote computer. Error: The process 'C:\WINDOWS\system32\cmd.exe' (command line '/c "D:\web deploy tester\test_cmd.cmd"') was terminated because it exceeded the wait time. Error count: 1. ' occurred in call to RunCommand Any ideas as to why this is happening and how to resolve it?

    Read the article

  • Migrating JSF 1.2 to 2.x resulted in java.lang.NullPointerException at com.sun.faces.config.InitFacesContext.cleanupInitMaps

    - by nudastack
    I'm in process of migrating a JSF 1.2 application to JSF 2.x. My application is using the following APIs: maven-jetty-plugin (version 6.1.10) maven-compiler-plugin (version 2.3.1) myfaces-api & -impl (version 1.2.7) jsf-facelets (version 1.1.14) richfaces (version 3.3.3) javax servlet jstl (version 1.2) javax servlet-api (version 2.5) and some other plugins as well. I updated the versions in faces-config.xml file and removed Facelets 1.1.14 libs and replaced MyFaces 1.2 by JSF 2.x libs, however the application didn't work. It threw the following exception: Could not instantiate listener org.apache.myfaces.webapp.StartupServletContextListener java.lang.ClassNotFoundException: org.apache.myfaces.webapp.StartupServletContextListener at org.codehaus.plexus.classworlds.strategy.SelfFirstStrategy.loadClass(SelfFirstStrategy.java:50) at org.codehaus.plexus.classworlds.realm.ClassRealm.loadClass(ClassRealm.java:244) at org.codehaus.plexus.classworlds.realm.ClassRealm.loadClass(ClassRealm.java:230) at org.mortbay.jetty.webapp.WebAppClassLoader.loadClass(WebAppClassLoader.java:375) at org.mortbay.jetty.webapp.WebAppClassLoader.loadClass(WebAppClassLoader.java:337) at org.mortbay.jetty.handler.ContextHandler.loadClass(ContextHandler.java:1035) at org.mortbay.jetty.webapp.WebXmlConfiguration.initListener(WebXmlConfiguration.java:629) at org.mortbay.jetty.webapp.WebXmlConfiguration.initWebXmlElement(WebXmlConfiguration.java:367) at org.mortbay.jetty.plus.webapp.AbstractConfiguration.initWebXmlElement(AbstractConfiguration.java:190) at org.mortbay.jetty.webapp.WebXmlConfiguration.initialize(WebXmlConfiguration.java:289) at org.mortbay.jetty.plus.webapp.AbstractConfiguration.initialize(AbstractConfiguration.java:133) at org.mortbay.jetty.webapp.WebXmlConfiguration.configure(WebXmlConfiguration.java:222) at org.mortbay.jetty.plus.webapp.AbstractConfiguration.configure(AbstractConfiguration.java:113) at org.mortbay.jetty.webapp.WebXmlConfiguration.configureWebApp(WebXmlConfiguration.java:180) at org.mortbay.jetty.plus.webapp.AbstractConfiguration.configureWebApp(AbstractConfiguration.java:96) at org.mortbay.jetty.plus.webapp.Configuration.configureWebApp(Configuration.java:124) at org.mortbay.jetty.webapp.WebAppContext.startContext(WebAppContext.java:1217) at org.mortbay.jetty.handler.ContextHandler.doStart(ContextHandler.java:510) at org.mortbay.jetty.webapp.WebAppContext.doStart(WebAppContext.java:448) at org.mortbay.jetty.plugin.Jetty6PluginWebAppContext.doStart(Jetty6PluginWebAppContext.java:110) at org.mortbay.component.AbstractLifeCycle.start(AbstractLifeCycle.java:39) at org.mortbay.jetty.handler.HandlerCollection.doStart(HandlerCollection.java:152) at org.mortbay.jetty.handler.ContextHandlerCollection.doStart(ContextHandlerCollection.java:156) at org.mortbay.component.AbstractLifeCycle.start(AbstractLifeCycle.java:39) at org.mortbay.jetty.handler.HandlerCollection.doStart(HandlerCollection.java:152) at org.mortbay.component.AbstractLifeCycle.start(AbstractLifeCycle.java:39) at org.mortbay.jetty.handler.HandlerWrapper.doStart(HandlerWrapper.java:130) at org.mortbay.jetty.Server.doStart(Server.java:222) at org.mortbay.component.AbstractLifeCycle.start(AbstractLifeCycle.java:39) at org.mortbay.jetty.plugin.Jetty6PluginServer.start(Jetty6PluginServer.java:132) at org.mortbay.jetty.plugin.AbstractJettyMojo.startJetty(AbstractJettyMojo.java:371) at org.mortbay.jetty.plugin.AbstractJettyMojo.execute(AbstractJettyMojo.java:307) at org.mortbay.jetty.plugin.AbstractJettyRunMojo.execute(AbstractJettyRunMojo.java:203) at org.mortbay.jetty.plugin.Jetty6RunMojo.execute(Jetty6RunMojo.java:184) at org.apache.maven.plugin.DefaultBuildPluginManager.executeMojo(DefaultBuildPluginManager.java:101) at org.apache.maven.lifecycle.internal.MojoExecutor.execute(MojoExecutor.java:209) at org.apache.maven.lifecycle.internal.MojoExecutor.execute(MojoExecutor.java:153) at org.apache.maven.lifecycle.internal.MojoExecutor.execute(MojoExecutor.java:145) at org.apache.maven.lifecycle.internal.LifecycleModuleBuilder.buildProject(LifecycleModuleBuilder.java:84) at org.apache.maven.lifecycle.internal.LifecycleModuleBuilder.buildProject(LifecycleModuleBuilder.java:59) at org.apache.maven.lifecycle.internal.LifecycleStarter.singleThreadedBuild(LifecycleStarter.java:183) at org.apache.maven.lifecycle.internal.LifecycleStarter.execute(LifecycleStarter.java:161) at org.apache.maven.DefaultMaven.doExecute(DefaultMaven.java:320) at org.apache.maven.DefaultMaven.execute(DefaultMaven.java:156) at org.apache.maven.cli.MavenCli.execute(MavenCli.java:537) at org.apache.maven.cli.MavenCli.doMain(MavenCli.java:196) at org.apache.maven.cli.MavenCli.main(MavenCli.java:141) at sun.reflect.NativeMethodAccessorImpl.invoke0(Native Method) at sun.reflect.NativeMethodAccessorImpl.invoke(Unknown Source) at sun.reflect.DelegatingMethodAccessorImpl.invoke(Unknown Source) at java.lang.reflect.Method.invoke(Unknown Source) at org.codehaus.plexus.classworlds.launcher.Launcher.launchEnhanced(Launcher.java:290) at org.codehaus.plexus.classworlds.launcher.Launcher.launch(Launcher.java:230) at org.codehaus.plexus.classworlds.launcher.Launcher.mainWithExitCode(Launcher.java:409) at org.codehaus.plexus.classworlds.launcher.Launcher.main(Launcher.java:352) at org.codehaus.classworlds.Launcher.main(Launcher.java:47) 2013-07-02 14:39:15.590::WARN: Unknown realm: default 2013-07-02 14:39:15.651::INFO: No Transaction manager found - if your webapp requires one, please configure one. SLF4J: Failed to load class "org.slf4j.impl.StaticLoggerBinder". SLF4J: Defaulting to no-operation (NOP) logger implementation SLF4J: See StaticLoggerBinder for further details. Tem 02, 2013 2:39:22 PM com.sun.faces.config.ConfigureListener contextInitialized INFO: Initializing Mojarra 2.2.0 ( 20130502-2118 https://svn.java.net/svn/mojarra~svn/tags/2.2.0@11930) for context '' Tem 02, 2013 2:39:22 PM com.sun.faces.config.ConfigureListener contextInitialized WARNING: JSF1059: WARNING! The com.sun.faces.verifyObjects feature is to aid developers not using tools. It shouldn''t be enabled if using an IDE, or if this application is being deployed for production as it will impact application start times. log4j:WARN No appenders could be found for logger (org.ajax4jsf.renderkit.ChameleonRenderKitFactory). log4j:WARN Please initialize the log4j system properly. log4j:WARN See #noconfig for more info. Tem 02, 2013 2:39:26 PM com.sun.faces.config.ConfigureListener contextInitialized SEVERE: Critical error during deployment: java.lang.NoSuchMethodError: org.mortbay.jetty.annotations.AnnotationParser.parseAnnotations(Lorg/mortbay/jetty/webapp/WebAppContext;Ljava/lang/Class;Lorg/mortbay/jetty/plus/annotation/RunAsCollection;Lorg/mortbay/jetty/plus/annotation/InjectionCollection;Lorg/mortbay/jetty/plus/annotation/LifeCycleCallbackCollection;)V at com.sun.faces.vendor.Jetty6InjectionProvider.inject(Jetty6InjectionProvider.java:93) at javax.faces.FactoryFinder.getImplGivenPreviousImpl(FactoryFinder.java:695) at javax.faces.FactoryFinder.getImplementationInstance(FactoryFinder.java:572) at javax.faces.FactoryFinder.access$500(FactoryFinder.java:140) at javax.faces.FactoryFinder$FactoryManager.getFactory(FactoryFinder.java:1120) at javax.faces.FactoryFinder.getFactory(FactoryFinder.java:379) at com.sun.faces.config.processor.FactoryConfigProcessor.verifyFactoriesExist(FactoryConfigProcessor.java:328) at com.sun.faces.config.processor.FactoryConfigProcessor.process(FactoryConfigProcessor.java:236) at com.sun.faces.config.ConfigManager.initialize(ConfigManager.java:435) at com.sun.faces.config.ConfigureListener.contextInitialized(ConfigureListener.java:214) at org.mortbay.jetty.handler.ContextHandler.startContext(ContextHandler.java:540) at org.mortbay.jetty.servlet.Context.startContext(Context.java:135) at org.mortbay.jetty.webapp.WebAppContext.startContext(WebAppContext.java:1220) at org.mortbay.jetty.handler.ContextHandler.doStart(ContextHandler.java:510) at org.mortbay.jetty.webapp.WebAppContext.doStart(WebAppContext.java:448) at org.mortbay.jetty.plugin.Jetty6PluginWebAppContext.doStart(Jetty6PluginWebAppContext.java:110) at org.mortbay.component.AbstractLifeCycle.start(AbstractLifeCycle.java:39) at org.mortbay.jetty.handler.HandlerCollection.doStart(HandlerCollection.java:152) at org.mortbay.jetty.handler.ContextHandlerCollection.doStart(ContextHandlerCollection.java:156) at org.mortbay.component.AbstractLifeCycle.start(AbstractLifeCycle.java:39) at org.mortbay.jetty.handler.HandlerCollection.doStart(HandlerCollection.java:152) at org.mortbay.component.AbstractLifeCycle.start(AbstractLifeCycle.java:39) at org.mortbay.jetty.handler.HandlerWrapper.doStart(HandlerWrapper.java:130) at org.mortbay.jetty.Server.doStart(Server.java:222) at org.mortbay.component.AbstractLifeCycle.start(AbstractLifeCycle.java:39) at org.mortbay.jetty.plugin.Jetty6PluginServer.start(Jetty6PluginServer.java:132) at org.mortbay.jetty.plugin.AbstractJettyMojo.startJetty(AbstractJettyMojo.java:371) at org.mortbay.jetty.plugin.AbstractJettyMojo.execute(AbstractJettyMojo.java:307) at org.mortbay.jetty.plugin.AbstractJettyRunMojo.execute(AbstractJettyRunMojo.java:203) at org.mortbay.jetty.plugin.Jetty6RunMojo.execute(Jetty6RunMojo.java:184) at org.apache.maven.plugin.DefaultBuildPluginManager.executeMojo(DefaultBuildPluginManager.java:101) at org.apache.maven.lifecycle.internal.MojoExecutor.execute(MojoExecutor.java:209) at org.apache.maven.lifecycle.internal.MojoExecutor.execute(MojoExecutor.java:153) at org.apache.maven.lifecycle.internal.MojoExecutor.execute(MojoExecutor.java:145) at org.apache.maven.lifecycle.internal.LifecycleModuleBuilder.buildProject(LifecycleModuleBuilder.java:84) at org.apache.maven.lifecycle.internal.LifecycleModuleBuilder.buildProject(LifecycleModuleBuilder.java:59) at org.apache.maven.lifecycle.internal.LifecycleStarter.singleThreadedBuild(LifecycleStarter.java:183) at org.apache.maven.lifecycle.internal.LifecycleStarter.execute(LifecycleStarter.java:161) at org.apache.maven.DefaultMaven.doExecute(DefaultMaven.java:320) at org.apache.maven.DefaultMaven.execute(DefaultMaven.java:156) at org.apache.maven.cli.MavenCli.execute(MavenCli.java:537) at org.apache.maven.cli.MavenCli.doMain(MavenCli.java:196) at org.apache.maven.cli.MavenCli.main(MavenCli.java:141) at sun.reflect.NativeMethodAccessorImpl.invoke0(Native Method) at sun.reflect.NativeMethodAccessorImpl.invoke(Unknown Source) at sun.reflect.DelegatingMethodAccessorImpl.invoke(Unknown Source) at java.lang.reflect.Method.invoke(Unknown Source) at org.codehaus.plexus.classworlds.launcher.Launcher.launchEnhanced(Launcher.java:290) at org.codehaus.plexus.classworlds.launcher.Launcher.launch(Launcher.java:230) at org.codehaus.plexus.classworlds.launcher.Launcher.mainWithExitCode(Launcher.java:409) at org.codehaus.plexus.classworlds.launcher.Launcher.main(Launcher.java:352) at org.codehaus.classworlds.Launcher.main(Launcher.java:47) 2013-07-02 14:39:26.652::WARN: Failed startup of context org.mortbay.jetty.plugin.Jetty6PluginWebAppContext@64dacd55{/,C:\Users\zmn\workspace\tracker-web\src\main\webapp} java.lang.RuntimeException: java.lang.NoSuchMethodError: org.mortbay.jetty.annotations.AnnotationParser.parseAnnotations(Lorg/mortbay/jetty/webapp/WebAppContext;Ljava/lang/Class;Lorg/mortbay/jetty/plus/annotation/RunAsCollection;Lorg/mortbay/jetty/plus/annotation/InjectionCollection;Lorg/mortbay/jetty/plus/annotation/LifeCycleCallbackCollection;)V at com.sun.faces.config.ConfigureListener.contextInitialized(ConfigureListener.java:273) at org.mortbay.jetty.handler.ContextHandler.startContext(ContextHandler.java:540) at org.mortbay.jetty.servlet.Context.startContext(Context.java:135) at org.mortbay.jetty.webapp.WebAppContext.startContext(WebAppContext.java:1220) at org.mortbay.jetty.handler.ContextHandler.doStart(ContextHandler.java:510) at org.mortbay.jetty.webapp.WebAppContext.doStart(WebAppContext.java:448) at org.mortbay.jetty.plugin.Jetty6PluginWebAppContext.doStart(Jetty6PluginWebAppContext.java:110) at org.mortbay.component.AbstractLifeCycle.start(AbstractLifeCycle.java:39) at org.mortbay.jetty.handler.HandlerCollection.doStart(HandlerCollection.java:152) at org.mortbay.jetty.handler.ContextHandlerCollection.doStart(ContextHandlerCollection.java:156) at org.mortbay.component.AbstractLifeCycle.start(AbstractLifeCycle.java:39) at org.mortbay.jetty.handler.HandlerCollection.doStart(HandlerCollection.java:152) at org.mortbay.component.AbstractLifeCycle.start(AbstractLifeCycle.java:39) at org.mortbay.jetty.handler.HandlerWrapper.doStart(HandlerWrapper.java:130) at org.mortbay.jetty.Server.doStart(Server.java:222) at org.mortbay.component.AbstractLifeCycle.start(AbstractLifeCycle.java:39) at org.mortbay.jetty.plugin.Jetty6PluginServer.start(Jetty6PluginServer.java:132) at org.mortbay.jetty.plugin.AbstractJettyMojo.startJetty(AbstractJettyMojo.java:371) at org.mortbay.jetty.plugin.AbstractJettyMojo.execute(AbstractJettyMojo.java:307) at org.mortbay.jetty.plugin.AbstractJettyRunMojo.execute(AbstractJettyRunMojo.java:203) at org.mortbay.jetty.plugin.Jetty6RunMojo.execute(Jetty6RunMojo.java:184) at org.apache.maven.plugin.DefaultBuildPluginManager.executeMojo(DefaultBuildPluginManager.java:101) at org.apache.maven.lifecycle.internal.MojoExecutor.execute(MojoExecutor.java:209) at org.apache.maven.lifecycle.internal.MojoExecutor.execute(MojoExecutor.java:153) at org.apache.maven.lifecycle.internal.MojoExecutor.execute(MojoExecutor.java:145) at org.apache.maven.lifecycle.internal.LifecycleModuleBuilder.buildProject(LifecycleModuleBuilder.java:84) at org.apache.maven.lifecycle.internal.LifecycleModuleBuilder.buildProject(LifecycleModuleBuilder.java:59) at org.apache.maven.lifecycle.internal.LifecycleStarter.singleThreadedBuild(LifecycleStarter.java:183) at org.apache.maven.lifecycle.internal.LifecycleStarter.execute(LifecycleStarter.java:161) at org.apache.maven.DefaultMaven.doExecute(DefaultMaven.java:320) at org.apache.maven.DefaultMaven.execute(DefaultMaven.java:156) at org.apache.maven.cli.MavenCli.execute(MavenCli.java:537) at org.apache.maven.cli.MavenCli.doMain(MavenCli.java:196) at org.apache.maven.cli.MavenCli.main(MavenCli.java:141) at sun.reflect.NativeMethodAccessorImpl.invoke0(Native Method) at sun.reflect.NativeMethodAccessorImpl.invoke(Unknown Source) at sun.reflect.DelegatingMethodAccessorImpl.invoke(Unknown Source) at java.lang.reflect.Method.invoke(Unknown Source) at org.codehaus.plexus.classworlds.launcher.Launcher.launchEnhanced(Launcher.java:290) at org.codehaus.plexus.classworlds.launcher.Launcher.launch(Launcher.java:230) at org.codehaus.plexus.classworlds.launcher.Launcher.mainWithExitCode(Launcher.java:409) at org.codehaus.plexus.classworlds.launcher.Launcher.main(Launcher.java:352) at org.codehaus.classworlds.Launcher.main(Launcher.java:47) Caused by: java.lang.NoSuchMethodError: org.mortbay.jetty.annotations.AnnotationParser.parseAnnotations(Lorg/mortbay/jetty/webapp/WebAppContext;Ljava/lang/Class;Lorg/mortbay/jetty/plus/annotation/RunAsCollection;Lorg/mortbay/jetty/plus/annotation/InjectionCollection;Lorg/mortbay/jetty/plus/annotation/LifeCycleCallbackCollection;)V at com.sun.faces.vendor.Jetty6InjectionProvider.inject(Jetty6InjectionProvider.java:93) at javax.faces.FactoryFinder.getImplGivenPreviousImpl(FactoryFinder.java:695) at javax.faces.FactoryFinder.getImplementationInstance(FactoryFinder.java:572) at javax.faces.FactoryFinder.access$500(FactoryFinder.java:140) at javax.faces.FactoryFinder$FactoryManager.getFactory(FactoryFinder.java:1120) at javax.faces.FactoryFinder.getFactory(FactoryFinder.java:379) at com.sun.faces.config.processor.FactoryConfigProcessor.verifyFactoriesExist(FactoryConfigProcessor.java:328) at com.sun.faces.config.processor.FactoryConfigProcessor.process(FactoryConfigProcessor.java:236) at com.sun.faces.config.ConfigManager.initialize(ConfigManager.java:435) at com.sun.faces.config.ConfigureListener.contextInitialized(ConfigureListener.java:214) ... 42 more How is this caused and how can I solve it? Here is the web.xml content: <context-param> <param-name>log4jConfigLocation</param-name> <param-value>WEB-INF/log4j.properties</param-value> </context-param> <welcome-file-list> <welcome-file>portal/html/index.xhtml</welcome-file> </welcome-file-list> <context-param> <param-name>org.apache.myfaces.ERROR_HANDLING</param-name> <param-value>false</param-value> </context-param> <context-param> <param-name>javax.faces.DEFAULT_SUFFIX</param-name> <param-value>.xhtml</param-value> </context-param> <!-- Special Debug Output for Development --> <context-param> <param-name>facelets.DEVELOPMENT</param-name> <param-value>false</param-value> </context-param> <context-param> <param-name>facelets.REFRESH_PERIOD</param-name> <param-value>2</param-value> </context-param> <context-param> <param-name>com.sun.faces.validateXml</param-name> <param-value>true</param-value> </context-param> <context-param> <param-name>com.sun.faces.verifyObjects</param-name> <param-value>true</param-value> </context-param> <context-param> <param-name>javax.faces.DATETIMECONVERTER_DEFAULT_TIMEZONE_IS_SYSTEM_TIMEZONE</param-name> <param-value>true</param-value> </context-param> <!-- Richfaces --> <context-param> <param-name>javax.faces.STATE_SAVING_METHOD</param-name> <param-value>server</param-value> </context-param> <context-param> <param-name>org.richfaces.SKIN</param-name> <param-value>tracker</param-value> </context-param> <!-- Richfaces end --> <context-param> <param-name>org.ajax4jsf.VIEW_HANDLERS</param-name> <param-value>com.sun.facelets.FaceletViewHandler</param-value> </context-param> <filter> <display-name>RichFaces Filter</display-name> <filter-name>richfaces</filter-name> <filter-class>org.ajax4jsf.Filter</filter-class> <init-param> <param-name>createTempFiles</param-name> <param-value>false</param-value> </init-param> <init-param> <param-name>maxRequestSize</param-name> <param-value>512000</param-value> </init-param> </filter> <filter-mapping> <filter-name>richfaces</filter-name> <servlet-name>Faces Servlet</servlet-name> <dispatcher>REQUEST</dispatcher> <dispatcher>FORWARD</dispatcher> <dispatcher>INCLUDE</dispatcher> </filter-mapping> <listener> <listener-class>org.apache.myfaces.webapp.StartupServletContextListener</listener-class> </listener> <listener> <listener-class>com.omega.src.util.InitializeListener</listener-class> </listener> <servlet> <servlet-name>Faces Servlet</servlet-name> <servlet-class>javax.faces.webapp.FacesServlet</servlet-class> <load-on-startup>1</load-on-startup> </servlet> <servlet-mapping> <servlet-name>Faces Servlet</servlet-name> <url-pattern>*.xhtml</url-pattern> </servlet-mapping> <welcome-file-list> <welcome-file>loginPage.xhtml</welcome-file> </welcome-file-list> <login-config> <auth-method>BASIC</auth-method> </login-config> and dependencies in pom.xml content: <dependencies> <dependency> <groupId>com.sun.faces</groupId> <artifactId>jsf-api</artifactId> <version>2.2.0</version> </dependency> <dependency> <groupId>com.sun.faces</groupId> <artifactId>jsf-impl</artifactId> <version>2.2.0</version> </dependency> <dependency> <groupId>javax.servlet</groupId> <artifactId>jstl</artifactId> <version>1.2</version> </dependency> <dependency> <groupId>javax.servlet</groupId> <artifactId>servlet-api</artifactId> <version>2.5</version> </dependency> <dependency> <groupId>org.richfaces.framework</groupId> <artifactId>richfaces-api</artifactId> <version>3.3.3.Final</version> </dependency> <dependency> <groupId>org.richfaces.framework</groupId> <artifactId>richfaces-impl</artifactId> <version>3.3.3.Final</version> </dependency> <dependency> <groupId>org.richfaces.ui</groupId> <artifactId>richfaces-ui</artifactId> <version>3.3.3.Final</version> </dependency> <dependency> <groupId>log4j</groupId> <artifactId>log4j</artifactId> <version>1.2.16</version> </dependency> <dependency> <groupId>org.hibernate</groupId> <artifactId>hibernate-core</artifactId> <version>3.6.8.Final</version> </dependency> <dependency> <groupId>postgresql</groupId> <artifactId>postgresql</artifactId> <version>9.1-901.jdbc4</version> </dependency> <dependency> <groupId>javassist</groupId> <artifactId>javassist</artifactId> <version>3.12.1.GA</version> </dependency> <dependency> <groupId>com.googlecode.gmaps4jsf</groupId> <artifactId>gmaps4jsf-core</artifactId> <version>1.1.4</version> </dependency> <dependency> <groupId>net.sf.jasperreports</groupId> <artifactId>jasperreports</artifactId> <version>4.5.0</version> </dependency> <dependency> <groupId>net.sourceforge.jexcelapi</groupId> <artifactId>jxl</artifactId> <version>2.6.12</version> </dependency> </dependencies>

    Read the article

  • date renderer issue in extjs

    - by harsh
    Hi, I have a date renderer issue for a column. when browser language is in english the date is displayed in this format 09/14/2009 09:23 AM But when i change the browser language to german(or any other language except english) the date is not rendered it displays NAN/NAN/NAN 12:NAN PM Here is the code.. var dateRenderer = Ext.util.Format.dateRenderer('m/d/Y h:i A'); var colModel = new Ext.grid.ColumnModel([ { header: xppo.st('SDE_DATE_OCCURRED'), width: 75, sortable: true, dataIndex: 'DateOccurred', renderer: dateRenderer } ]); How can i render the date in other languages.Please help me with this issue. Thanks

    Read the article

  • Why do I get a connection error / timeout when using python suds to connect to Microsoft CRM?

    - by Chris R
    When I try to connect to an MS CRM web service using suds/python-ntlm, I am getting a timeout on requests. However, the code that I'm trying to replace -- which calls out to the cURL command line app to do the same call -- succeeds. Clearly something is different in the way that cURL is sending the command data, but I'll be damned if I know what the difference is. Below are the full details of the various calls. Anyone got any tips? Here's the code that is making the request, followed by the output. The cURL command code is below that, and its response follows. Hosts, users, and passwords have been changed to protect the innocent, of course. wsdl_url = 'https://client.service.host/MSCrmServices/2007/MetadataService.asmx?WSDL' username = r'domain\user.name' password = 'userpass' from suds.transport.https import WindowsHttpAuthenticated from suds.client import Client import logging logging.basicConfig(level=logging.INFO) logging.getLogger('suds.client').setLevel(logging.DEBUG) logging.getLogger('suds.transport').setLevel(logging.DEBUG) ntlmTransport = WindowsHttpAuthenticated(username=username, password=password) metadata_client = Client(wsdl_url, transport=ntlmTransport) request = metadata_client.factory.create('RetrieveAttributeRequest') request.MetadataId = '00000000-0000-0000-0000-000000000000' request.EntityLogicalName = 'opportunity' request.LogicalName = 'new_typeofcontact' request.RetrieveAsIfPublished = 'false' attr = metadata_client.service.Execute(request) print attr Here's the output: DEBUG:suds.client:sending to (http://client.service.host/MSCrmServices/2007/MetadataService.asmx) message: <SOAP-ENV:Envelope xmlns:ns0="http://schemas.xmlsoap.org/soap/envelope/" xmlns:ns1="http://schemas.microsoft.com/crm/2007/WebServices" xmlns:xsi="http://www.w3.org/2001/XMLSchema-instance" xmlns:SOAP-ENV="http://schemas.xmlsoap.org/soap/envelope/"> <SOAP-ENV:Header/> <ns0:Body> <ns1:Execute> <ns1:Request xsi:type="ns1:RetrieveAttributeRequest"> <ns1:MetadataId>00000000-0000-0000-0000-000000000000</ns1:MetadataId> <ns1:EntityLogicalName>opportunity</ns1:EntityLogicalName> <ns1:LogicalName>new_typeofcontact</ns1:LogicalName> <ns1:RetrieveAsIfPublished>false</ns1:RetrieveAsIfPublished> </ns1:Request> </ns1:Execute> </ns0:Body> </SOAP-ENV:Envelope> DEBUG:suds.client:headers = {'SOAPAction': u'"http://schemas.microsoft.com/crm/2007/WebServices/Execute"', 'Content-Type': 'text/xml'} DEBUG:suds.transport.http:sending: URL:http://client.service.host/MSCrmServices/2007/MetadataService.asmx HEADERS: {'SOAPAction': u'"http://schemas.microsoft.com/crm/2007/WebServices/Execute"', 'Content-Type': 'text/xml', 'Content-type': 'text/xml', 'Soapaction': u'"http://schemas.microsoft.com/crm/2007/WebServices/Execute"'} MESSAGE: <SOAP-ENV:Envelope xmlns:ns0="http://schemas.xmlsoap.org/soap/envelope/" xmlns:ns1="http://schemas.microsoft.com/crm/2007/WebServices" xmlns:xsi="http://www.w3.org/2001/XMLSchema-instance" xmlns:SOAP-ENV="http://schemas.xmlsoap.org/soap/envelope/"> <SOAP-ENV:Header/> <ns0:Body> <ns1:Execute> <ns1:Request xsi:type="ns1:RetrieveAttributeRequest"> <ns1:MetadataId>00000000-0000-0000-0000-000000000000</ns1:MetadataId> <ns1:EntityLogicalName>opportunity</ns1:EntityLogicalName> <ns1:LogicalName>new_typeofcontact</ns1:LogicalName> <ns1:RetrieveAsIfPublished>false</ns1:RetrieveAsIfPublished> </ns1:Request> </ns1:Execute> </ns0:Body> </SOAP-ENV:Envelope> ERROR: An unexpected error occurred while tokenizing input The following traceback may be corrupted or invalid The error message is: ('EOF in multi-line statement', (16, 0)) --------------------------------------------------------------------------- URLError Traceback (most recent call last) /Users/crose/projects/2366/crm/<ipython console> in <module>() /var/folders/nb/nbJAzxR1HbOppPcs6xO+dE+++TY/-Tmp-/python-67186icm.py in <module>() 19 request.LogicalName = 'new_typeofcontact' 20 request.RetrieveAsIfPublished = 'false' 21 ---> 22 attr = metadata_client.service.Execute(request) 23 print attr /Users/crose/virtualenv/advanis/lib/python2.6/site-packages/suds/client.pyc in __call__(self, *args, **kwargs) 537 return (500, e) 538 else: --> 539 return client.invoke(args, kwargs) 540 541 def faults(self): /Users/crose/virtualenv/advanis/lib/python2.6/site-packages/suds/client.pyc in invoke(self, args, kwargs) 596 self.method.name, timer) 597 timer.start() --> 598 result = self.send(msg) 599 timer.stop() 600 metrics.log.debug( /Users/crose/virtualenv/advanis/lib/python2.6/site-packages/suds/client.pyc in send(self, msg) 621 request = Request(location, str(msg)) 622 request.headers = self.headers() --> 623 reply = transport.send(request) 624 if retxml: 625 result = reply.message /Users/crose/virtualenv/advanis/lib/python2.6/site-packages/suds/transport/https.pyc in send(self, request) 62 def send(self, request): 63 self.addcredentials(request) ---> 64 return HttpTransport.send(self, request) 65 66 def addcredentials(self, request): /Users/crose/virtualenv/advanis/lib/python2.6/site-packages/suds/transport/http.pyc in send(self, request) 75 request.headers.update(u2request.headers) 76 log.debug('sending:\n%s', request) ---> 77 fp = self.u2open(u2request) 78 self.getcookies(fp, u2request) 79 result = Reply(200, fp.headers.dict, fp.read()) /Users/crose/virtualenv/advanis/lib/python2.6/site-packages/suds/transport/http.pyc in u2open(self, u2request) 116 return url.open(u2request) 117 else: --> 118 return url.open(u2request, timeout=tm) 119 120 def u2opener(self): /System/Library/Frameworks/Python.framework/Versions/2.6/lib/python2.6/urllib2.pyc in open(self, fullurl, data, timeout) 381 req = meth(req) 382 --> 383 response = self._open(req, data) 384 385 # post-process response /System/Library/Frameworks/Python.framework/Versions/2.6/lib/python2.6/urllib2.pyc in _open(self, req, data) 399 protocol = req.get_type() 400 result = self._call_chain(self.handle_open, protocol, protocol + --> 401 '_open', req) 402 if result: 403 return result /System/Library/Frameworks/Python.framework/Versions/2.6/lib/python2.6/urllib2.pyc in _call_chain(self, chain, kind, meth_name, *args) 359 func = getattr(handler, meth_name) 360 --> 361 result = func(*args) 362 if result is not None: 363 return result /System/Library/Frameworks/Python.framework/Versions/2.6/lib/python2.6/urllib2.pyc in http_open(self, req) 1128 1129 def http_open(self, req): -> 1130 return self.do_open(httplib.HTTPConnection, req) 1131 1132 http_request = AbstractHTTPHandler.do_request_ /System/Library/Frameworks/Python.framework/Versions/2.6/lib/python2.6/urllib2.pyc in do_open(self, http_class, req) 1103 r = h.getresponse() 1104 except socket.error, err: # XXX what error? -> 1105 raise URLError(err) 1106 1107 # Pick apart the HTTPResponse object to get the addinfourl URLError: <urlopen error [Errno 60] Operation timed out> The cURL command is: /opt/local/bin/curl --ntlm -u "domain\user.name:userpass" -k -d @- -A "Mozilla/4.0 (compatible; MSIE 6.0; Windows NT 5.2; SV1; .NET CLR 1.1.4322; .NET CLR 2.0.50727; .NET CLR 3.0.04506.648; .NET CLR 3.5.21022; InfoPath.1)" -H "Connection: Keep-Alive" -H "Content-Type: text/xml; charset=utf-8" -H "SOAPAction: http://schemas.microsoft.com/crm/2007/WebServices/Execute" https://client.service.host/MSCrmServices/2007/MetadataService.asmx The data that is piped to that cURL command: <soap:Envelope xmlns:soap="http://schemas.xmlsoap.org/soap/envelope/" xmlns:xsi="http://www.w3.org/2001/XMLSchema-instance" xmlns:xsd="http://www.w3.org/2001/XMLSchema"> <soap:Header> <CrmAuthenticationToken xmlns="http://schemas.microsoft.com/crm/2007/WebServices"> <AuthenticationType xmlns="http://schemas.microsoft.com/crm/2007/CoreTypes">0</AuthenticationType> <CrmTicket xmlns="http://schemas.microsoft.com/crm/2007/CoreTypes"></CrmTicket> <OrganizationName xmlns="http://schemas.microsoft.com/crm/2007/CoreTypes">CMIFS</OrganizationName> <CallerId xmlns="http://schemas.microsoft.com/crm/2007/CoreTypes">00000000-0000-0000-0000-000000000000</CallerId> </CrmAuthenticationToken> </soap:Header> <soap:Body> <Execute xmlns="http://schemas.microsoft.com/crm/2007/WebServices"> <Request xsi:type="RetrieveAttributeRequest"> <MetadataId>00000000-0000-0000-0000-000000000000</MetadataId> <EntityLogicalName>opportunity</EntityLogicalName> <LogicalName>new_typeofcontact</LogicalName> <RetrieveAsIfPublished>false</RetrieveAsIfPublished> </Request> </Execute> </soap:Body> </soap:Envelope> Here's the response: <?xml version="1.0" encoding="utf-8"?> <soap:Envelope xmlns:soap="http://schemas.xmlsoap.org/soap/envelope/" xmlns:xsi="http://www.w3.org/2001/XMLSchema-instance" xmlns:xsd="http://www.w3.org/2001/XMLSchema"> <soap:Body> <ExecuteResponse xmlns="http://schemas.microsoft.com/crm/2007/WebServices"> <Response xsi:type="RetrieveAttributeResponse"> <AttributeMetadata xsi:type="PicklistAttributeMetadata"> <MetadataId>101346cf-a6af-4eb4-a4bf-9c3c6bbd6582</MetadataId> <SchemaName>New_TypeofContact</SchemaName> <LogicalName>new_typeofcontact</LogicalName> <EntityLogicalName>opportunity</EntityLogicalName> <AttributeType> <Value>Picklist</Value> </AttributeType> <!-- stuff here --> </AttributeMetadata> </Response> </ExecuteResponse> </soap:Body> </soap:Envelope>

    Read the article

  • Cassandra inserts using Net::Cassandra::Easy in Perl

    - by knorv
    When using the Perl module Net::Cassandra::Easy to interface with Cassandra I use the following code to read colums col[123] from rows row[123] in column-family Standard1: my $cassandra = Net::Cassandra::Easy->new(keyspace => 'Keyspace1', server => 'localhost'); $cassandra->connect(); my $result = $cassandra->get(['row1', 'row2', 'row3'], family => 'Standard1', byname => ['col1', 'col2', 'col3']); This works as expected. However, when trying to insert row row1 with .. $result = $cassandra->mutate(['row1'], family => 'Standard1', insertions => { "col1" => "Value to set." }); .. I get the error message Can't use string ("0") as a SCALAR ref while "strict refs" in use at .../Net/GenThrift/Thrift/BinaryProtocol.pm line 376. What am I doing wrong?

    Read the article

< Previous Page | 77 78 79 80 81 82 83 84 85 86 87 88  | Next Page >